CLSU ABE Review2021 - Agricultural Machinery and Mechanization

You might also like

Download as pdf or txt
Download as pdf or txt
You are on page 1of 338

CENTRAL LUZON STATE UNIVERSITY

AGRICULTURAL AND BIOSYSTEMS ENGINEERING REVIEW CLASS 2021

AGRICULTURAL MACHINERY AND


MECHANIZATION

ENGR. ALEXIS T. BELONIO


MS, PAE, ASEAN Engineer

Agricultural Machinery and Mechanization CLSU ABE Review Class 2021


AGRICULTURAL AND BIOSYSTEMS ENGINEERING
STUDENT’S HANDBOOK AND REVIEWER

5
Agricultural
Machinery and
Mechanization
VOLUME

Alexis T. Belonio 2021 Edition


The Author:

Alexis T. Belonio is a Professional Agricultural Engineer and a registered ASEAN


Engineer. He finished his Bachelor of Science in Agricultural Engineering and Master
of Science degrees from CLSU. He is a Senior Research Fellow at the Philippine Rice
Research Institute, Science City of Munoz, Nueva Ecija. He served as Affiliate
Professor at the College of Engineering, Central Luzon State University (CLSU),
Science City of Munoz, Nueva Ecija and as Adjunct Assistant Professor at the
Departments of Agricultural and Chemical Engineering at the College of Engineering
and Agro-Industrial Technology, University of the Philippines at Los Banos. He was
former an Associate Professor at the Department of Agricultural Engineering and
Environmental Management, College of Agriculture, Central Philippine University,
Iloilo City. Since 1983, he has been deeply involved in teaching, research, technology
development, and entrepreneurial activities related to the field of agricultural
engineering. He served as a Reviewer for the Agricultural Engineering Board
Examination at TGIM, CLSU, BUCAF, CBSUA, SPAMAST, MSU-Gen San, BPSU, and MRI
Review Centers in the fields of Agricultural Power, Machinery and Allied Subjects as
well as Agricultural Processing, Structures and Allied Subjects.

He was awarded by the Philippine Society of Agricultural Engineers (PSAE) as the “Most
Outstanding Agricultural Engineer in the field of Farm Power and Machinery”, in 1993. In
that same year, he was named by the Professional Regulation Commission (PRC) as the
“Outstanding Professional in the field of Agricultural Engineering”. And in 1997, he was
awarded by the TOYM Foundation and the Jerry Roxas Foundation as the “Outstanding
Young Filipino (TOYF) in the field of Agricultural Engineering”. In 2008, he was awarded
as “Associate Laureate of the Rolex Awards for Enterprise ” (Geneva, Switzerland) and in
2010 as a “Laureate in Economic Business Development Category of The Tech Awards”
(San Jose, California). He was adjudged as one of the “Inspiring Modern-Day Filipino
Heroes” by Yahoo Southeast Asia in 2011 and one of the “25 Heroes-for-Better” by the
Western Union in 2015. Recently, he has received the “2016 ASEAN Outstanding
Engineering Achievement Awards” given by the ASEAN Federation of Engineering
Organizations (AFEO), recipient of AMY Intellectual Property Award given by Philippine
Chamber of Commerce and Industry in 2018, and as one of the recipients of 2019 Manila
Water Foundation, Inc. (MWFI) Prize for Engineering Excellence Award.

As a dedicated professional, he is technically assisting various agricultural companies in


the country and abroad. He has written and published several research and technical
papers, technical handbooks, including instructional and review materials.
Agricultural Machinery and
Mechanization
AGRICULTURAL AND BIOSYSTEMS ENGINEERING REVIEWER
VOLUME 5

by

Alexis T. Belonio, MS, PAE, ASEAN Engineer

Copyright © 2021

No part of this book is allowed to be photocopied or reproduced in any form


without any written permission from the author.
Acknowledgment/Prayer:

I am very much thankful to you God the Father, in the name of Jesus, and by the power
of Thine Holy Spirit for inspiring me to prepare this review material to help the
agricultural and biosystems engineering graduates as they prepare for the Professional
Agricultural and Biosystems Engineering Licensure Examination. May this review
material be instrumental to them on their pursuit of passing the board examination and
may You grant them the desire of their hearts in this endeavor!

All glory and honor belong to You Lord, in Jesus name, amen!
PREFACE
This book, Agricultural Engineering Student’s Handbook and Reviewer
Volume 5, was prepared primarily for the agricultural and biosystems
engineering graduates who are preparing for the professional agricultural
and biosystems engineering licensure examination. With this material, they
can be refreshed and updated on the new principles, systems, and
developments in agricultural and biosystems engineering. Also, they can
develop their analytical ability in analyzing the problems related to current
practices in agricultural and biosystems engineering particularly in
agricultural machinery and mechanization subject.

The contents of the book are short lecture material on each topic plus series
of questions and problems that compel the students to review the
fundamentals, theories, and concepts in agricultural and biosystems
engineering. Formula and samples problems are provided at the end of each
topic. The questions and problems are grouped into subtopics. Most
questions are practical in nature but tricky in nature to test whether students
have thorough understanding of the fundamental principles in each of the
different subject matters in agricultural machinery and mechanization used
ranging from production to post harvest and processing of agricultural crops
including their development, extension, and economics . The answers for
the questions and problems will be provided to the students as they pursue
the review proper so that they can grasp a deeper understanding and wider
know-how of the subject matter.

This book is still in draft form and is produced in limited copies only to those
who will attend the review classes. Additional items will be included in the
future to make this material more comprehensive. Comments and
suggestions are welcome for the improvement of this book.

May this book become useful to the you aspiring Agricultural and Biosystems
Engineers as you prepare for the Professional Board Examination!

God bless!

Alexis T. Belonio, MSc, ASEAN Engineer


Agricultural and Biosystem Engineer
Table of Contents

Chapter No. of
Pages
I INTRODUCTION 11
II MATERIALS OF CONSTRUCTION 18
III MANUFACTURING PROCESSES 18
IV DESIGNING AND DEVELOPING AGRICULTURAL MACHINES 7
V FUEL AND LUBRICANTS 10
VI PRIME MOVERS FOR AG MACHINERY 16
VII AGRICULTURAL MACHINERY MANAGEMENT 10
VIII TILLAGE EQUIPMENT 47
IX SEEDING AND PLANTING EQUIPMENT 25

X CULTIVATION AND WEEDING EQUIPMENT 10


XI SPRAYING AND DUSTING EQUIPMENT 16
XII HARVESTING EQUIPMENT 22
XIII THRESHING AND SHELLING EQUIPMENT 13

XIV CROP DRYING EQUIPMENT 20


XV RICE MILLING EQUIPMENT 19
XVI FEED MILLING AND MIXING EQUIPMENT 9
XVII CHOPPING, SHREDDING, AND DECORTICATING 9
EQUIPMENT
XVIII WATER PUMPING EQUIPMENT 27
XIX SELECTION OF AGRICULTURAL MACHINERY 7
XX SAFETY ON AGRICULTURAL MACHINERY 5
XXI PRODUCTION AND OPERATING COST ANALYSIS 12
I. INTRODUCTION

Agricultural Machinery – deals Agricultural Field Equipment –


with the study of the application, Equipment designed primarily for
use and management of various agricultural field operation. These
agricultural machines, tools and include agricultural tractors, self-
propelled machines, implements, and
equipment employed for the
their combinations.
production and post-production
operations. 1. Agricultural Tractor – Traction
Agricultural Mechanization – machine designed primarily to supply
develop-ment, manufacture and power to agricultural implements and
extension of appropriate farmstead equipment. It propels itself
agricultural tools, implements, and provides force to the direction of
and machinery using animal, travel to enable attached soil-engaging
human, mechanical, electrical, and and other agricultural implements to
other renewable sources of perform their intended function.
energy for agricultural production
and development as well as 2. Agricultural Implement -
Implement that is designed to perform
postharvest operations.
agricultural operations. These are
further classified into: (a) towed; (b)
Classifications of Agricultural mounted; and (c) semi-mounted
Machines implements.

1. Agricultural Field Equipment 3. Self-Propelled Machine –


a. Agricultural Field Implement Implement designed with integral
b. Agricultural Tractors power unit to provide both mobility and
c. Self-Propelled Machines power for performing agricultural
operations.
2. Farmstead Equipment
a. Livestock Equipment
Farmstead Equipment – Equipment
b. Waste Handling Equipment other than agricultural field equipment
c. Material Handling Equipment used in agricultural operation for the
d. Crop Drying and Storage production of food and fiber. These
Equipment include livestock equipment, waste
e. Others handling system, crop drying and milling
systems, material handling systems, and
others.
Classifications of Agricultural Field Power Source for Agricultural
Equipment According to Operation Machines and Equipment

1. Tillage Equipment - include plow 1. Agricultural Tractors - Machines


and harrow used to prepare the used primarily for traction or pulling
land to make it ready for seeding purposes. They can also deliver power
and planting operations. for both stationary and mobile jobs.
They can pull or tow implement
2. Crop Establishment Equipment – through traction from drive wheel and
used to place seeds or plant parts or pull implement through the draft
into the prepared seedbed and developed from the drawbar.
subsequently fertilize them.
2. Internal Combustion Engines –
3. Crop Maintenance Equipment – Power unit that supplies torque and
include cultivating, weeding, rotational speed to drive stationary or
spraying, dusting, as well as mobile agricultural machines using
irrigation equipment aimed to make petrol (gasoline or diesel) as fuel.
sure that the crop grows to produce
its potential yield. 3. Electric Motor – Power unit
commonly used for farmstead
4. Harvesting and Threshing equipment for driving machines such
Equipment - equipment used to as fans and blower, pumps, conveyors
gather and separate the produce or and many others using electricity as
harvest from the panicles and make energy source.
it ready for primary processing
operations. 4. Draft Animals – Domesticated
animals that provides power for pulling
5. Drying Equipment - equipment used small farm implements such as plow
to reduce the moisture of the and harrow. This includes horse, ox,
product to a level that it can be and bovine.
safely stored with minimum loss
from spoilage. 5. Human – Power provided with the
use of hand tools to do some tasks like
6. Milling Equipment - equipment used small vegetable farming operation,
to process the product ready for harvesting, weeding, etc.
consumption.

7. Storage Equipment - equipment


used to keep the product safe from
deterioration or spoilage for prolong
period of storage.
Agricultural Mechanization - develop- Mechanization Level
ment, manufacture, and extension of
appropriate agricultural tools, Philippine Center for Postharvest
implement, and machinery using animal, Development and Mechanization
human, mechanical, electrical, and other (PhilMech) reported that
renewable sources of energy for mechanization level of farms in
agricultural production, development, the Philippines is 1.23 horsepower
and post harvest operations. per hectare (hp/ha). Rice and corn
farms had the highest level of
Mechanized Agriculture - process of available farm power at 2.31 hp/ha.
using agricultural machinery to The area with high mechanization
mechanize the work of agriculture, level is Pangasinan while Kalinga
greatly increasing farm worker Apayao has low mechanization.
productivity. The effective mechaniza-
tion contributes to increased production
in two major ways: (1) timeliness of
operation and (2) good quality of work.

Agricultural mechanization or farm


mechanization - refers to the use
of agricultural equipment or farm
equipment before, during and after
agricultural or farming processes.
Effect of Farm Mechanization Declaration of Policy

1. Increased agricultural productivity 1. Promote the development and


and profitability adoption of modern, appropriate and
2. Shorter time of operations cost-effective, and environmentally-
3. Better quality of work safe agricultural and fishery machines
4. Efficient utilization of crop inputs and equipment.
2. Provide conducive environment to
• Undoubtedly, farm mechanization the local assembling and
displaced animal power from 60 to manufacturing of engines, machinery,
100% but resulted in less time for and equipment.
farm work. 3. Ensure quality and safety of locally-
manufactured and imported
machines and equipment.
Problems of Mechanization 4. Strengthen support services to
farmers in terms of credit faculties,
1. Limited access to credit research, training, and extension
and agricultural insurance programs.
2. Low farm mechanization 5. Unify, rationalize, and strengthen the
3. Inadequate postharvest facilities implementation as well as
4. Inadequate irrigation coordination of the mechanization
5. Limited support for research and program.
development (R&D), 6. Deliver integrated support services
6. Weak extension service to farmers, fisherfolks, and other
stakeholders and assist them to be
able to viably operate and manage
• AFMech Law - Republic Act 10601 their agricultural and fishery
or the Agricultural and Fisheries mechanization program.
Mechanization (AFMech) Law.
Approved in June 5, 2013,
published in June 14, 2013 and Bureau of Agricultural and Fishery
subsequently took effect on June Engineering (BAFE) – Created to
28, 2013. strengthen the agricultural and fishery
• The Law mandates the formulation engineering group and directly
of a comprehensive national policy supervised by DA Undersecretary.
on cost-effective and
environmentally-safe agriculture Divisions under BAFE
and fisheries to achieve food
security and increase farmers' 1. Engineering, Plans, Design and
income. Specifications.
2. Programs and Project Management
3. Standard Regulation and
Enforcement
4. Administration and Finance
BAFE Functions: Philippine Center for Postharvest and
Mechanization (PhilMech) – Coordinate,
1. Coordinate, oversee, and monitor the plan, and execute agriculture and fishery
national planning and implementation mechanization and postharvest RDE
of agri-fishery (AF) engineering, etc. programs and DA projects.
2. Assist in the national planning,
coordination, and implementation of Agri-Fisheries Machinery and Equipment
AF mechanization program. Service Centers - Business enterprise
3. Prepare, evaluate, validate, and under the AFMech Law aimed to provide
recommend engineering plans, etc. farmers after-sales service and warranty,
4. Coordinate and integrate all AF provide custom farm mechanization
engineering activities. services, repair and trouble shooting
5. Coordinate and monitor the services, as well as training on
enforcement of standards. maintenance and proper use of
6. Implement accreditation and agricultural machinery and equipment.
registration of manufacturers.
7. Implement accreditation and Agricultural and Fishery Machinery –
registration scheme for AF machinery. Machinery and equipment for the
8. Issue permit to operate to production, post production, and
manufacturers in coordination with processing , manufacture, transportation.
technology generators. and distribution of agricultural and fishery
9. Promulgate and implement products.
accreditation guidelines for testing
centers. Agricultural and Fishery Mechanization -
Development, adoption, assembly,
manufacture and application of
Bureau of Agriculture and Fisheries appropriate, location specific, and cost-
Product Standards (BAFPS) – Develops effective agricultural and fishery
standards for agriculture and fishery machinery.
machinery, tools and equipment in
coordination with accredited testing Agro-industrial Processing – Local
centers, and other concerned activities to maintain or raise the quality
government and private entities. or change the form or characteristics of
agricultural, fishery, and forestry products.

Agricultural Machinery Testing and


Evaluation Center (AMTEC) – Designated
as premier and reference testing center
in the country and for the purpose of
functional coordination and integration,
and closely coordinate its activities with
BAFPS and BAFE.
Acronyms: Terminologies

DA-BAFE - Department of Agriculture Arable land - land that is capable of


Bureau of Agricultural and being plowed and used to grow
Fishery Engineering crops.
DA-BAR – Department of Agriculture
Bureau of Agricultural Research Arid land - land with scarce water
DOST – Department of Science and resources where crop is difficult to
Technology grow.
DOE – Department of Energy
PRC BoAE – Board of Agricultural Fallow land – arable lang left
Engineering without sowingfor one or more
TESDA – Technical Education and Skill years to allow the land to recover
Development Authority and store organic matter while
PhilMech – Philippine Center for retaining moisture and disrupting
Postharvest Development and the life cycle of pathogens.
Mechanization
PhilRice –Philippine Rice Research Interplanting - practice of planting
Institute short-term annual crop with long-
AMTEC – Agricultural Machinery Testing term annual or biennial crop.
and Evaluation Center
LGU’s – Local Government Units Multiple Cropping - practice of
BAFS – Bureau of Agricultural and growing more than one crop on the
Fishery Standards same land in one year.
DOLE – Department of Labor and
Employment Mixed Cropping - practice of
DTI – Department of Trade and Industry growing two or more crops
simultaneously and intermingled
with no row arrangement.

Inter-Cropping - practice of
growing two or more crops in
alternate rows.

Inter Culture - practice of


cultivating one crop underneath a
perennial crop.
REFERENCES

AMDP. 1987. Handbook on


Agricultural Mechanization in the
Philippines. AMDP. College of
Engineering and Agro-Industrial
Technology. U.P. at Los Banos,
College, Laguna.

ASAE. ASAE Standards 1997.


American Society of Agricultural
Engineers. 2960 Niles RoadSt.
Joseph, MI. USA.

Republic Act No. 10601. Official


Gazette of the Republic of the
Philippines. 15pp

Suministrado, D. Status of
Agricultural Mechanization in the
Philippines. Paper presented at the
Regional Forum on Sustainable
Agricultural Mechanization in Asia
and Pacific. Quigdao, China. October
2013. 24pp.
QUESTIONS

4. Equipment designed primarily for


1. Application, use and
agricultural field operation.
management of various agricultural
a. Farmstead Equipment
machines, tools and equipment
b. Agricultural Implement
used for production and post-
c. Agricultural Field Equipment
production operations.
d. Self-Propelled Machine
a. Agricultural Mechanization
e. None of the above
b. Agricultural Machinery
c. Agricultural and Fishery
5. Traction machine designed
Machinery
primarily to supply power to
d. Agricultural and Food Machinery
agricultural implement and
e. None of the above
farmstead equipment.
a. Agricultural Tractor
2. Machinery and equipment for
b. Agricultural Implement
the production, harvesting,
c. Agricultural Field Equipment
processing, storage, manufacture,
d. Self-Propelled Machine
transportation and distribution of
e. None of the above
agricultural and fishery products.
a. Agricultural and Fisheries
6. Implement that is designed to
Machinery
perform agricultural operations and
b. Agricultural and Aquacultural
is further classified as towed,
Machinery
mounted and semi-mounted
c. Agricultural and food machinery
implements.
d. Agricultural Machinery
a. Agricultural Tractor
e. None of the above
b. Agricultural Implement
c. Agricultural Field Equipment
3. Utilization of hand tools,
d. Self-Propelled Machine
implements for draft animals, and
e. None of the above
mechanically-powered machines for
agricultural land development,
7. Level of mechanization of rice
production, harvesting and on-farm
and corn farms.
processing.
a. 0.92 horsepower per hectare
a. Agricultural development
b. 1.23 horsepower per hectare
b. Agricultural machinery and
c. 2.31 horsepower per hectare
equipment
d. None of the above
c. Agricultural mechanization
d. All of the above
8. Present farm mechanization level based 13. Business enterprise under the
on latest survey of PHILMECH. AFMech Law aimed to provide
a. 0.92 horsepower per hectare farmers after-sales service and
b. 1.23 horsepower per hectare warranty, provide custom farm
c. 2.31 horsepower per hectare mechanization services, and
d. None of the above services on repair and trouble
shooting, as well as training on
9. Area with high mechanization level. maintenance and proper use of
a. Nueva Ecija agricultural machinery and
b. Pangasinan equipment.
c. Iloilo a. Agri-fisheries Machinery and
d. None of the above Equipment Service Centers
b. Farmer ’s Machines Service
10. Area with low mechanization level. Centers
a. Kalinga c. Farm Machines Service Center
b. Nueva Viscaya d. None of the above
c. Zambales
d. None of the above 14. Current cost sharing scheme
on farm equipment between the
11. Act Promoting Agricultural and Department of Agriculture (DA)
Fisheries Mechanization Development in and qualified farmers’
the Philippines. organization.
a. Republic Act 10600 a. 50 : 50
b. Republic Act 10601 b. 85 : 15
c. Republic Act 10602 c. 15 : 85
d. All of the above a. None of the above

12. In order to strengthen the farm and the 15. Monitoring body for the
fishery groups of DA, the ____ was created service warranty provided by the
to prepare, evaluate and recommend assemblers, manufacturers,
engineering plans for farm and fisheries importers, suppliers, distributors,
machinery. and dealers of agri-fisheries
a. Agricultural Fisheries Mechanization machines under AFMech Law.
Committee a. LGU and BAFE
b. National Agro-fishery Mechanization b. AMTEC and BAFE
Program c. PSAE and MATEC
c. Bureau of Agricultural and Fisheries d. None of the above
Engineering
d. None of the above
16. Bureau Director and Assistant e. Unify, rationalize and strengthen
Director of BAFE were appointed by the implementation, coordination
the ____. of activities and mechanism on
a. DA Secretary agricultural and fisheries
b. President of the Philippines mechanization programs and
c. Board of Directors of BAFE projects.
d. None of the above
f. Deliver integrated support
17. Which of the following is a services to farmers and other
declaration of the policy of AFMech stakeholders, and assist them to be
Law? able to viably operate and manage
a. Promote the development and their agricultural and fisheries
adoption of modern, appropriate and mechanization projects.
cost-effective and environmentally- g. All of the above
safe agricultural and fisheries
machinery and equipment to enhance 18. Arable land is a land that is
farm productivity and efficiency in ____.
order to achieve food security and a. not capable of being tilled and
safety and increase farmer ’s income. planted
b. Provide a conducive environment b. capable of being plowed and
for local assembling and used to grow crops.
manufacturing of engines, machinery c. a dry land that needs irrigation.
and equipment for agricultural and d. None of the above
fisheries production, processing and
marketing. 19. Arid land is a land with ____.
c. Ensure the quality and safety of the a. scarce of water resources
locally manufactured or imported where crop is difficult to grow.
machinery and equipment by b. sufficient supply of water for
strengthening regulation through the crop growth.
development and enforcement of c. water logged condition where
machinery and machine performance crops is difficult to grow.
standards, regular testing and d. None of the above
evaluation, registration, and the
accreditation and classification of 20. Practice of growing more than
suppliers, assemblers and one crop on the same land in one
manufacturers to ensure compliance year.
to prescribe quality standards. a. Mixed crop
d. Strengthen support services such b. Multiple cropping
as credit facilities, research, training c. Inter-cropping
and extension programs, rural d. None of the above
infrastructure, and marketing
services.
21. Practice of growing two or more
crops simultaneously and
intermingled with no row
arrangement.
a. Mixed cropping
b. Inter cropping
c. Multiple cropping
d. None of the above

22. Practice of growing two or more


crops in alternate rows.
a. Inter-planting
b. Inter-cropping
c. Inter-culture
d. All of the above

23. Practice of cultivating one crop


underneath a perennial crop.
a. Inter-planting
b. Inter-cropping
c. Inter-culture
d. All of the above
II. ENGINEERING MATERIALS

• Durability, service, and cost of Manufacturing Metals


agricultural machines depend largely on
the kind and quality of materials used in •Metallic ore is mined beneath the earth
manufacturing the machine. surface. Iron is produced from iron ore
• Machines fabricated from substandard while copper is produced from copper ore.
materials are usually easy to wear off Aluminum is produced from an ore called
and have short life span. bauxite.
• Heavy materials usually affects power •A blast furnace is used to produce metal
requirement and performance of the by passing a hot gases to the furnace that
machine. is mixed with iron ore, limestone, and coke
• Engineers should have a knowledge of at about 3000°C.
the physical characteristics, selective and •Melted metal is separated into a ladle for
effective utilization of materials and cooling to form pig iron.
their processing techniques to avoid •Pigs are then melted and poured into
wastes and spoilage due to misuse of mold to make iron, steel, and cast iron.
materials.

Classifications Metal Characteristics

• Metals ▪Hard
• Ferrous - Iron and steel ▪Easy to shape
• Cast Iron, Carbon Steel, Alloy Steel, ▪High melting temperature
etc. ▪Low specific heat
• None- Ferrous ▪Good electrical conductivity
• Aluminum, copper, magnesium, etc. ▪Good thermal conductivities
• Non-Metal ▪Ability to be deformed without fracture
• Inorganic materials
• Glass, ceramics, plastics, etc.
• Organic Materials
• Wood, rubber, leather, canvass, etc.
Classifications of Metal

Pure Metal - single element that are not


combined with any other chemical
element. They are too soft, low in
strength, or low in some other desired
property to be used in any commercial
applications.

Alloy - mixture of two or more metals to


produce new metal. Example are
stainless steel, bronze, etc. • Soft-Centered Steel – durable to shock
because the mild steel in the center
Iron - pure ferrite. Two general types are: layer is deformable and has higher wear
• Pig iron – produced from ore of resistance than hard steel at the outer
hematite or magnetite in the blast side. They are usually used for
furnace by melting with limestone and moldboard and shear of a plow.
coke. • Low-Carbon Steel – with carbon content
• Cast Iron – with carbon content of 1.7 not exceeding 0.25%.
to 6/7%. They are low cost, good • Medium-Carbon Steel – contains 0.25 to
casting property, high compressive 0.50% carbon. They are usually used in
strength, high wearing resistance, making structural and machinery steel.
brittle and lower tensile strength. • High-Carbon Steel – carbon content is
above 0.50%. They are usually used in
the manufacture of spring and tool steel.
• Mild Steel (MS) – contains carbon of
0.15 to 0.25%. They are malleable and
easy to cut and weld.
• Cold-Rolled Steel (CRS) – medium
carbon steel which are used for parts
and components of machines requiring
greater strength and hardness, such as
shafting and connecting rods.
• Stainless Steel (SS) – 1% Ni, 11-14% Cr,
0.6% Mn, 0.6% Si. They have high
resistance to corrosion and oxidation.
Steel - molten steel that is cast into an They are used for valves, nozzles, and
ingot and then rolled, forged, hammered, dairy machines.
pressed, or machined into desired shape.
Structural steel, steel bars and cylinder,
and steel plate are commonly used as
machine elements and frame.
Typical Properties of Iron and Steel • Tubular Products – include all hollow
Material % Tensile % Brinell metal shapes such as pipes and tubes. They
Carbon Strength elonga hard-ness are available in round, square, and
(Approx (kg/cm) -tion Number
) in 5 rectangular shapes. Specifications are given
cm in terms of shape, schedule number, size,
Iron and length.
0.01 2800 40 50-90
(99.97%)

Soft Steel 0.10 3500 35 120

Structural
0.25 4200 30 150
Steel
Machiner
0.40 5600 25 180
y Steel
Spring
0.75 7000 12 240
Steel

Tool Steel 0.90 9100 8 260

Shapes of Metal Bars and Sheets


Metal Shapes

• Sheets, Strips, and Plates


• Sheet - piece of metal which has been
rolled into a sheet of 3/16 in. or less in
thickness. Commonly available sizes are 3’
wide x 6’ long and 4’ wide x 8’ long. The
thickness is expressed in terms of gauge
number which is equivalent to number of
sheets in 1in. thick pile of metal sheet.
• Strips – long sheet of metal that is less
than 12in. wide. Structural Shapes of Bars
• Plate – metal with thickness over 3/16
in. Commonly available size is 4’ wide x 8’
long. Thickness are given in terms of mm
or in inch.
• Metal Bars – made of different shapes
and are usually available in standard length
of 20 ft. or 6 m. They are purchased in
terms of quantity, shape, size, and kind of
metal used.
• Structural Shapes – commonly used in Cold-Rolled Steel Bar Shapes
the construction of agricultural machines.
They are specified in terms of quantity
wanted, the kind of material and shape,
size, and length of each piece.
Non-Ferrous - includes all the metals such
as copper, aluminum, magnesium, and
zinc in which iron is not present in large
amount.

Non-Metallic Materials
▪Wood
▪Plastics
▪Rubber
▪Ceramics
▪Fibers
▪Glass

Rubber - material obtained from rubber


tree.
•Capable to withstand extreme
deformability with more or less complete
recovery upon removal of deforming
force.
•Engineering applications:
• shock, noise, and vibration control
• Sealing
• Corrosion protection
• Friction production
• Electrical and thermal insulation
• Waterproofing
• Confining other materials,
• Load bearing

Plastics
•Large group of materials consisting of
combination of carbon and oxygen,
hydrogen, nitrogen, and other organic and
organic elements.
•Non-metallic material that can be
molded into shape. They are light in
weight, resistance to deterioration by
moisture, low elastic and thermal
conductivity, and good color range.
•They can be classified into
thermoplastics and thermo-setting
plastics.
Examples
▪Acrylic thermoplastics
▪Epoxy
▪Flouroplastics
▪Nylon
▪Phenolic Resin
▪Phenylene oxide
▪Polystyrene
▪Polyurethene
▪Polyvinyl chloride
▪Silicone

Engineering Ceramics
•Materials ranging from glass to furnace
brick.
•Include ceramic oxides, glass-ceramics,
carbides and nitrides.

Fibers
•The oldest engineering material that
includes jute, flax, and hemp.
•Usually used for engineered products
such as rope, cordage, nets, water hose,
and containers. Plant and animal fibers
are used for felt, paper, brushes, and
heavy structural cloth.
•This includes metal fibers, glass fibers,
and aramid fibers.

Glass
•A non-crystalline or amorphous solid.
•Most glass are made from silica, lime,
and sodium carbonate.
•Types include soda-lime glass,
Borosilicate glass, lead-alkali glass,
aluminosilicate glass, silica glass, and
fused silica.
Standard Materials

•Standard parts of agricultural machines


play a very important role in the
construction and performance of the
machine.
•They are especially manufactured and
mass produced to perform a specific
function.
•They can be readily purchased from
stores and hardware, and are available in
various sizes.

Bearings - used to hold parts of power


transmission in position.
• General Classes
• Sliding Bearing
• Rolling Bearing
• Ball Bearing
• Roller Bearing
• Tapered Roller
• Spherical
• Needle

Oil Seals - used to prevent oil or any fluid


from leaking though the bearing parts.

Pulley and Belt - used to transmit power


from a driver shaft to the driven shaft.
•Classified into: (a) flat belt, and (b) V-belt.

Sprocket and Chain - parts of the machine


that transmit power at higher torque but
at lower speed from a driver to the driven
shaft.
•Classification includes: (a) Hook link
chain, and (b) roller chain.
Gears - used in transmitting power when
the machine is compact and shafts are
placed close together.
• Classifications includes:
(a) spur (d) worm
(b) helical (e) spheroid
(c) bevel (f) etc.

Universal Joint and Flexible Shafts -


direct drive transmission system used to
transmit power to a misaligned shaft.

Cam - device that produces intermittent


motion or a specific motion to a member
called follower.
•Disc with a lobe on one side and when
rotate produces intermittent motion.

Bolts and Nuts - threaded fastener


intended to be mated with nut. They are
used to hold machine parts together.

Screws - externally threaded fastener


capable of being inserted into holes in
assemble parts of mating with a
preformed internal thread or forming its
own thread.
•Classification of screws:
• Set screw
• Cap screw
• Log screw
• Wood screw

Rivets - low-cost permanent fasteners


well suited to automatic assembly
operation.
•The primary reason for riveting is low
in-place cost.
•Classifications: (a) Tubular rivet, and (b)
Blind rivet.
Washers - used on the end either beneath
the head of the bolt or beneath the nut.
•They are used primarily as a seat to
distribute load in a fastener system.
• Classifications
• Flat or plain washer
• Lock washer
- Internal tooth
- External tooth
- Helical spring tooth

Pins - inexpensive and effective fastener


and are used when loading is in shear.
They are commonly used to fasten shaft in
some control linkages.
• Classifications are:
(a) hollow spring pin
(b) tapered Pin
(c) cotter pin
(d) “hair ” pin.

Retaining Rings - used to provide a


removable shoulder to accurately locate,
retain, or lock components on shaft or in
bores and housing.
REFERENCES

ASAE. 1997 ASAE Standards. Standard


Engineering Practice Data. ASAE USA.

PAES. Philippine Agricultural Engineering


Standards.

Moore, H.F. and M.B. Moore. Textbook


of the Material Engineering.
International Student Edition. McGraw-
Hill Book Company. Tokyo. 371pp.
QUESTIONS and PROBLEMS

1. Materials used in the fabrication of 6. Materials that are rust resistant.


agricultural machines. a. GI sheets
a. Standard b. BI sheets
b. Fabrication c. All of the above
c. All of the above d. None of the above
d. None of the above
7. Recommended material for the
2. Materials that are ready to adopt as manufacture of pin sleeves and hitch pins
component of agricultural machines such for walking-type power tillers.
as bearing, belts, pulley, etc. a. Mild steel
a. Standard b. Cold-rolled steel
b. Fabrication c. Spring steel
c. All of the above d. None of the above
d. None of the above
8. Material commonly used as axle for
3. Materials that need to be processed agricultural machines.
before it can be used as component of a. Cold-rolled steel shaft
agricultural machines such as bars, sheets, b. Malleable steel round bar
plates, etc. c. Round cast iron
a. Standard d. None of the above
b. Fabrication
c. All of the above 9. Kind of steel used for making tractor
d. None of the above drawn disk plow bottoms.
a. Tool steel
4. Property of a material to join together b. Soft-centered steel
with other metal at liquid state. c. Molybdenum steel
a. Ductility d. None of the above
b. Workability
c. Fusibility 10. Steel bar used as machine axle is ____.
d. None of the above a. hot-rolled steel
b. cold-rolled steel
5. Ability of a material to resist indention. c. mild steel
a. Brittleness d. None of the above
b. Toughness
c. Hardness 11. Steel bars commonly used in the
d. All of the above manufacture of agricultural machines.
a. Spring steel
b. Mild steel
c. Cold-rolled steel
d. None of the above
12. What is the rule-of-thumb in the 18. Mild steel with 0.15 to 0.23% carbon
amperage requirement of a welding rod? content (cold rolled) is designated by
a. 1-in. diameter per 1000 amp American Iron and Steel Institute as ____.
b. 0.001-in. diameter per 1 amp a. AISI 1021
c. 0.1-in. diameter per 100 amp b. AISI 1080
d. All of the above c. AISI 1045
d. None of the above
13. Which of the following metals is the
strongest? 19. Materials used in the manufacture of
a. Tool steel disc plows and disc blades for harrows.
b. Spring steel a. AISI 1020
c. Stainless steel b. AISI 1080
d. All of the above c. AISI 1045
d. None of the above
14. The higher the carbon content of the
steel, the ____ it becomes. 20. Steel with at least 0.80% carbon is
a. weaker designated by American Iron and Steel
b. stronger Institute as ____.
c. smaller a. AISI 1020
d. None of the above b. AISI 1080
c. AISI 1045
15. Standard length of bars available in d. None of the above
hardware.
a. 10 ft 21. Cold-rolled steel shaft with 0.37% to
b. 20 ft 0.44% carbon content is designated by
c. 30 ft American Iron and Steel Institute as ____.
d. None of the above a. AISI 1040
b. AISI 1020
16. When buying angle bars, the right c. AISI 1080
specification is ____. d. None of the above
a. thickness and width
b. thickness and length 22. ANSI means ____.
c. width and length a. American National Steel Institute
d. All of the above b. American National Standard Institute
c. American National Steel and Iron
17. Required material designation for the d. None of the above
construction of gear based on PAES
standard. 23. Material that is resistant to salt
a. AISI designation 1021 corrosion.
b. AISI designation 1045 a. Mild steel
c. AISI designation 1080 b. Cast iron
d. None of the above c. Stainless steel
d. All of the above
24. Stainless steel used for food industry. 30. Metal sheet with gauge number 14
a. SS series 400 means ____.
b. SS series 300 a. 14mm thick
c. SS series 200 b. 14 pieces of metal sheets in a stock
d. None of the above c. 14 pieces in one-inch thickness
d. None of the above
25. Stainless steel used for high-
temperature applications. 31. In a 2in. pile of sheet, gauge-20 metal
a. SS 104 sheet has ____.
b. SS 204 a. 20 sheets
c. SS 304 b. 40 sheets
d. None of the above c. 60 sheets
d. None of the above
26. Stainless steel used for architectural
purposes such as for rails for stairways. 32. Which of the following metal sheets
a. SS 102 is thicker than gauge 20?
b. SS 202 a. Gauge 18
c. SS 302 b. Gauge 22
d. None of the above c. Gauge 24
d. Two of the above
27. The higher the schedule specification
of a steel pipe, the lower is its ____. 33. The higher the gauge number of a
a. length metal sheet, the lower the ____.
b. nominal diameter a. thickness
c. thickness b. width
d. None of the above c. length
d. None of the above
28. If a metal has ⅛ in. thickness, it is
called as ____. 34. Type of paint most suitable for
a. strips metals.
b. sheet a. Latex paint
c. plate b. Enamel paint
d. All of the above c. Acrylic paint
d. None of the above
29. Material 4ft wide x 8ft long with
1/8in. thick is ____. 35. Which of the following materials is
a. metal sheet used for acrylic paint.
b. metal plate a. Paint thinner
c. metal strip b. Lacquer thinner
d. None of the above c. Primer epoxy
d. None of the above
36. Coating material used to protect 42. Bolt designated as M10x2 means ____
metals from rusting. in diameter
a. Primer epoxy a. 10 mm
b. Red oxide b. 12 mm
c. Red lead c. 20 mm
d. All of the above d. None of the above

37. Substance commonly used as coating 43. M10x2 is a ____ threaded bolt.
to protect iron against corrosion. a. 2-mm pitch thread
a. Tin b. 0.2-in. pitch thread
b. Zinc c. 0.002-in. pitch thread
c. Aluminum d. None of the above
d. All of the above
44. The bearing is designated as 6205.
38. Which of the following materials is What is the bore diameter of the bearing?
not corrosion resistant and is used in a. 20 mm
fertilizer applicator? b. 25 mm
a. Wood and plastic c. 0.5 mm
b. Stainless steel d. None of the above
c. All of the above
d. None of the above 45. What is the type of the bearing in Item
44?
39. Spherical roller bearing. a. Thrust bearing
a. 1206 b. Tapered-roller bearing
b. 2206 c. Single-row deep-groove ball bearing
c. 3206 d. Spherical-roller bearing
d. None of the above e. None of the above

40. Example of self-aligning ball bearing. 46. What series the bearing in Item 44
a. 1205 belong?
b. 2205 a. 100 series (extra light)
c. 3205 b. 200 series (light)
d. None of the above c. 300 series (medium)
d. 400 series (heavy)
41. Medium-carbon steel plate is e. None of the above
designated as AISI 1040, the carbon
content is ____. 47. Number C-300 sand paper is finer than
a. 10% ____.
b. 1% a. C-120
c. 40% b. C-400
d. 0.4% c. C-600
e. None of the above d. None of the above
48. Which of the following is not 54. Which of the following is not
considered as fabrication materials for considered as engineering plastic?
agricultural machines? a. Polystyrene
a. Angle bar b. PVC
b. BI sheet c. Polypropylene
c. Bolt, nuts, and washer d. Polyethylene
d. None of the above e. None of the above

49. Which of the following are standard 55. An easy to cut and weld steel
materials for agricultural machines? containing 0.15 to 0.25% carbon and is
a. Screws commonly used in the construction of
b. Bolts, nuts, and washer agricultural machinery.
c. Bearings a. Mild steel
d. All of the above b. Cold-rolled steel
c. Hot-rolled steel
50. Alloy is a mixture of ____ . d. None of the above
a. the same metal
b. two or more different metals 56. A steel material with high resistance to
c. one metal and the other is non-metal corrosion and oxidation containing 1% Ni,
d. None of the above 11-14% Cr, 0.6% Mn, and 0.6% Si and is
commonly used in food processing
51. Stainless steel is generally a mixture industry.
of ____. a. Mild steel
a. iron and chromium b. Food steel
b. iron and copper c. Stainless steel
c. iron and aluminum d. None of the above
d. None of the above
57. A group of plastic materials that has
52. Bronze is generally a mixture of____. better mechanical and/or thermal
a. copper and tin properties than the more widely used
b. copper and aluminum commodity plastic.
c. copper and iron a. Engineering plastic
d. None of the above b. Organic plastic
c. Inorganic plastic
53. Medium carbon steel (0.25 to 0.50% d. None of the above
carbon) which are commonly used for
parts of machines requiring greater 58. Steel classification containing 0.25 to
strength and hardness, such as axles and 0.50% carbon usually used in making
connecting rods. structural and machinery steel.
a. Mild steel a. Low carbon steel
b. Stainless steel b. Medium carbon steel
c. Cold-rolled steel c. High carbon steel
d. None of the above d. None of the above
59. Steel classification with carbon 64. Copper alloy containing 5-20% tin
content not exceeding 0.25% and is commonly used in the manufacture of
commonly used as a material in the machine parts such as bushing, spring,
fabrication of agricultural machines. pipe, fittings, sprayer and pump.
a. Low-carbon Steel a. Brass
b. Medium-carbon steel b. Bronze
c. High-carbon steel c. Kelmet
d. None of the above d. None of the above

60. Steel classification with carbon 65. Copper alloy containing 10 to 40% zinc
content of above 0.50% and is usually alloy commonly used in the manufacture
used in the manufacture of spring and of radiator, pipe, screen, and instrument
tool steel. parts.
a. Low carbon steel a. Brass
b. Medium carbon steel b. Bronze
c. High carbon steel c. Kelmet
d. None of the above d. None of the above

61. An iron material with 1.7 to 6.7% 66. Long sheet of metal less than 12in.
carbon content that is low cost and with wide commonly used in the manufacture
good casting property, high compressive of building materials.
strength, high wearing resistance, brittle a. Sheet
and lower tensile strength. b. Plate
a. Pure iron c. Strips
b. Pig iron d. None of the above
c. Cast iron
d. None of the above 67. Characteristics of metals that are not
acted upon by outside forces which
62. Iron produced in a blast furnace by include color, density, weight, and
melting ore of hematite or magnetite electrical and heat conductivity.
with limestone and coke. a. Physical
a. Pure iron b. Mechanical
b. Pig iron c. Chemical
c. Cast iron d. None of the above
d. None of the above
68. Characteristics of metals giving their
63. Copper alloy containing 20 to 40% composition and their reaction to other
lead commonly used as bearing for metals such as resistance to corrosion.
diesel engine. a. Physical
a. Brass b. Mechanical
b. Bronze c. Chemical
c. Kelmet d. None of the above
d. None of the above
69. Characteristics of metal exhibited 75. Material’s resistance to being
when outside forces are applied to them. compressed.
a. Physical a. Malleability
b. Mechanical b. Compressive strength
c. Chemical c. Toughness
d. None of the above d. None of the above

70. The greatest stress seen in the 76. Capability of a material to be shaped
material’s resistance to being pulled and formed.
apart and is sometimes called “ultimate a. Malleability
strength”. b. Compressive strength
a. Tensile strength c. Toughness
b. Yield strength d. None of the above
c. Compression strength
d. None of the above 77. Ability of a material to withstand
bending and twisting and a desirable
71. Classification of stress usually property for loader frame and for
designated at a 0.2% permanent crankshaft.
elongation. a. Malleability
a. Tensile strength b. Compressive strength
b. Yield strength c. Toughness
c. Compression strength d. None of the above
d. None of the above
78. Characterized by ease with which
72. Tendency of a material to break metals are machined or cut by a machine
under impact. tool.
a. Ductility a. Ductility
b. Hardness b. Machinability
c. Brittleness c. Toolability
d. None of the above d. None of the above

73. Indication of how much a material is 79. Property to which metal, when it is at
deformed as it is stretched. the liquid state, easily join with other
a. Ductility metals.
b. Hardness a. Fixability
c. Brittleness b. Connectivity
d. None of the above c. Fusibility
d. None of the above
74. The microscopic measurement of
metal that shows differences in sizes that 80. Material’s resistance to penetration
relates directly to strength. and permanent deformation.
a. Grain size a. Ductility
b. Particle size b. Hardness
c. Material size c. Brittleness
d. None of the above d. None of the above
81. Type of steel used in turning 86. The higher the schedule number of
machines when cutting metals. the pipe, the ____ is the pipe.
a. Mild steel a. bigger
b. Spring steel b. weaker
c. Tool steel c. stronger
d. None of the above d. None of the above

82. Steel used as shock absorber 87. Prevailing price of stainless steel
attached to the wheel and to the sheet.
thresher support frame in mobile rice a. Times 1 of the MS sheet of the same
thresher. dimension and thickness
a. Mild steel b. Times 2 of the MS sheet of the same
b. Spring steel dimension and thickness
c. Tool steel c. Times 3 of the MS sheet of the same
d. None of the above dimension and thickness
d. None of the above
83. Steel type commonly used as
pegtooth for axial-flow rice threshers. 88. Prevailing price of perforated sheet.
a. Mild steel a. Times 1 of the MS sheet of the same
b. Cold-rolled steel dimension and thickness.
c. Tool steel b. Times 2 of the MS sheet of the same
d. None of the above dimension and thickness.
c. Times 3 of the MS sheet of the same
84. Fabrication materials being sold by dimension and thickness.
construction suppliers such as angle d. None of the above
bars, round bars, pipes, etc. are usually
painted at the end of each piece for the 89. Screw type used to fix metal parts
purpose of ____. together.
a. making it attractive to the potential a. Self-tapping screw
buyers. b. Cap screw
b. preventing the material from rusting c. Lag screw
c. easy identification of similar stock d. None of the above
sizes
d. None of the above 90. Screw with shield to properly fix the
materials together.
85. Pipe schedule matters with the pipe a. Cap screw
____. b. Tex screw
a. diameter c. Self-tapping screw
b. length d. None of the above
c. thickness
d. None of the above
91. A steel material containing 0.035 to
0.4% carbon and is widely used for the
production of steel plates, structural
steels, bars, etc.
a. Mild steel
b. Hard steel
c. Soft-centered steel
d. All of the above

92. Machine element used to tighten


bolt.
a. Washer
b. Rivet
c. Nut
d. All of the above

93. Machine element used to keep


castellated nut from loosening.
a. Roller pin
b. Spring
c. Cotter pin
d. None of the above
III. MANUFACTURING PROCESSES

Manufacturing of agricultural machines Agricultural machinery manufactures and


in the Philippines begun as early as distributors were organized nationwide to
Spanish period when tools and small caters the need of the agriculture sector.
equipment are fabricated for farm use.

In early 60’s manufacturing was


concentrated on the fabrication of small
farm tools and attachment for four-
wheel tractors used in sugar plantation.

Locally made hand tractors started to


appear in 1972. The design of hand
tractor were adopted from
manufacturers in Bicol and at IRRI.
About 33 manufactures and enterprises
assembles farm machines.

In the middle of 70’s hand tractor


produce locally has captured 40% of the
market. Most of the manufacturers
fabricating hand tractors have the
capability in producing other farm
machines.

In 1984, the DA-IRRI Program for


Extension of Small Farm Equipment
reported that there were about 206
cooperating manufactures in the country
producing various agricultural machines.

In early 90’s, several agencies of DA


started the development of agricultural
machineries in the Philippines. This
includes PhilMech, PhilRice to name a
few. Cooperating manufacturers were
established nationwide.
Shearing
•This is done by placing a piece of metal
usually in the form of sheet, plate, strip, or
rod between two blades of knives.
•The machine used for metal shearing are
called metal shears.
•Bench shears are used for cutting heavy
sheet metal and metal plates.
•Tin shear or snip is used for cutting metal
sheets using hand.

Sawing
•It is the process of moving a hard metal
saw blade with teeth along its edge across a
metal to be cut.
•Hand Hacksaw – A manually operated saw
used for cutting metal by pushing and
pulling the blade on the metal being cut.
•Power Hacksaw – It is simply an
improvement of hand hack saw. It is an
extremely economical and efficient way of
cutting bigger sizes of metal bars.
•Blades of power hack saw are available
from 3 to ten teeth per inch.
•For mild steel, aluminum, and copper, 4 to
6 teeth per inch blades are used. For tubing
and small section 10 to 14 teeth per inch
are used.
•Stroke per minute ranged from 40 to 150
strokes per min. Soft metal uses high speed
while hard metal uses low speed.
•Bandsaw – It uses an endless blade which
move past the piece being cut in only one
direction.
•Cutting hard metals requires blades with
small teeth while soft materials requires
larger blades.
•The use of too coarse blade on hard metal
will cause the blade to become dull very
quickly. However, saw with very small teeth
requires longer time to cut metal.
•The rule is that at least two teeth should
be used when cutting a piece of metal.
Flame Cutting
•It is one of the fastest method of cutting
bars or steel plates having a thickness of
1/16 to 6 inches.
•In flame cutting the piece of steel must be
heated to its kindling temperature of
approx 1600°F.
•Acetylene gas, pure oxygen and a cutting
torch are usually used for flame cutting.

Plasma Cutter - Uses electricity and


compressor to carryout cutting of metals
especially stainless steel at a faster rate
compared to other cutting tools and
equipment.

•Wire Cutting
•Laser Cutting
Milling Processes
•Facing - It is the process of cutting or
squaring of the end of a piece of work.
•Center Drilling – It is done with a
combination drill and countersink.
•Straight Turning – The work is first placed
on the centers, and the cutting tool is set
for cutting.
•Drilling Reaming and Counter Boring –
These are commonly performed with a drill
press or by a lathe. In here the work piece
is held in a lathe chuck while the drill,
reamer, countersink, or counter boring tool
is held in the tailstock.
•Knurling – It is the process of making the
handle of hand tool rough in order to give a
better grip.
•Boring – It is the cutting and enlarging of a
round hole to make a more exact size and
accurate with its axis.
•Threading – It is the process of making a
threads on screw either right-hand, left-
hand, external, or internal threads.
WELDING PRCESSES
• Soldering
• Brazing
• Oxyacetylene Welding
• Arc Welding
• Resistance Welding
• MIG Welding
• TIG Welding

Soldering
• It is the common method of joining
metal sheet especially non-ferrous metal.
• Soldering is done at a relatively low
temperature of 427°C compared with
other methods such as brazing and arch
welding.

Brazing
• It is the joining technique of metal
using oxyacetylene welding equipment
by melting a non-ferrous filler rod at a
temperature above 427 C but below the
melting point of metal being joined.
• Brazing is much stronger joint
compared with soldering.

Oxyacetylene Welding
• It is the common for of gas welding
process using oxygen and acetylene
gases to provide heat to melt and join
metal by means of fusion.
• Regulator – It the special valve to
properly mix the oxygen and the
acetylene gas.
• Hose – This caries the gases from the
cylinder to torch. Read hose is used for
acetylene gas while the green hose is
used for the oxygen gas.
• Blow pipe or torch
Arc Welding
• Process of welding metal by passing
high current into a flux-coated welding
rod. The flux serves as shield to prevent
oxidation to produce stronger weld.
• The arc welding electrodes melt and
provide filler metal to the joint.

Resistance Welding
• Uses the heat generated by electric
current passing through a small area of
the metal being joined. The pressure
forces the heated area together until
they have fused.
• Spot welding is the common form of
resistance welding.

MIG Welding
• Metal inert gas (MIG) welding – uses
continuous roll-fed welding rod.
• An inert gas is used to protect the weld
from oxidation. For steel, it uses carbon
dioxide while for aluminum and other
metals, a mixture of argon and helium is
used.
• MIG welding is easier than arc welding
in terms of heat setting, rate of wire
feed, and feed, and mixture of gases.
• MIG welding machine uses welding
wire instead of welding rod. CO 2 gas is
used as gas shield for the material being
weld to minimize warping. It can be
used in welding mild steel, stainless
steel, etc.
TIG Welding
• Tungsten inert gas (TIG) welding -
widely used for welding aluminum,
stainless steel, and other non-ferrous
metals.
• It is similar to oxyacetylene welding in
that the filler rod is separate from the
torch and is fed with the opposite hand.
Classification of Manufacturer
According to Manufacturing Capability

• By Piece – The manufacturer


fabricates the machine by piece only or
one at a time. Usually one to two
persons are commissioned to do the
entire work to produce the machine.
The machine basically is directly
contracted from individual farmer or
user.

• By Batch – The manufacturer


fabricates the machine in several
number of pieces as per contract with
the dealer.

• Mass Production – The manufacturer


produces the machine beyond the
number of pieces produced in the batch
system. Even without direct order or
purchase from the customer or dealer,
the manufacturer still produce the
machine for possible unseen market.
Classification of Manufacturer According
to Size

• Small Manufacturer – Usually capable


of manufacturing agricultural machines
by piece only. They are usually backyard
fabrication in operation and basically use
common fabrication equipment such as
hand saw, welding machines, and some
power drill and sanders.

• Medium-Size Manufacturer – Capable


to operate and produce agricultural
machine by batch and do mass
production of small machines if ever
needed. They have more equipment and
facilities to do the manufacturing
operation than the small manufacturers
do. Special process needed to
manufacture a certain machine is done by
subcontracting to other machine shops.

• Large-Size Manufacturer – Capable of


mass producing agricultural machines
because they have almost complete line
of equipment needed in manufacturing.
Small and medium manufacturers
sometime request subcontracting job to
this class of manufacturers.
Classification of Agricultural Machinery
Manufacturing Industry

•Manufacturers – are those that


produces one or several types of farm
machinery and usually do not sell other
items from other manufacturers.

•Manufacturer-Dealers – manufactures
their own line of item but also sell either
consigned items or several brands of
single cylinder engines, pumps, pump
set, and other types of farm machinery.

•Dealers – carry several items and brand


from varying sources and are not usually
engaged in manufacture and fabrication.
REFERENCES

Kazanas H. C. and L. Hannah.


Manufacturing Processes: Metals.
Printice-Hall, Inc. Englewood Cliffs, New
Jersey. 120pp.

Manaligod, H. T. Agricultural machinery


Manufacturing and Distribution
Problems and Prospects. Philippine
Agricultural Engineering Journal.

Ong, D. C. Situational Analysis of the


Constraints of Manufacturing and
marketing of Postharvest Equipment.
Philippine Agricultural Engineering
Journal.

Repp, V. E. et. al. 1982. Metalwork.


Technology and Practice. McKnight
Publishing Company. Bloomington,
Illinois. 598pp.

Scheck, L. A. and G.C. Edmonton. 1976.


Practical Welding. Bruce and Glencde,
Inc. London. 150pp
QUESTIONS and ANSWERS

1. Method of treating metal by applying 7. Mixing of two or more substances one


welding deposit to strengthen the peg- of which is iron.
tooth of a rice thresher. a. Casting
a. Heat treatment b. Forging
b. Welding c. Alloying
c. Hard facing d. None of the above
d. None of the above
8. Gas-metallic arc welding that uses inert
2. Filing is classified as ____. gas shielding to produce a cleaner and
a. cold working sounder weld compared with that of
b. hot working conventional welding.
c. casting a. Gas welding
d. None of the above b. MIG welding
c. TIG welding
3. Heat treatment method to increase d. All of the above
the hardness of steel by rapid cooling.
a. Tempering 9. Widely used method of welding
b. Quenching aluminum, copper, stainless steel, and
c. Annealing other difficult-to-weld metals.
d. None of the above a. TIG welding
b. MIG welding
4. Process of making products by c. Gas welding
pouring melted metal into a mold and d. All of the above
then allowing it to cool.
a. Hot working process 10. Welding process commonly used for
b. Casting thin metal sheets.
c. Powder metallurgy a. Brazing
d. None of the above b. Soldering
c. Spot welding
5. Cutting of holes on a material by d. None of the above
means of shearing process.
a. Drilling 11. Welding process that uses an electrode.
b. Punching a. Gas welding
c. Boring b. Resistance welding
d. None of the above c. Arc welding
d. None of the above
6. Welding process wherein current is
allowed to pass through the prongs of 12. Tool that can cut metal bars faster.
the welding machine which joined two a. Shear cutter
metal sheets together. b. Band saw
a. Resistance welding c. Grinding wheel
b. Oxyacetylene welding d. None of the above
c. Arc welding
d. None of the above
13. Welding job is to be performed at the 18. C-300 sand paper is finer than ____.
engine drive of a multiple-pass rice mill. a. C-120
What would you recommend to prevent b. C-400
possible accident during the repair work? c. C-600
a. Remove the engine from the drive. d. None of the above
b. Disconnect the line from the terminal
of the battery. 19. If a welding rod is 1/8 in. in diameter
c. Remove the ground line of the E6013, the recommended amperage for the
welding machine from the engine drive. welding machine is ____.
d. None of the above a. 75 amp
b. 100 amp
14. Device suitable for cutting stainless c. 125 amp
steel plates. d. 150 amp
a. Shear cutter e. None of the above
b. Plasma cutter
c. Acetylene gas cutter 20. In what position does the electrode In
d. All of the above Item 19 can be used?
a. Flat
15. Machine tool used to reduce the b. Horizontal
diameter of a metal. c. Vertical
a. Router d. Overhead
b. Power drill e. All of the above
c. Lathe f. None of the above
d. None of the above
21. Which of the following position is not
16. Basic hand tools that must be possible to be weld with arc welding
provided by manufacturers to buyers of method.
engines. a. Flat
a. Open wrench and adjustable wrench b. Horizontal
b. Philip and flat-screw driver c. Overhead
c. Spark-plug wrench d. None of the above
d. All of the above
22. Producer of one or several types of farm
17. The welding rod electrode is machinery and usually does not sell other
designated as E6013, what is the tensile items from other manufacturers.
strength of the joint when welded a. Manufacturer
properly? b. Manufacturer-dealer
a. 120 ksi (kips per square inch) c. Dealer
b. 60 ksi d. None of the above
c. 30 ksi
d. None of the above
23. These producers manufacture their 28. Classification of manufacturers that
own line of items and sell together with fabricate only one machine at a time and
either consigned items or several brands usually has one to two workers
of single cylinder engines, pumps, pump commissioned to do the entire work to
set, and other types of farm machinery. produce the machine.
a. Manufacturer a. By mass production
b. Manufacturer-dealer b. By batch
c. Dealer c. By piece
d. None of the above d. None of the above

24. They carry several items and brand 29. Classification of manufacturers that
from varied sources and usually do not fabricate the machine in several number of
engaged in manufacture and fabrication. pieces as per contract with the dealer.
a. Manufacturer a. By mass production
b. Manufacturer-dealer b. By batch
c. Dealer c. By piece
d. None of the above d. None of the above

25. Manufacturer capable of mass 30. Classification of manufacturers that


producing agricultural machines for it has produces a machine beyond the number of
almost a complete line of equipment pieces produced in the batch system and
needed in manufacturing. still produce the machine, even without
a. Small-size manufacturer direct order or purchase from the customer
b. Medium-size manufacturer or dealer, for possible unseen market.
c. Large-size manufacturer a. By mass production
d. None of the above b. By batch
c. By piece
26. Manufacturing enterprise capable of d. None of the above
manufacturing agricultural machines by
piece only. 31. Which of the following organizations
a. Small-size manufacturer supports the agricultural machinery
b. Medium-size manufacturer manufacturing industry in the Philippines.
c. Large-size manufacturer a. AMMDA – Agricultural Machinery
d. None of the above Manufacturers and Distributors Association
of the Philippines
27. Manufacturer capable to operate and b. MIAP - Metal Industries Association of
produce agricultural machine by batch the Philippines
and does mass production of small c. LAMMA- Laguna Agro-Industrial
machines whenever needed. Machinery Manufacturer Association, Inc.
a.Small-size manufacturer d. All of the above
b.Medium-size manufacturer e. Two of the above
c.Large-size manufacturer
d.None of the above
32. Process of heating and cooling 37. Hardened piece of high-carbon steel
metals in their solid state so as to change to prevent cracks or shatters requires an
their properties. additional heat treatment process called
a. Heat treatment ____, which is done by allowing the
b. Hardening material to slowly cool to ambient for a
c. Tempering longer period of time, before it can be
d. None of the above used.
a. quenching
33. Which of the following properties of b. tempering
metal that is changed by heat treatment? c. hardening
a. Hardness d. None of the above
b. Toughness
c. Machinability 38. Which of the following statements is
d. Elasticity true?
e. All of the above a. The amount of carbon content largely
f. Two of the above determine the maximum hardness the
heat treatment will produce.
34. The resulting grain particles when b. The carbon content on steel makes
steel is heated through its upper- steel hardening possible.
transformation temperature and c. Pure iron cannot be hardened by heat
becomes a very hard steel when treatment.
suddenly cooled. d. Plain carbon steel is composed
a. Very fine principally of iron and carbon.
b. Coarse e. All of the above
c. Very course f. Two of the above
d. None of the above
39. Which of the following welding
35. Process of heating metal slowly to equipment is most suitable in welding
proper hardening temperature and then copper and aluminum?
cooling rapidly by quenching in water, a. Arc welding machine
brine or oil. b. MIG welding machine
a. Hardening c. TIG welding machine
b. Tempering d. All of the above
c. Quenching
d. None of the above 40. Welding process which uses liquefied
petroleum gas and oxygen to melt metal
36. High-carbon steel when hardened and join them by means of fusion.
becomes ____ due to internal stresses a. Oxyacetylene welding
resulting from rapid cooling. b. TIG welding
a. soft c. Plasma welding
b. hard d. None of the above
c. very brittle
d. None of the above
41. Which of the following welding 47. Material used as coating in metals to
equipment uses carbon dioxide gas as be joined by soldering in order to prevent
cooling element? oxidation.
a. Arc welding machine a. Acid
b. MIG welding machine b. Alkaline
c. TIG welding machine c. Flux
d. All of the above d. None of the above

42. Which of the following welding 48. Method of welding that uses non-
equipment uses argon gas as cooling ferrous rod heated with oxyacetylene
element? torch at a temperature relatively above
a. Arc welding machine 427°C.
b. MIG welding machine a. Soldering
c. TIG welding machine b. Brazing
d. All of the above c. Resistance welding
d. None of the above
43. Which of the following welding
equipment does not use any gas as 49. Flux used during brazing to remove
cooling element? oxidation on metals being joined.
a. Arc welding machine a. Borax powder
b. MIG welding machine b. Sodium borate
c. TIG welding machine c. All of the above
d. All of the above d. None of the above

44. Welding method commonly used for 50. Bronze welding rod used in brazing
non-ferrous metal at relatively low basically consists of ____ copper and zinc.
temperature of around 427°C. a. 60/40
a. Soldering b. 50/50
b. Brazing c. 40/60
c. TIG welding d. None of the above
d. None of the above
51. Shorter but larger diameter storage
45. Solder used in soldering process tank of oxyacetylene welding is for ____.
consists of ____ lead and tin. a. oxygen
a. 50/50 b. acetylene
b. 40/60 c. carbon dioxide
c. 30/70 d. None of the above _
d. None of the above
52. Acetylene cylinder at full pressure
46. In oxyacetylene welding, the taller contains ___.
cylinder is where the ____ is stored. a. 150 psi
a. oxygen b. 250 psi
b. acetylene c. 350 psi
c. carbon dioxide d. None of the above
d. None of the above
53. Oxygen cylinder at full pressure 58. If there is a proper mixture of
contains ____. acetylene and oxygen, it will produce ___.
a. 1100 psi a. carbonizing flame
b. 2200 psi b. oxidation flame
c. 3300 psi c. neutral flame
d. None of the above d. None of the above

54. Component of a welding machine 59. Welding process that welds two pieces
used to mix the gas and the oxygen in an of metal sheets together using the flow of
oxyacetylene welding equipment. electric current of around 120,000 Amp.
a. Hose a. Arc welding
b. Torch b. Plasma welding
c. Regulator c. Resistance welding
d. None of the above d. None of the above

55. Which of the following is not part of 60. Other term used for resistance or spot
an oxyacetylene welding equipment? welding.
a. Oxygen and acetylene tanks a. Full welding
b. Regulator b. Tack welding
c. Hose c. Intermittent welding
d. Blow pipe d. None of the above
e. Lighter
f. Tongs 61. Welding too long in resistance welding
g. None of the above will create ____.
h. All of the above a. good weld
b. hole in metals being weld
56. In oxyacetylene welding, if the c. weak weld in metals
mixture of oxygen is too much with the d. None of the above
acetylene, it will produce ____.
a. carbonizing flame 62. Which of the following uses a flux-
b. oxidation flame coated welding rod?
c. neutral flame a. MIG welding equipment
d. None of the above b. Oxyacetylene welding equipment
c. Arc welding equipment
57. If acetylene is too much with the d. None of the above
oxygen, it will produce ____.
a. carbonizing flame 63. E6013 electrode is used for ____.
b. oxidation flame a. steel alloy
c. neutral flame b. mild steel
d. None of the above c. tool steel
d. None of the above
64. E7025 is used for ____. 70. Machine tools commonly used in
a. steel alloy making slots, keyways, etc.
b. mild steel a. Drill press
c. tool steel b. Lathe
d. None of the above c. Shaper
d. None of the above
65. Recommended current for 1/8 in.
electrode. 71. Process of forming metal by pushing it
a. 60 amp on a metal die.
b. 100 amp a. Extrusion
c. 250 amp b. Forging
d. None of the above c. Rolling
d. None of the above
66. Recommended current for 1/16 in.
electrode. 72. shop tool used for measuring the
a. 60 amp outside and inside diameter of a metal.
b. 100 amp a. Square
c. 250 amp b. Caliper
d. None of the above c. Trammel
d. None of the above
67. Recommended current for ¼ in.
electrode. 73. Tool used to scribe large circle or arc in
a. 60 amp a piece of metal that works similarly with
b. 100 amp divider.
c. 250 amp a. Square
d. None of the above b. Scriber
c. Trammel
68. Which of the following is not a d. None of the above
machine tool?
a. Lathe 74. Tool used to drive a cross recess screw.
b. Shaper and planers a. Ordinary flat-crew driver
c. Drill Press b. Phillip screw driver
d. Mill machines c. Allen wrench-screw driver
e. Grinder d. All of the above
f. None of the above
75. Tool used to drive slotted machine
69. Metal milling process of making the screws.
handle of a tool rough to provide a a. Ordinary flat-screw driver
better grip. b. Phillip screw driver
a. Boring c. Allen wrench-screw driver
b. Cutting d. All of the above
c. Knurling
d. None of the above
76. Thermal cycle involving heating to, 80. Thermal process used to increase the
and holding at a suitable temperature hardness and strength of steel by
and then cooling at a suitable rate for austenitizing then followed by cooling at a
such purposes as reducing hardness, rate sufficient to achieve partial or
improving machinability, facilitating cold complete transformation of metal to
working, producing a desired martensite.
microstructure, or obtaining desired a. Tempering
mechanical or other properties. b. Quenching
a. Annealing c. Annealing
b. Stress relieving d. None of the above
c. Case hardening
d. Quenching and tempering 81. One of the heat treatment processes of
e. None of the above steel and other ferrous alloys where these
materials are heated above their critical
77. Thermal cycle involving heating to a temperatures long enough for
suitable temperature usually 1000 to transformations to take place.
1200 F, holding long enough to reduce a. Austenitizing
residual stress from either cold b. Tempering
deformation or thermal treatment and c. Annealing
the cooling slowly enough to minimize d. None of the above
the development of new residual
stresses. 82. One or more processes of hardening
a. Annealing steel in which the outer portion, or case, is
b. Stress relieving made substantially harder then the inner
c. Case hardening portion, or core.
d. Quenching and Tempering a. Annealing
e. None of the above b. Stress relieving
c. Case hardening
78. Austenitizing temperature for d. Quenching and Tempering
different grades of carbon, alloys and tool e. None of the above
steels.
a. 100° to below 400°C 82. Material formed in carbon steels by the
b. 400° to 800°C rapid cooling (quenching) of the austenite
c. Above 800° to 1200°C form of iron at such a high rate that carbon
d. None of the above atoms do not have time to diffuse out of
the crystal structure in large enough
79. Process of reheating metal to a quantities to form cementite (Fe3 C).
temperature below the transformation a. Austenitize
range and then cooling at any desired b. Martensite
rate to improve the ductility and c. All of the above
toughness of the material. d. None of the above
a. Quenching
b. Tempering
c. Annealing
d. None of the above
IV. DESIGNING AND DEVELOPING AGRICULTURAL MACHINES

n Agriculture is the primary industry of


the Philippines.
n Agricultural development can be speed
up through mechanization.
n Increase in crop production and
reduction in postharvest losses can be
achieved by the use of appropriate
agricultural machines.
n At present, there is a limited supply of
locally available designs of agricultural
machines in the Philippines.
n Experiences have shown that designing
agricultural machines suitable for
farmers is quite a difficult job because
one should come up with a machine
that is low cost but effective and
efficient.
n In order for the designer to meet this
challenge, he or she needs to have
enough skills and ability to do the job.

Approaches in Designing Agricultural


Machines

n The designers need to have a full


understanding of the variable of the
field conditions and other factors
which influence machinery
performance including the
environmental and human-related
factors. Note that the machine should
be operationally reliable and
economically acceptable to farmers.
n The designers need to apply
engineering principles so that
functional requirements of a certain
machine are met. This includes the
laws of mechanics, strength of
materials, fluid flows, heat transfer, etc.
n The designers should have the
knowledge of the analytical design
process which is essential in the design
of machine components, main frames,
mounting arrangement, balancing, etc.
n The designers must integrate analytical
design and results of experimental
investigation to develop machines that
are simple to fabricate, affordable, light
in weights, easy to maintain, and
require readily available and low cost
materials.
n The designer should also have the
knowledge regarding the level of
manufacturing skills, processes,
viability, and cost of materials in order
to have a simple machines that could
be locally manufactured.
n The design of agricultural machines
should involved a multi-disciplinary
approach that involves product design,
development, testing and evaluation,
and modification prior to
commercialization.

Important Notes:
n Unless the designer tries to meet the
needs of actual users, no amount of
design work will be utilized.
n The success of a good design is
indicated by the extent by which the
machine is adopted by farmers.
Definition of Terms

Designers – Those who employ agents of


technological skills and facility and make
use of the creative energies to bring about
some sort of advantage that is useful to
man.
Designing – Highly innovative and cross-
disciplinary process that uses the most
acceptable, aesthetic, efficiency, and
economical means to satisfy people’s
need.

Philosophy of Design

nIncludes the process of organizing the


methodology needed to proceed from
concept to final product, determining
which technological resources to use, the
economics of production and operation,
and the human and social factors that will
be needed.

Classifications of Agricultural Machine


Design

•Original Design – does not exist before


and it is only done by few dedicated
designers who have personal qualities of
a sufficiently high order. This also
requires a lot of research, experimental
activities, and creative brain.
• Adaptive Design – more concerned of
the adoption of existing designs. This
does not demand special knowledge or
skills. Problems can be solved with
ordinary training.
• Development Design - Begins with an
existing design but the final result may
quietly differ from the initial product.
General Design Development Phase 5 – Fabrication of Proto-Type
Phases Unit

Phase 1 – Conceptual Design Proto-type unit of the machine will


Preparation be first fabricated to determine its
Sketches are prepared showing the performance once tested.
machine to be designed and
explaining how it will work. Phase 6 – Conduct Functional and
Performance Testing of the Machine
Phase 2 – Literature Search
Finding of there are similar existing Testing whether the machine parts
machine being considered. Results work and to determine its
of the previous studies can help performance whether the expected
reduce the required parameters in result is achieved or not.
developing the machine.
Phase 7 – Modify and Improve Parts
Phase 3 – Preparation of the Design of the Machine
Calculation
Design calculations for the different Machine parts that need
parameters needed in the design modification and improvement after
should be done in order to arrive testing must be done until the
into the nearest possible dimension designer is fully satisfied.
and performance as expected.
Phase 8 – Conduct Pilot Test
Phase 4 – Preparation of Materials
and Tools/Equipment for Actual field testing and evaluation of
Fabrication the machine to the target user must
be done to further check if the
Materials to be used as per design machine can fully provide the
must be prepared prior to solution to the target beneficiaries.
fabrication. Fabrication tools and
equipment must be available so Phase 12 – Build Final Model for
that the machine can be well Commercialization
fabricated as per design.
Final model is built to check further
its acceptability to users before to
commercialization.
REFERENCES

Krutz, G. Thompson, L, and P. Claar.


1984. Design of Agricultural
machines. John Wiley and sons, Inc.
New York, USA. 473pp
RNAM. Agricultural Machinery
Design and Data Handbook. (Seeders
and Planters). RNAM/ESCAP. United
Nation Bldg. Rajadamnerm Avenue,
Bangkok, Thailand. 1991. 137pp.
PAES. Philippine Agricultural
Engineering Standards.
ASAE. 1997 ASAE Standards.
Standard Engineering Practice Data.
ASAE USA.
QUESTIONS and PROBLEMS

1. Highly innovative and cross-disciplinary 5. Design classification that does not exist
process that uses the most acceptable, before and it is only done by few
aesthetic, efficiency, and economical dedicated designers who have personal
means to satisfy people’s need. qualities of a sufficiently high order.
a. Designing a. Adaptive design
b. Design b. Development design
c. Designer c. Original design
d. None of the above d. None of the above

2. Process of organizing the methodology 6. Design classification that is more


needed to proceed from concept to final concerned of the adoption of existing
product, determining which technological designs and does not demand special
resources to use, the economics of knowledge or skills.
production and operation, and the human a. Adaptive design
and social factors that will be needed. b. Development design
a. Law of design c. Rational design
b. Principle of design d. None of the above
c. Philosophy of design
d. None of the above 7. Design classification that begins with
an existing design but the final result may
3. In this design, problems can be solved quite differ from the initial product.
with ordinary training. a. Original design
a. Industrial design b. Development design
b. Empirical design c. Empirical design
c. Adaptive design d. None of the above
d. None of the above
8. Design classification that requires lots
4. Design classification that does not exist of research, experimental activities, and
before and it is only done by few creative brain.
dedicated designers who have personal a. Original design
qualities of a sufficiently high order. b. Development design
a. Adaptive design c. Adaptive design
b. Development design d. None of the above
c. Original design
d. None of the above
9. Engineering principles that need to be
applied in designing agricultural
machines include ____.
a. laws of mechanics
b. strength of materials
c. fluid flow, heat transfer, etc.
d. All of the above

10. Design of agricultural machines


should involve multi-disciplinary
approach to include ____.
a. product design and development
b. testing and evaluation
c. modification prior to pilot testing and
commercialization
d. All of the above

11. In designing agricultural machines,


the designer needs to ____.
a. have a thorough understanding of the
variables related to field conditions and
other factors that influence the
performance of the machinery
b. apply engineering principles in order
to meet functional requirements of a
certain machine
c. have thorough knowledge on the
analytical design process and must
integrate analytical design and results of
experimental investigation
d. All of the above

12. Those who employ agents of


technological skills and facility and make
use of their creative energies to bring
about some sort of advantages that are
useful to man.
a. Designs
b. Designers
c. Designing
d. None of the above
V. FUELS AND LUBRICANTS

Fuels used in agricultural engines are Gasoline - product of distillation of crude


made from crude oils. oil.

Crude oils are composed of hydrogen • Classification


and carbon and are called hydrocarbons. • Natural gas gasoline - manufactured
Each hydrocarbon has definite specific from the gas that issues from oil well
gravity and boiling point. or is obtained in the distillation of
crude oil.
• Straight-run gasoline or raw gasoline
Liquefied Petroleum Gas (LPG) - consists - produced by the distillation of crude
of propane and butane. LPG is used as oil.
engine fuel but the fuel supply system for • Cracked gasoline - manufactured from
LPG engines differs from that of gasoline
heavier distillation fraction,
engines. particularly gas oil.
• Blended gasoline - consists of all
types of natural raw or cracked
Liquefied Natural Gas (Natural gas) - gasoline and are mixed together in the
used directly as it comes from the oil well
refining process.
without being subjected to any complex
refining or purifying process. It consists
largely of methane and other
hydrocarbon gases.

Properties of LPG
Substance Density (g/cm3 ) Boiling Temp. Octane Member
(°C) (Research)
Methane - -125 110
Ethane 0.374 -53 104
Propane 0.508 -8 100
Butane 0.584 0 92
Pentane 0.631 36 61
Properties of Gasoline and Engine Kerosene - heavier product of the
Performance distillation of crude oil than gasoline.
• Density = 0.78 - 0.84 g/cm3
• Volatility • Octane Number = 30 – 40
• Antiknock property (octane number) - • Spark ignition engine cannot start by
index to resist detonation during only a kerosene fuel because the initial
combustion. High octane number is point of kerosene is higher than
necessary for high compression ratio gasoline.
and high performance engines. With • Gasoline is used as a start up fuel for
gasoline engine, raising the octane kerosene engine.
number improves engine performance
by allowing the compression ratio to
increase, thereby increasing the power Gas Oil or Diesel Fuel
and efficiency of the gasoline.
Tetraethylead compound is an additive • Diesel includes the heaviest oils used in
used to raise octane number and is the internal combustion engine.
most popular gasoline knock • Density = 0.82 - 0.89 g/cc
supressor. The amount added to • Ignition characteristics (cetane
gasoline is less than 0.3 ml/li. number) takes place in an engine in
• Corrosion - Sulfur compounds are three stages:
corrosive to metals. (a) ignition lag;
• Existent gum - Fuels have a tendency (b) burning injection; and
to form viscous liquids or solids called (c) burning after injection.
gum. The presence of large quantities • Excessive ignition lag results in the
of gum usually causes sticking of intake accumulation of unburned fuel which
valve and heavy deposits in the intake when subjected to high pressure and
manifold. temperature may detonate giving rise
• Gravity - Gravity of fuel is usually to diesel knock.
expressed in API (American Petroleum • In diesel engine, the desirable value of
Institute) degrees. cetane number is necessary to obtain a
good-quality ignition of fuel. High
API = (141.5 /specific gravity at 15°C) cetane fuel permits an engine to be
- 131.5 started at lower temperature.
(Specific gravity of gasoline is between • Relative high viscosity is important for
0.69 - 0.77.) diesel fuel because it must furnish the
lubrication for the injection pump and
the injector.
Fuel for Two-Cycle Gasoline Engine • The lowest temperature at which the
fuel ceases to flow is known as pour
The fuel for two-cycle gasoline engine is a point.
mixture of gasoline and lubricating oil
(ratio approximately 15-20 gasoline: 1
lubricating oil).
Lubricants Viscosity Index

• The primary purpose of lubrication is to • Viscosity of oil varies inversely with


reduce friction and wear between metal temperature. It is express as the
surfaces. Other functions are for variation in viscosity with a change in
cooling, sealing, and cleaning. temperature.
• An oil with high viscosity index has less
change in viscosity with a change in
temperature than an oil with low
viscosity index.
• Thus, high viscosity index is desirable.

Flash and Fire Point

• When oil is heated, vapor is given off


fast enough to burn if contacted with a
spark or flame. The temperature of oil
at which the vapor momentarily is called
Engine Oil (Motor Oil) the flash point and that which a flame
constitute for approximately 5 seconds is
• The most important physical property called the fire point.
of lubricating oil is the viscosity. It is • In general, oil having high flash and fire
internal resistance of fluid as one layer points has high boiling points. Flash
is moved in relation to another layer. points of engine oils range from 120° to
• Lubricating oil is classified in to (1) for 350°C.
gasoline engine, and (2) for diesel
engine.
• Society of Automotive Engineers (SAE) Carbon Residue
classified lubricating oil in terms of
viscosity only. • Lubricating oil often has a tendency to
• Based on quality form carbon deposit in the engine.

For Gasoline For Diesel


Engine Engine
Principal Additives and Their Action 7. Pour Point Depressants - Certain
chemicals that improve their pour point
characteristics, even though the
1. Antioxidants - Organic compounds
lubricant has not been fully dewaxed in
that increase the resistance of an oil to
oxidation and thereby reduce the the refining process.
formation of certain acidic products
8. Viscosity Index Improvers - Under
that may cause corrosion of certain
certain conditions it is found desirable
types of bearing.
to use an oil with viscosity that does
2. Anticorrosives - Certain compounds not increase excessively at low
temperatures. Chemicals have been
containing sulfur, phosphorous, or
found that which aid in retarding the
nitrogen that may be added to an oil
tendency to thicken as the temperature
for the purpose of forming a protective
drops.
film on the bearing surfaces thus
preventing corrosion from any acid
present in the oil.

3. Detergents - Certain compounds that


prevent the building up of carbons and
gummy or carbonaceous deposits on
the piston and under the rings or on
other engine parts.

4. Dispersants - Certain chemicals that


cause any finely divided insoluble
particles of carbon to remain in
suspension in the oil rather than to
separate out in the form of sludge
deposits.

5. Extreme Pressure Agents - Under


certain conditions of high pressure and
temperature, an ordinary lubricant
even with high viscosity will not
provide sufficient oil film between the
metal surfaces to control wear.

6. Foam Inhibitors - Some types of


engine oils have a tendency to absorb
air when agitated vigorously, thus
forming a foam. Certain chemicals
accelerate the rate of breakdown of
the foam but may not actually prevent
foaming.
The API Service Classification for Diesel Engine
Classification Definition
CA Light-duty diesel engine service
CB Moderate-duty diesel engine service. Service typical of diesel engines operated in mild
to moderate duty with lower quality fuels which necessitate more protection from wear
and deposits
CC Moderate-duty diesel and gasoline engine service.
Service typical lightly supercharged diesel engines operated in moderate to severe duty

CD Severe-duty diesel engine service


Service typical of supercharged diesel engines in high-speed, high-output duty requiring
highly effective control of wear and deposit

Gear Oil

• Gear oil is used for gear transmission


lubrication.
• The API Service classification for gear
oils are as follows:

API Services Classification for Gear Oils


GL-1 Low load and speed, moderate condition

GL-2 Severe condition


GL-3 Heavy condition, transmission gear

GL-4 High torque, hypoid gear


GL-5 High speed, severe condition, automotive
transmission

SAE Viscosity Numbers of Gear Oil

SAE Viscosity Kinematic Viscosity (100°C)


Number mm2 /s
Min Max
70W 4.1 -
75W 4.1 -
80W 7.0 -
85W 11.0 -
90 13.5 2.0
140 24.0 41.0
250 41.0 -
Hydraulic Oil Greases

• Fluids are used in the hydraulic lifting • A semifluid or semisolid lubricants and
devices and hydrostatic transmission of are used primarily for moving parts when
tractors and other agricultural vehicles pressures are high and for parts that are
are the petroleum base oils. In general, concealed or inaccessible such as wheel
lubricating oils, particularly turbine oils bearings, spring shackles, axle bearings,
and gear oils usually having viscosity universal joints, and water pumps.
range varying from SAE 5W to 20W are • A mixture of a metallic soap and mineral
used. oil. Certain chemicals maybe added to
provide stabilization, oxidation,
resistance, rust prevention, and other
Properties Commonly Used Hydraulic
Oils desirable characteristics.

Density (15°C) 0.865 – 0.885 g/cm3


Bulk modulus 1300 – 17 Mpa
Flash point 200 – 260 °C
Kinematic viscosity 32 – 36 mm2 /s
(38°C)
Kinematic viscosity 5 – 6 mm2 /s
(100°C) Classification of Grease
Viscosity index 100 – 110
Class Common Characteristics/Uses
Specific heat 1.84 – 1.93 kJ/kg-K Name
Calcium Cup General purpose greases at
grease temperature below 80°C
and are water resistant at
Factors that Cause Deterioration of Oils low temperature
Sodium Fiber Fibrous and well used to
1. Airborne matter - Some dusts reach grease high speed and
temperatures. Non water
the oil in the oil reservoir from air resistant and are used for
cleaner and crankcase breather. wheel bearings and
2. Products of combustion - Include universal joint
water, acids, and carbon. Aluminu Motor or Used for high speed gear
3. Product corrosion - Corrosive attacks m chassis lubrication
on metal parts by the acids of grease
combustion may cause rust deposits Lithium Multi-
enter the oil. purpose
4. Unburned fuel - Due to incomplete grease
combustion.
5. Metallic particles - Result of water
REFERENCE

Yamazaki, M. Fuels and Lubricants.


Handout Sheet. JICA Agricultural
Machinery Management Course. Japan.
25pp
QUESTIONS AND PROBLEMS

1. Substance that carries energy and 12. High-cetane-fueled engine uses ____.
produces power when ignited. a. diesel as fuel and is difficult to start
a. Heat at lower temperature
b. Fuel b. diesel as fuel and is easy to start at
c. Air lower temperature
d. None of the above c. gasoline fuel and is easy to start at
lower temperature
2. Elements necessary for combustion. d. None of the above
a. Fire
b. Air 13. Specific gravity of gasoline is at the
c. Fuel range of ____.
d. All of the above a. 0.67 to 0.76
e. Any two of the above b. 0.79 to 0.85
c. 0.85 to 0.91
6. Gas produced during perfect d. All of the above
combustion of fuel.
a. Carbon monoxide 14. Auto-ignition temperature of diesel is
b. Carbon dioxide ____ that of gasoline.
c. Hydrogen a. the same with
d. None of the above b. less than
c. more than
10. Caloric value of fuel is ____. d. None of the above
a. the amount of heat contained by the
fuel per unit weight 15. Auto-ignition temperature of
b. the weight of fuel to be heated gasoline.
c. the heat needed to burn a certain a. 256°C
weight of fuel b. 280°C
d. None of the above c. 301°C
d. None of the above
11. Amount of heat required to raise the
temperature of one pound water by one 16. Auto-ignition temperature of diesel.
degree Fahrenheit. a. 256°C
a. kcal b. 280°C
b. BTU c. 301°C
c. Joule d. None of the above
d. None of the above
17. Lowest temperature where the fuel 22. When low viscosity oil is used in the
evaporates to form combustible transmission box of a puddling type tiller,
concentration of gas. there will be ____ in the system.
a. Flash point a. more heat produced from liquid
b. Spontaneous combustion friction produce
temperature b. more mechanical friction
c. Cetane Rating c. no friction
d. None of the above d. None of the above

18. High flash point indicates that the 23. If the shaft rpm in a transmission box
fuel will ___ burn. is increased, the coefficient of friction of
a. not fluid will ____ .
b. easily a. remains the same
c. hardly b. decrease
d. None of the above c. increase
d. None of the above
19. Flash point temperature of gasoline.
a. - 10°F 24. The higher the viscosity of a fluid, the
b. - 45°F ____ is the energy required to slide the
c. + 10°F surfaces at a given speed and gap.
d. None of the above a. lesser
b. greater
20. Flash point temperature of diesel. c. None of the above
a. 70° to 95°F
b. 100° to 130°F 25. Lubricants viscosity basically ____
c. 135° to 160°F with the increase in temperature.
d. None of the above a. increases
b. decreases
21. When high viscosity oil is used in the c. do not change
transmission box of a puddling-type tiller d. None of the above
is high, there will be ____ in the system.
a. more heat from liquid friction 26. Measure of oil viscosity-temperature
produce behavior.
b. more mechanical friction will occur a. Oil Index
c. no friction b. Viscosity Index
d. None of the above c. V/T Index
d. None of the above
27. Viscosity index of SAE 10W-30 oil.
a. 100 V.I.
b. 120 V.I.
c. 140 V.I.
d. None of the above

28. Viscosity Index of SAE 10W oil.


a. 100 V.I.
b. 120 V.I.
c. 140 V.I.
d. None of the above

29. Multi-grade oils have ____ VI than


single grade oil, i.e., viscosity changes
less with temperature.
a. lower
b. middle
c. higher
d. None of the above

30. Basically, oil viscosity ____ with


pressure.
a. increases dramatically
b. decreases dramatically
c. do not change
d. None of the above
VI. Primary Prime Movers for Agricultural
Machines

Major Power Sources

Internal Combustion Engine


• Single-cylinder and Multi-
cylinder engines
• Spark-ignition and
Compression-Ignition engines
Electric Motor
• Single-phase and three-
phase motors
• AC and Dc motors

Internal Combustion Engine

A heat engine that burns fuel


and air inside a combustion
chamber located within the
engine proper.
A machine use to power
tractors, combine harvesters,
and other self-propelled
equipment

Classifications of ICE

(1) According to Ignition

Spark-Ignition Engine
•uses a spark to ignite fuel (2) According to Number of
Stroke
•Gasoline is use as fuel
Two-Stroke Cycle Engine
Compression-Ignition Engine •Produces power in one
•Uses high compression to raise revolution of the crankshaft
the temperature of the air in a
Four Stroke Cycle Engine
chamber and ignites the injected •Produces power in two
fuel without spark revolution of the crankshaft
•Diesel is use as fuel
(3) According to Number
of Cylinders
Single-Cylinder
• One piston
Multiple-Cylinder
• More than one piston
(3, 4, 6, 8, 12)

(4) According to the


Positon of Drive Shaft

n Horizontal Drive Shaft –


Drive shaft is in horizontal
position while the piston is
upward position
n Vertical shaft – The
drive shaft is vertical
position while the piston is
in sideward position

Basic Components of the Crankshaft – It receives power


Main Engine System from the piston and transmit
it to the other parts to do
Cylinder Head – It houses the useful work
valves, spark plug or fuel
injector, and serves as cover Piston Rings (compression
of the cylinder block and oil rings) – it retain
compression, reduce cylinder
Cylinder Block – It encloses wall contact thereby reducing
the piston and confines the friction losses and wear
expanding gases
Piston pin – it joins the
Valves – It allows the flow of connecting rod and the piston
fuel, air, or exhaust gases
Flywheel – it is the energy
Piston - It is a hollow storage device of the engine
cylinder with closed top to keep it at uniform speed
upon which the expanding when the crankshaft is not
gases act and transmit force receiving power to the piston
to the crankshaft
Crankcase – it encloses and
supports the crankshaft and
serves as reservoir for the
lubricating oil
Engine Subsystems
n Engine Displacement (PD) –
n Ignition System – responsible volume displaced by a piston in
for supplying heat for one stroke
combustion of fuel
n Clearance Volume (CV) –
n Fuel System – responsible for volume in the combustion
supplying fuel in the cylinder when the piston is at
combustion chamber the TDC position
n Starting System – responsible n Compression Ratio (CR) – ratio
to starting-on the engine of the total volume to the
clearance volume
n Cooling System – responsible
to keep the temperature of n Mean Effective Pressure (MEP)
the engine lower – amount of pressure
generated during the power
n Exhaust System – responsible stroke of the engine
to discharge combustible
gases from the combustion n Revolution – one complete
chamber rotation of the crankshaft
n Transmission System – n Cycle – series of event
responsible for transferring occurring one after the other in
the power generated from the a definite order and repeats the
combustion chamber to the event after the last one has
drive shaft occurred

n Firing Order – The sequence of


piston power in multiple
Terminologies cylinder in delivering the power
n Bore – size of the opening of the stroke
cylinder almost equal to the the n Indicated Horsepower – the
diameter of the piston plus the power generated at the
rings
combustion chamber
n Stroke – distance traveled by a n Brake Horsepower – power
piston from the TDC to the BDC available at the drive shaft or
n Top Dead Center (TDC) – the the crankshaft of an engine
uppermost position of the piston n Friction Horsepower – power
during the compression and loss due to friction of piston and
exhaust stroke other component parts of the
n Bottom Dead Center (BDC) – the engine
lowest most position of the piston n Rated Horsepower – power as
during the intake and power stroke specified by the manufacturers
n Mechanical Efficiency – ratio of the
brake horsepower and the
indicated horsepower
n Thermal Efficiency – ratio of the
brake horsepower to the power
available at the fuel
n Firing Order – sequence of
delivering of power stroke by the
piston in multi-cylinder engine
n Specific fuel consumption –
amount of fuel consumed by brake
horsepower of the engine

Cycles of Events
n Intake Stroke – fuel and air or air Running Mates
alone is suck in the combustion
chamber as the piston moves in Pistons that go up and down
downward direction together in the same position
n Compression Stroke – fuel and air but of different function. One
or air is compressed by the piston of them is performing
as it moves in upward direction compression or power stroke
n Power Stroke – fuel and air is and the other is on the intake or
ignited to produce power at the exhaust stroke.
downward movement of the
piston and transmitted it into the
crankshaft of the engine
n Exhaust Stroke – burn fuel and
gases is discharged from the
combustion chamber

Vertical-Engine Running Mates

V-Engine Running Mates


Engine Specifications Engine Performance Curve

It gives the physical features It gives the characteristics


and performance of an engine in terms of
characteristics of an engine the power, fuel
in terms of power, fuel consumption, and torque
for a given operating
consumption, and efficiency
speed.

Brand ACME Ducati Honda Robin

Model AON IS 7 GX 160 EY 15


48W D

Type 4- 4- 4-cycle Air


stroke cycle air cooled
vertical vertica cooled, ,4
air l air gasolin cycle
cooled cooled e gasoli
diesel horizont ne,
Bore x 85 mm 75 mm 68 mm
al shaft 63 mm
horizo
Stroke x 85 x 78 x 45 xntal,
46
mm mm mm mm
PTO
Displac 482 cc 345 cc 163 cc shaft
143 cc
ement

Output 11.3 7.5 hp 5.3 3.5


Power hp/3000 / 3200 hp/3600 hp/400
rpm rpm rpm 0 rpm
Maxim 2.52 1.1 kg- 0.68
um kg- m/2500 kg-
torque m/2058 rpm m/280
rpm 0 rpm
Ignition Direct Transist
system injectio or
n
Maxim 2.66 193gts 240
um fuel li/hr /hp-hr g/hp-hr
consu
mption
Dry 55 kg 48 kg 14 kg 13.2
Weight kg
USEFUL FORMULA

Piston Displacement Piston Displacement Rate


π D2
PD = -------- L n PDR = 2 π PD N
4
where: where:
PD - piston displacement, cm3 PDR - piston displacement rate, cm3 /min
Dp - piston diameter, cm PD - piston displacement, cm3
L - length of stroke, cm N - crankshaft speed, rpm
n - number of cylinders
Compression Ratio Mechanical Efficiency
PD + CV BHP
CR = ------------- ξm = --------- x 100
CV IHP
where: where:
CR - compression ratio ξm - engine mechanical efficiency, dec
PD - piston displacement, cm3 BHP - brake horsepower, hp
CV - clearance volume, cm3 IHP - indicated horsepower, hp
Indicated Horsepower Brake Horsepower
PL AN n
IHP = ----------------- BHP = IHP ξm
33000 c or
where: = IHP – FHP
IHP - indicated horsepower, hp where:
P - mean effective pressure, psi BHP - brake horsepower, hp
L - length of stroke, ft IHP - indicated horsepower, hp
A - area of bore, in2 ξm - engine mechanical efficiency, dec
N - crankshaft speed, rpm FHP - friction horsepower, hp
n - number of cylinder
c - 2 for four-stroke engine and
1 for two- stroke engine
Rate of Explosion Specific Fuel Consumption
V
N SFC = ---------- S
ER = -------- Ne t
c where:
where: SFC - specific fuel consumption, kg/W-sec
ER - explosion rate, explosion per minute V - fuel consumption, m3
N - crankshaft speed, rpm Ne - brake output
c - 2 for four-stroke engine and t - time, sec
1 for two-stroke engine S - specific gravity of fuel, kg/m3
Thermal Efficiency, Theoretical Thermal Efficiency, Effective
C Wt C Ne
ξtheo = ----------- x 100 ξeff = ------------ x 100
Qt Hu B
where: where:
ξtheo - theoretical thermal efficiency, % ξeff - effective thermal efficiency, %
C - conversion constant C - conversion constant
W t - theoretical work, kg-m Ne - effective output, watt
Qt - supplied heat quantity, kcal/hr Hu - calorific value of fuel, kcal/kg
B - indicated work, kg/hr
Fuel Consumption Specific Fuel Consumption

Fc = Fu / To SFC = Fc ρf / Ps

where: where:
Fc - fuel consumption, lph SFC - specific fuel consumption, g/kW-hr
Fu - fuel used, li Fc - fuel consumption, lph
To - total operating time, hr ρf - fuel density, kg/li
Ps - shaft power, Kw
Piston Area Number of Times Intake Valve Opens
π D2 N
A p = ------------ TO = --------
4 c
where: where:
A p - piston area, cm2 TO - number of times intake valve opens
D - piston diameter, cm N - crankshaft speed, rpm
c - 2 for four-stroke engine and
1 for two-stroke engine
Stroke-to-Bore Ratio Break Mean Effective Pressure
S (75) 50 BHP
R = --------- BMEP = --------------------
B L AN n
where: where:
R - stroke to bore ratio BMEP - brake mean effective pressure,
S - piston stroke, cm kg/cm2
B - piston diameter, cm BHP - brake horsepower, hp
L - piston stroke, m
A - piston area, cm2
N - number of power stroke per min
n - number of cylinders
ELECTRIC MOTOR

Electric motors are


machines that converts
electrical energy to
mechanical power by
producing rotational
motion and torque.

Motor Job
n The motor must be able to
drive the load at a proper
speed.
Use of Electric Motor n The motor must be able to
n Pumping of Water start the load.
n Driving Grain Milling
Equipment n The motor must be able to
n Driving Size Reduction withstand the abuse of the
Equipment surroundings.
n Driving Material Handling
Equipment
n Driving various food
equipment
Horsepower Rating of Motors

n The greater the horsepower


Advantages
rating of the motor, the greater
n Low initial cost compared
the motor ’s ability to develop
with engine
mechanical power.
n Low operating cost
n Available horsepower rating of
n Long life
motors
n Easy to operate
• 1/6, ¼, 1/3, ½, ¾, 1, 1-1/2, 2, 3,
n Capable of withstanding
5, and 7-1/2 hp.
temporary overloads
n Compact construction
Speed Rating
n Safe
n Little affected by hot or
The most common speed of
cold weather
the motor ranges from 1700 to
n Quiet
1750 rpm
n Automatic and remote
control
Full Load Current and
Recommended Wire and Fuse
Size for Single Phase
Induction Motor @ 220 Volt
Motor Approx Extens Branch Approx.
Hp full ion- Circuit Rating
Load Cable Fuse Overload
Current Wire Size Protection
Size

1/6 1.6 16 15 1.8

1/4 2.3 16 15 2.6

1/2 3.7 14 15 4.3

3/4 5.1 12 15 5.9

Motor Classification 1 6.5 12 20 7.5

n Split-Phase Motor –Motor with 1-1/2 9.2 10 30 10.6


low starting load.
2 12 10 40 13.8
n Capacitor-Start Induction Motor
– Motor capable of starting
heavier load than split phase 3 17 8 50 19.6
motor.
5 28 6 60 33.4
n Repulsion Start Induction Motor
– Motor that develops high
starting torque Motor Nameplate
n Squirrel Cage Induction Motor
The nameplate rating of a capacitor-
start electric motor showed that the
motor rated power is 1.5 kW. If the
rated current and voltages are 13.5
General Rules in Determining the Amp and 220 volts, respectively,
Size of a Motor what is the percentage allowable
power provided by the
n Write manufacturer ’s or manufacturers for the motor.
supplier ’s for the specific
application of the motor.
n If motor will replace a man, ¼
hp motor can do the job.
n To replace an engine, multiply
engine hp by 2/3. That is, 1 hp
x 2/3 is ¾ hp or 2 hp x 2/3,
select 1-1/2 hp.
USEFUL FORMULA

Horsepower Output (Three-Phase) Power in Circuit (Single-Phase)


η I E pf
HP = √3 --------------- P=EI
746
where: where:
HP - power output, hp P - power, w
E - voltage, v E - voltage, v
I - current, amp I - current, amp
η - efficiency, dec
pf - power factor, dec
Power in Circuit (Three Phase) KVA (Single-Phase Circuit)
E I
P = √3 E I KVA = ---------------
1000
where: where:
P - power, w KVA - kilovolt amp
E - voltage, v E - voltage, v
I - current, amp I - current, amp
KVA (Three-Phase Circuit) Horsepower Output (Single-phase)
1.732 E I ξ I E pf
KVA = ---------------- HP = ------------------
1000 746
where: where:
KVA - kilovolt amp HP - power output, hp
E - voltage, v E - voltage, v
I - current, amp I - current, amp
ξ - efficiency, dec
pf - power factor, dec
Slip (Three-Phase Motor) Rotor Speed (Synchronous Motor)

S = [Ns – N] / Ns Ns = 120 [ƒ / P]
where: where:
S - slip, dec Ns - rotor speed, rpm
Ns - motor synchronous speed, rpm ƒ - frequency of stator voltage, hertz
N - actual motor speed, rpm P - number of pole
Motor Size to Replace Human Motor Size to Replace Engine

MHP = NH ¼ MHP = EHP 2/3

where: where:
MHP - motor power, hp MHP - motor power, hp
NH - number of human EHP - engine power, hp
References
n Brown, R. 1956. Farm
Electrification. McGraw-Hill
Book company. New York.
n Miller, R. and R. Miller. 1984.
Small Gasoline Engines.
Theodore Audel and Company.
Boston. 632pp.
n Schulz, E. J. 1977. Diesel
Mechanics. McGraw-Hill Inc.
U.S.A. 420pp.
n Stephenson, G.E. 1984. Small
Gasoline Engines. 4 th Ed.
Delmar Publishers Inc. Canada.
279pp.
QUESTIONS and PROBLEMS

1. An engine that burns fuel inside the 7. The decrease or increase of engine
cylinder. speed or flow of fuel.
a. External combustion engine a. Throttling
b. Internal combustion engine b. Clutching
c. Sterling engine c. Atomization
d. None of the above d. None of the above

2. The part of an engine that slides up 8. A device that is used to actuate the throttle
and down the cylinder and transmits of an engine to increase its speed.
power to the crankshaft. a. Governor
a. Connecting rod b. Accelerator
b. Cylinder block c. Actuator
d. None of the above
c. Piston
d. Spark plug 9. A device that meters the fuel and the
e. None of the above air in spark-ignition engines and mixes
them into a harmonious mixture before
3. The amount of power that the the mixture enters the combustion
manufacturer specified for an engine. chamber of an engine.
a. Indicated horsepower
a. Governor
b. Brake horsepower b. Air injector
c. Rated horsepower c. Carburetor
d. None of the above d. All of the above
4. The constant volume combustion
10. The usable power delivered by the
cycle. engine.
a. Otto cycle a. Indicated horsepower
b. Diesel cycle b. Brake horsepower
c. Dual cycle c. Rated horsepower
d. None of the above
d. None of the above
5. The constant pressure combustion 11. An internal combustion engine that
cycle. produces power in one revolution of the
a. Otto cycle crankshaft.
b. Diesel cycle a. Four-stroke-cycle engine
c. Dual cycle b. Two-stroke-cycle engine
d. None of the above c. Rotary engine
d. None of the above
6. The distance of the movement of
piston from the top dead to the bottom
dead center.
a. Stroke
b. Bore
c. Valve clearance
d. None of the above
12. The ratio of total volume to the 18. During the power stroke, which of the
clearance volume. following valve is open?
a. Compression ratio a. Intake
b. Air-fuel ratio b. Exhaust
c. Bore stroke ratio c. Intake and exhaust
d. None of the above d. None of the above

13. The volume displaced by the piston in 19. The number of spark plugs for a 6-
making an upward or a downward stroke. cylinder diesel engine.
a. Piston displacement volume a. 6
b. Clearance volume b. 12
c. Total volume c. 4
d. None of the above d. None of the above

14. An engine has a piston displacement 20. At the downward stroke of the two-
of 38.4 in³. If the clearance volume of the stroke cycle engine, which of the events
cylinder is 2.5 in³, what is its compression are taking place?
ratio? a. Power, intake, and exhaust
a. 1:12 b. Compression
b. 1:16 c. Power & exhaust
c. 1:8 d. None of the above
d. None of the above
21. An engine that injects fuel into the
15. The ratio of the piston displacement combustion chamber near the end of the
volume plus the clearance volume over combustion stroke and is subsequently
the clearance volume. ignited only by heat of compression.
a. Compression ratio a. Gasoline engine
b. Air-fuel ratio b. Diesel engine
c. Bore-stroke ratio c. Steam engine
d. None of the above d. None of the above

16. Suppose a 4-cylinder engine has 3.5in. 22. The power transmitted to the piston
bore and 4.0in. stroke, what is the engine by the gas in the cylinder.
displacement. a. Brake horsepower
a. 154 in³ b. Indicated horsepower
b. 175 in³ c. Drawbar horsepower
c. 225 in³ d. All of the above
d. None of the above
23. The number of piston explosion in a
17. A device that provides ignition to a single-cylinder, four-stroke cycle engine
gasoline engine. and in 100 revolution of the crankshaft.
a. Fuel injector a. 100
b. Carburetor b. 50
c. Spark plug c. 25
d. None of the above d. None of the above
24. A device that connects and 29. What comes in during the intake
disconnects the power input to and from stroke of a compression-ignition engine?
the power output. a. Fuel and air
a. Clutch b. Fuel
b. Gear c. Air
c. Pulley d. None of the above
d. None of the above
30. A four-stroke, six-cylinder biomass gas
25. The space when the piston is at the producer engine has 8in. cylinder bore
top dead center. and 12in. stroke. If the engine is running
a. Piston displacement volume at 500 rpm speed, what is its indicated
b. Clearance volume horsepower? The mean effective pressure
c. Total volume is 80 psi.
e. None of the above a. 22.5 hp
b. 30.5 hp
26. How many piston strokes are there in c. 40.5 hp
one revolution of the crankshaft? d. None of the above
a. One
b. Two 31. Engine brake thermal efficiency.
c. Four a. Brake hp over fuel hp
d. None of the above b. Rated hp over indicated hp
c. Friction hp over flywheel hp
27. What is the indicated horsepower of a d. None of the above
four-stroke, six-cylinder engine with 4in.
bore and 4in. stroke. The engine is 32. A device that creates ignition of fuel in
running at 1500 rpm and the mean a diesel engine.
effective pressure is 80 psi. a. Spark plug
a. 35 hp b. Piston
b. 45 hp c. Carburetor
c. 52 hp d. None of the above
d. None of the above
33. The part of an engine that converts
28. The sequence by which the cylinder reciprocating motion to rotary motion.
delivers the power stroke. a. Crankshaft
a. Ignition b. Piston
b. Firing order c. Camshaft
c. Flash point d. None of the above
d. None of the above
34. Device that converts electrical energy 40. Which of the following are the jobs of
to mechanical energy to produce power. a motor?
a. Battery a. To drive the load at proper speed.
b. Motor b. To be able to start a load.
c. Generator c. To be able to withstand the abuse of
d. None of the above the surroundings.
d. All of the above
35. Term that indicates the maximum e. None of the above
load that can be successfully carried by
the motor if it is to be operated 41. NEMA stands for ____.
continuously and remain within a safe a. National Electric Motors Association
temperature range. b. National Electrical Manufacturers
a. Duty rating Association
b. Phase c. National Electrical Machinery
c. Service factor Association
d. None of the above d. None of the above

36. Motor torque at zero speed or 42. Based on NEMA standards, which of
maximum torque required to start the the following statements is true?
load. a. The higher the motor power, the higher
a. Twisting Torque
is the efficiency.
b. Driving Torque
c. Starting Torque b. The higher the efficiency of the motor,
d. None of the above the higher is the power factor.
c. The higher the motor power, the higher
37. Type of motor with medium starting is the power factor.
torque commonly used in industry. d. All of the above
a. Split-phase motor
b. Capacitance-type motor 43. Motor that is usually connected to the
c. Repulsion-start motor grid.
d. None of the above a. AC Motor
b. DC Motor
38. Type of motor with low starting c. Both AC & DC Motors
torque. d. None of the above
a. Split-phase motor
b. Capacitance-type motor
c. Repulsion-start motor
d. None of the above

39. Type of motor with a very high starting


torque.
a. Split-phase motor
b. Capacitance-type motor
c. Repulsion-start motor
d. None of the above
44. Important information necessary for
the proper application of the motor.
a. Specification
b. Nameplate
c. Operators manual
d. None of the above

45. Term referring to how frequent the


motor is used.
a. Duty rating
b. Phase
c. Service factor
d. None of the above

46. What size of motor would you


recommend if 1hp engine is to be
replaced?
a. ¾hp motor
b. 1hp motor
c. 1.5hp motor
d. None of the above

47. Part of an electric motor that protects


it from dust and water.
a. Fan
b. Enclosure
c. Oil seal
d. None of the above

48. Type of a motor enclosure that


protects the motor from dangerous
locations.
a. Drip-proof enclosure
b. Totally-enclosed enclosure
c. Explosion-proof enclosure
d. None of the above

49. Number of individual voltages applied


to the motor.
a. Ampacity
b. Voltage Drop
c. Phase
d. All of the above
e. None of the above
VII. AGRICUTURAL MACHINERY
MANAGEMENT

Field efficiency - Ratio of effective field Effective field capacity - Actual rate of land
capacity to field capacity, expressed in processed in a given time.
percent.
Efc = ξf (0.1 W V)
ξf = (efc/tfc) x 100 where
where: Efc - effective field capacity, ha/hr
ξf- field efficiency, % ξfn - field efficiency, decimal
efc- effective field capacity, ha/hr W - implement width, m
tfc- theoretical field capacity, ha/hr V - implement velocity, kph

Functional efficiency - Ratio of the actual


effectiveness of a machine to its
theoretical effectiveness, expressed in
percent. Threshing efficiency of a
combine is an example of a functional
efficiency.

ξfn = (Aem/Tem) x 100


where:
ξfn - field efficiency, %
Aem - actual effectiveness of machine,
kg/hr
Tem - theoretical effectiveness of the
machine, kg/hr
Theoretical field capacity - Rate of the Effective operating width - The width over
performance obtained if machine which the machine actually works. It may
performs its function 100% of the time at
be more or less than the measured width
a given operating speed using 100% of its
theoretical width. of the machine.

Tfc = (0.1 W V) Theoretical operating width - The


where measured width of the working portion of
Efc - effective field capacity, ha/hr
a machine. For row crop machines, it is the
W - implement width, m
V - implement velocity, kph average row width times the number of
rows.
Field speed - Average rate of machine
travel in the field during an uninterrupted
period of functional activity. For
example, functional activity would be
interrupted when the implement is raised
out of the soil.

Field time - The time a machine spends in


the field measured from the start of
functional activity to the time the
functional activity for the field is
completed.

Economic life of machine - The useful


service life of a machine before it
becomes unprofitable for its original
purpose due to obsolescence of wear.

Field load factor - The ratio of engine


power used in performing an operation
to engine power available.
Individual operation - Operating one or Custom cost, machine - The amount paid
more similar machines as one unit. for hiring equipment and operator services
to perform a certain task. Custom cost
Parallel operation - Causing two or more normally include a charge for the
similar machines to perform their operation of the basic machine, and may
respective functions simultaneously. or may not include supplemental labor and
equipment for such tasks as handling into
Series operation - Causing two or more storage or transport of a harvested crop,
machines to perform their respective transportation of seed or fertilizer to the
functions in sequence; each machine field, etc. Charges may be determined on
operation, except the first, it is the basis of area, time, transport distance
dependent upon previous operations, or quantity of crop processed.
and stopping one machine would halt all
subsequent machines. Operating costs, machine - Costs which
depend directly on the amount of machine
System machines - An arrangement and use. Examples are labor, fuel, lubrication,
use of two or more machines to and repair and maintenance costs.
achieved a desired output.
Ownership costs, machine - The costs
Timeliness - Ability to perform an activity which do not depend on the amount of
at such a time that crop return is machine use. Examples are depreciation,
optimized considering quantity and interest on investment, taxes, insurance,
quality of product. and storage.

Timeliness coefficient - A factor used to


estimate the reduction in crop return
(quantity and quality) due to lack of
timeliness in performing an activity.
Total cost, machine - The sum of Obsolete - The condition of a machine
ownership and operating costs. when it is either out of production and
parts to repair and update it are not
Actual depreciation - Change in value of a available from normal suppliers, or it can
machine. be replaced by other machine or method
that will produce greater profit.
Estimated depreciation - The change in
value as determined by the difference Price - Market value per unit. Examples
between purchase price and estimated are the price of grain, usually dollars per
future vale, both in constant dollars. unit measure; of labor, dollars per hour; or
machines, dollars per machine.
Straight line, declining balance, sum of
years' digits depreciation - Methods to Rent - A rental agreement is a short-term
spread the change in machine value over contract that permits the use of
the economic life of the machine. This machinery in exchange for a fee.
method may disagree with estimated
depreciation. Gross - The return for sale of a service or
product and the value received for a
Lease - A lease in a contract for the use service or product before expenses are
of a machinery for an agreed period of deducted.
time in return for periodic payments.
Ownership remains with the lessor. The Net - The return for sale of a service or
lessee acquires the right of temporary product and the value received for a
possession and use. service or product, less all expenses
except income taxes.
Obsolescence - The process of becoming
obsolete.
Continuous duty - A service requirement
that demands operation for an
indefinitely long period of time.

Failure - The inability of a machine to


perform its function under specified field
and crop conditions.

Fuel consumption, specific - The fuel


consumed by an engine to deliver a given
amount of energy (kilograms of fuel per
kilowatt-hour).

Maintenance and service - Periodic


activities to prevent premature failure
and to maintain good functional
performance. Examples are refueling,
changing oil and filters, cleaning,
lubricating, and adjusting components.

Major overhaul - Extensive rebuilding


which extends the useful life of a
machine, increases its value or adapts the
machine for a different use.

Repair - Restoring a machine to operative


condition after breakdown, excessive
wear, or accidental damage. Repairs are
less extensive than major overhauls and
normally do not alter the value of the
machine.
Equipment Used and Primary Energy Sources in the Different Stages
of Operation in Rice Farming

Activity Machine Primary Energy Source (s)


Land Preparation Power tiller, plow, harrow, Fossil Fuel and Animals
leveler, etc; puddling-type
floating tiller; small four-wheel
drive tractors with rotary tiller
Seed Preparation/Propagation Human
Planting
Direct Seeding Drum seeder Human
Broadcasting Human
Transplanting Transplanter Human and Fossil fuel
Field Maintenance
Irrigation/Drainage Pumps Fossil Fuel
Fertilization Fertilizer applicator Human
Pest Management Knapsack Sprayer Human and Fossil Fuel
Harvesting
Hand Sickle and scythe Human
Reaper Reaper Harvester Fossil Fuel
Combine Combine Harvester Fossil Fuel
Threshing Rice Thresher Fossil Fuel and Human
Drying Flatbed and re-circulating dryers Solar, Biomass, and Fossil Fuel
Cleaning and bagging Grain cleaners Fossil Fuel, Electricity and Human
Storage Inclined conveyor Human and Electricity
Milling and bagging Rice mill Electricity, Fossil Fuel, and Human
Storage/Marketing of Milled Truck Human and Fossil Fuel
Rice

Power Requirement, Fuel/Electricity Consumption, and Field/Throughput


Capacity of the Commonly Used Rice Machines

Fuel/Electricity Field/Throughput
Machine Power Requirement
Consumption Capacity
Power Tiller
Plowing 7-Hp Diesel Engine 0.87 L h-1 1 ha d-1 (8 hrs)
First harrowing -do- -do- -do-
Second harrowing -do- -do- -do-
Leveling -do- -do- -do-
Irrigation Pump 8-Hp Diesel Engine 0.8 to 1.0 L h-1 10 L s-1 for 24 hours
per ha
Combine Harvester 60-Hp Diesel Engine 15 L ha-1 2.5 ha d -1
Axial-Flow Rice Thresher 16-Hp Gasoline Engine 2 L h-1 1 ha harvest per 6 hrs
Flatbed Dryer 12-Hp Diesel Engine; 90,000 1.2 L h-1 ; 30 kg hr-1 6 tons for 10 hrs
kcal hr-1 Heat Energy Input rice husks
Biomass Furnace
Re-circulating Dryer 10-kW Electric Motor; 55,000 11.6 to 15 kW hr ton- 6 tons for 7-9 hrs
kcal/hr Heat Energy Input 1

Kerosene Burner 6 L h-1 of kerosene


Grain Cleaner 3-Hp, 220-volt Electric Motor 0.39 to 0.58 kW hr 30 to 45 sacks per day
sack-1
Multiple Pass Rice Mill 20-kW, 220-volt Electric motor 20 kW hr ton-1 1 ton paddy per hr
Custom Rate for Labor and for Hiring Agricultural Machines

Machine Cost of Labor, Operation, or Rental


Small 4-wheel Tractor P1000 for the driver per day plus 2 sacks of palay per
day spent.
Power Tiller (1 plowing and 3 P4,500.00 per hectare
harrowing)
Planting
Manual P500 per person plus two snacks and lunch.
Harvesting Combine 1 sack per 7 to 10 sacks of harvested palay
Drying P40 to 60 per bag of palay using mechanical dryer or
P20 per bag for the labor in sundrying.
REFEREENCE

ASAE. ASAE Standards 1997. American


Society of Agricultural Engineers. 2960
Niles Road St. Joseph, MI. USA.
QUESTIONS and PROBLEMS

1. The actual coverage rate by an 8. A booklet provided by manufacturers


implement based on the total field time. that indicates the adjustment, care and
a. Field efficiency maintenance procedure as well as the
b. Effective field capacity safety of the machine.
c. Theoretical field capacity a. Operator ’s manual
d. None of the above b. Technical manual
c. Product catalog
2. The ratio between the mass of fuel d. None of the above
consumed per unit time and the
corresponding power measured. 9. ASABE stands for ___.
a. Specific fuel consumption a. American Society of Automotive and
b. Fuel to mass ratio Biosystems Engineers
c. Fuel consumption rate b. American Society of Agricultural and
d. None of the above Biological Engineers
c. American Society of Agricultural and
Biosystems Engineers
5. A distinguishing symbol affixed to d. All of the above
products by a manufacturer in order to
identify them as his goods. 10. Based on the Philippine Agricultural
a. Trade name Engineers Standard (PAES), warranty for
b. Trade mark parts and services of a heated-air
c. Brand name mechanical grain dryer is within ___ after
d. All of the above the acceptance of the machine by the
buyer.
6. A name given by a manufacturer to a. 12 months
his products to distinguish them from the b. 18 months
ones produced by other manufacturers. c. 24 months
a. Trade mark d. None of the above
b. Manufacturer ’s name
c. Brand name 11. According to PAES, warranty for the
d. None of the above construction and durability of mechanical
rice thresher shall be within ___ after the
7. A booklet provided by manufacturers purchase of the machine.
that gives the step-by-step procedure for a. 6 months
repair or for disassembling parts of the b. 12 months
machine. c. 18 months
a. Operator ’s manual d. None of the above
b. Technical manual
c. Product catalog
d. All of the above
12. A service offered to customers by 15. The useful life of a machine before it
manufacturers in repairing and replacing becomes unprofitable for its original
unintentionally-damaged part of a purpose due to obsolescence or wear.
machine for a specified period. a. Economic life
a. Manufacturing service b. Life span
b. Customer ’s service c. Profitable life
c. After-sales-service d. All of the above
d. All of the above
16. The ability of a machine to perform a
13. After 4 months of continuous use, the timely operation that optimizes return
pillow-block bearing of a brand new rice from crop considering quality and quantity
thresher was heavily worn out. As an of the product.
Agricultural Engineer, what would you a. Operating time
advise to the farmer-owner? b. Timeliness
a. Bring the bearing to the manufacturer c. Functional time
of the thresher for replacement . d. None of the above
b. Buy a new bearing and replace it.
c. Ask the manufacturer to pay for the 17. These are periodic activities that must
bearing unit. be done to prevent premature failure of a
d. None of the above machine and to maintain its good
functional performance.
14. The handle of a power tiller was a. Repair
detached from the main frame assembly b. Major overhaul
due to a tremendous lateral load during c. Maintenance and Service
plowing. The machine was purchased 4 d. All of the above
months ago from a nearby manufacturer.
What would you recommend to the 18. An assurance of the quality of
farmer? materials and of workmanship of the
a. Buy new handle and replace the old product offered for sale or the length of
one. satisfactory use expected from a product
b. Consult the manufacturer and request under normal use.
for replacement of the handle. a. After-sales service
c. Bring a welding machine and weld the b. Warranty
handle to the frame of the power tiller. c. Insurance
d. None of the above d. None of the above

19. The pattern efficiency of a circuitous-


type field machine on a rectangular field.
a. 93.0%
b. 95.7%
c. 94.8%
d. None of the above
VIII. TILLAGE EQUIPMENT

Tillage - mechanical manipulation of soil


for any desired purpose.

Primary Objectives

• Elimination and permanent control of


the original vegetation (weeds);
• Creation of conditions favoring the
germination, emergence, and growth
of the cultivated plants; and
• Conservation and improvements of the
soil as the growth medium cultivated
crops.

Basic Operation

• Reduction of volume (compaction) of


the soil
• Cutting of soil
• Shear plane formation (crumbling,
pulverization) in the soil
• Transport or movement of soil from
one place to another place
• Deformation of soil mass

Terminologies
Furrow – trench left when the plow
bottom cuts and turns the furrow slice.
Back Furrow – raise ridge left at the
center of the strip of land when plowing is
started from center to side. Furrow Slice – soil mass cut, lifted,
pulverized, inverted, and thrown to one
Dead Furrow – open trench left between side of the plow bottom
the adjacent strips of land after finishing
Headland – unplowed soil at the end of
plowing operation.
the furrow strip.
Furrow Crown – peak of the turned
furrow slice. Ridge Height – bed height, hill height,
windrow height, or height of the soil
Furrow Depth – ditch depth, pat depth, above a specified soil surface.
trench depth, or depth of depression
below a specified soil surface
Classifications of Tillage Operation

A. Primary Tillage – constitutes the


initial major soil working operation. It
is normally designed to reduce soil
strength, cover plant residues, and
rearrange aggregates. Common
equipment used for this operation are
as follows:

1. Moldboard Plow
2. Disk plow
3. Rotary Plow
4. Chisel Plow
5. Subsoiler

Root Bed – soil profile modified by tillage


or amendments for use by plant roots.
Root Zone – part of the soil profile
exploited by the roots of the plants.
Seedbed – soil zone which affects
germination and emergence of seeds.
Plow Draft

Amount of pull required to move the


plow. Plow draft is dependent on the
size of the plow and the depth of
plowing. In plowing, at least ¾ of the
available power should be used.

One moldboard plow bottom requires


about 10-20 hp in order to pull the
implement.
Effects of Primary Tillage Equipment
Implement Mold- Disk Rotary Chisel
Effect board Plow Plow Plow
Factors Affecting the Draft of Plows
Plow

Pulverizing Medium Medium Big Small • Soil condition


Loosening Big Medium Medium Big • Topography
Mixing Small Medium Big Small • Adjustment of plow
• Hitch of the tractor
Inverting Big Medium Small Small
• Depth and rate of plowing
• Sharpness of shares, coulters, and
jointers
Factors Affecting the Energy Requirement
of Tillage Operation
Center of Resistance of the Plow
• Soil type Point where all the horizontal and
• Condition of the soil (Moisture content, vertical forces meets together. It is also
bulk density, and structure) known as Plow Center Resistance.
• Shape working depth, and speed of tool
• Climatic condition Moldboard – at the intersection
between the share and the moldboard
• Extent to which the power source and
and to the right of the shin.
implement or tool match
• Correct adjustment or use of the Disk – at the left and below the center
implement of the disk and is closer to the furrow
wall.
B. Secondary Tillage Equipment - Types of Gang Arrangement of Disk
implement intended to create refined Harrow
soil conditions following primary
tillage operation. Tillage equipment • Single Action - Two gangs placed end
under this classification are as to end which throws the soil opposite
follows: in direction.
• Double Action or Tandem – Two sets
1. Harrow (spike tooth, spring tooth, of gangs follow behind the front gangs
disk, power harrows) so as the front gang throws the soil
2. Roller outward while the rear gang throws
3. Pulverizer the soil inward.
4. Mulcher • Offset – The harrow is set in an offset
5. Lister position either on the right or on the
left of the tractor.

Single-Action & Tandem

Factors Affecting the Penetration of Offset Tandem


Disk Harrow

• Angle of disk gang


• Weight of the harrow
• Sharpness of disks
• Size of disks
• Concavity of disk
• Angle of hitch
Classifications of Tractor Mounted
Implement

Trailing – The implement is attached to


the tractor hitch and is being adjusted
mechanically both for vertical and
horizontal operations.

Semi-Mounted – Called as direct Integral-Mounted


connected implement wherein the front
of the implement is connected to the Drawbar Full
tractor while the furrow wheel of the Force required to pull an implement. A
implement supports the rear end. The function of the surface of the implement
hydraulic system of the tractor controls that reacts with the soil, the soil draft, and
the furrow wheel and sometimes the the number of implement bottoms. Also it
ground wheel is provided to control the is the specific draft of the implement
depth of tillage operation. multiply by the width of the implement.

Integral Mounted – The implement is Drawbar Horsepower


mounted at the rear of the tractor 3- Power required to move the implement
point hitch system where it is lifted or during the operation. This can be
set for operation by the hydraulic determined by multiplying the drawbar
system of the tractor. The lifting and pull with the velocity of travel of the
setting of the implement is manually implement.
controlled by the tractor operator.
Theoretical Field Capacity
Rate of tilling the field at a given width and
velocity without any changes in width of
cut and speed of travel of the machine
during operation.

Effective Field Capacity


Rate of tilling the field at a given width and
velocity considering there are changes in
Trailing
the actual field of cut and time loss during
operation.

Field Efficiency
Ratio of the effective field capacity to the
theoretical field capacity multiply by 100.

Semi-Mounted
Power Requirement and Field Efficiency of Various Tillage Implement
Equipment Draft Requirement Speed Field Efficiency
(kph) (%)
Plow, indigenous 0.14-0.7 kg/cm2 1.6 - 3.5 30-60
Plow, moldboard 0.21-1.12 kg/cm2 2.4 - 5.0 30-80
Plow, disk 0.21-1.00 kg/cm2 2.5 - 5.0 30-80
Disk Harrow, single action 0.45-1.50 kg/cm 1.6 - 4.2 65-85
Disk Harrow, double action 1.20-2.70 kg/cm 1.5 - 4.0 60-80
Rotary Tiller 0.70-3.50 kg/cm 0.8 - 2.8 60-80
Harrow, Spike/peg 1.80-2.70 kg/peg 1.6 - 6.0 70-90
Harrow, spring tine 10.0-25.0 kg/tine 1.6 - 5.0 70-90
Rolling or Leveling 0.15-0.90 kg/cm 0.8 - 2.5 60-80
Cultivating 6-20 kg/shank 1.6 - 4.0 60-85

Implement Cutting Width based on ASABE


Disk Plow W = 0.95 N S + 0.3 D
Single Disk Harrow W = 0.95 N S + 0.3 D
Tandem Disk Harrow W = 0.95 N S + 1.2 D
Double Offset Disk Harrow W = 0.95 N S + 0.85 D
Offset Disk Harrow W = 0.95 N S + 0.6 D

where:
N - number of spaces between disk blades
S - blade spacing, m
D - diameter of disk blades, m
Other Tillage Implement

1. Roller Packer – Implement for


crushing soil clods.
2. Roller Harrow – Implement used for
seedbed preparation which crushes
the soil clods and smooth and firms
the soil surface. Bed Shaper
3. Bed Shaper – Soil-handling implement
which forms uniform ridges of soil to
predetermined shapes.
4. Culti-Packer – Used to compact the
surface of the soil in order to minimize
water evaporation.
5. Soil Sifter – Used to remove stones
from the field. Culti-Packer
6. Soil Chopper – Used to form non-
uniform ridges to entrapped water
especially when rained or applied with
surface water during irrigation.
7. Soil Dredger – Used to construct
irrigation and drainage canal.

Soil Sifter

Roller Packer

Soil Dredger

Power Harrow
USEFUL FORMULA

Plow Area of Cut Draft of Plow

A c = W c Dc F = A c δs
where: where:
A c - area of cut of plow, m2 F - draft of plow, kg
W c - width of cut, m A c - area of cut, m2
Dc - depth of cut, m δs - specific resistance of soil, kg/m2
Drawbar Horsepower Field Efficiency
F V EFC
DHP = ----------- ξf = --------- x 100
76.2 TFC
where: where:
DHP - drawbar horsepower ξf - field efficiency, %
F - draft of implement, kg EFC - effective field capacity, ha/hr
V - velocity of implement, m/s TFC - theoretical field capacity, ha/hr
Theoretical Field Capacity Effective Field Capacity

TFC = 0.1 W i Vi EFC = TFC ξf


where: where:
TFC - theoretical field capacity, ha/hr EFC - effective field capacity, ha/hr
W i - width of implement, m TFC - theoretical field capacity, ha/hr
Vi - implement speed, kph ξf - field efficiency, decimal
Number of Implement Unit Time to Finish Tillage Operation
Af Af
NI = --------------- To = ---------------
To EFC EFC NI
where: where:
NI - number of implement units To - time required to finish tillage
A f - area of the farm, ha operation, hr
To - total operating time to finish A f - area of the farm, ha
operation, hr EFC - effective field capacity, ha/hr
EFC - effective field capacity of NI - number of tillage implement
implement, ha/hr
Field Efficiency Fuel Consumption Rate

Fe = (efc / tfc) 100 FC = V / t


where: where:
Fe - field efficiency, % FC - fuel consumption, lph
efc - effective field capacity, m2 /hr V - volume of fuel consumed, l
tfc - theoretical field capacity, m2 /hr t - total operating time, hr
Width-of-Cut of Disc Plow Width-of-Cut of Disc Harrow
(Single Action)
0.95 N S + D 0.95 N S + 0.3 D
W = --------------------------- W = ------------------------------
1000 1000
where: where:
W - width of cut, m W - width of cut, m
N - number of disk N - number of disk
S - disk spacing, mm S - disk spacing, mm
D - diameter of disk, mm D - diameter of disk, mm
Width-of-Cut of Disc Harrow Width-of-Cut of Disc Harrow
(Tandem Type) (Offset Type)
0.95 N S + 1.2 D 0.95 N S + 0.6 D
W = ---------------------------- W = -----------------------------
1000 1000
where: where:
W - width of cut, m W - width of cut, m
N - number of disk N - number of disk
S - disk spacing, mm S - disk spacing, mm
D - diameter of disk, mm D - diameter of disk, mm
Draft of Moldboard Plow

D = 7.0 + 0.049 S2 : silty clay


D = 6.0 + 0.053 S2 : clay loam
D = 3.0 + 0.021 S2 : loam
D = 3.0 + 0.056 S2 : sandy silt
D = 2.8 + 0.013 S2 : sandy loam
D = 2.0 + 0.013 S2 : sand
where:
D - unit draft of implement, N/cm2
S - implement speed, kph
SAMPLE PROBLEMS

Problem 1

A tractor pulls a 1m-wide heavy-duty offset disk harrow at 6kph operating speed.
What is the theoretical field capacity of the machine? What is the effective field
capacity if field efficiency is 80%?

Given:
Implement - heavy-duty offset-disk harrow
W - 1.0 m
V - 6 kph
Required: Theoretical Field Capacity
Solution:

TFC = 0.1 W V
= 0.1 (1 m) (6 kph)
= 0.60 ha/hr
EFC =. 0.48 ha/hr

Problem 2

A rotary tiller has a 2m operating width. If the tractor runs at 2kph speed, what is
the theoretical field capacity of the machine?

Given:
Operating width - 2 m
Operating speed - 2 kph

Required: Theoretical Field Capacity

Solution:

TFC = 0.1 W V
= 0.1 (2 m) (2 kph)
= 0.4 ha/hr
Problem 3

What is the force required to pull a single-moldboard plow on a clay loam soil
having 30cm width-of-cut and 15cm depth-of-cut? Soil draft is 8 psi.

Given:
Width-of-cut - 30 cm
Depth-of-cut - 15 cm
Soil draft - 8 psi

Required: Force to Pull the Plow

Solution:

F = A Ds
= (30 cm) (15 cm) (8 psi x in/2.54 cm x kg/2.2 lb)
= 253 kg

Problem 4

If the plow in Problem 3 has 5kph average speed, what is its drawbar horsepower?

Given:
Force to pull the plow - 253 kg
Velocity of the plow - 5 kph

Required:
Drawbar Horsepower

Solution:

DHP = F V
= 253 kg (5 kph x 1000 m/km x hr/3600 sec)
= 4.6 hp
Problem 5

What is the effective field capacity of a rotary tiller tilling theoretically at 0.4 ha/hr,
with 85% field efficiency?

Given:
Theoretical field capacity - 0.4 ha/hr
Field efficiency - 85%

Required: Effective Field Capacity

Solution:

EFC = TFC ξf
= 0.4 ha/hr x 0.85
= 0.34 ha/hr

Problem 6

What is the effective field capacity of 2 moldboard plows having 25cm width,
each plowing at 7kph speed? The field efficiency is 80%.

Given:
Number of plows - 2
Plowing speed - 7 kph
Field efficiency - 80%

Required: Effective Field Capacity

Solution:

EFC = Np W ξf
= 2 (0.25 m) (7 kph) (0.8)
= 0.28 ha/hr
Problem 7

The field capacity of a rotary tiller was measured at 0.23 ha/hr. If the operating
width and speed of the machine is 1.5 m and 2 kph, respectively, what is the field
efficiency of the tiller?

Given:
Actual field capacity - 0.23 ha/hr
Operating width - 1.5 m
Operating speed - 2 kph

Required: Field Efficiency

Solution:

ξf = AFC x 100 / TFC


TFC = 0.1 (1.5 m) (2 kph)
= 0.3 ha/hr

ξf = 0.23 ha/hr (100) / 0.3 ha/hr


= 77%

Problem 8

What is the theoretical field capacity of a 3m-wide tandem-disk harrow pulled by a


4-wheel-drive tractor running at 10kph speed?

Given:
Width of implement - 3.0 m
Speed of the tractor - 10 kph

Required: Theoretical Field Capacity

Solution:

TFC = 0.1 W V
= 0.1 (3 m) (10 kph)
= 3 ha/hr
Problem 9

A 3-bottom disk plow is plowing at a speed of 8 kph. The effective width per plow
bottom is 25 cm. If the plowing efficiency is 80%, what is the theoretical and the
effective field capacities of the implement?

Given:
Plow - disk
Number of disk -3
Speed - 8 kph
Width - 25 cm/plow
Efficienty - 80%

Required: Theoretical Field Capacity


Effective Field Capacity
Solution:
TFC = b V
= 0.25 x 0.1 x 3 x 8
= 0.6 ha/hr

EFC = TFC ξf
= 0.6 ha/hr x 0.8
= 0.48 ha/hr

Problem 10

Find the effective field capacity of a 5-bottom disk plow with 25cm width-of-cut
per plow bottom. Assume a 0.8 field efficiency and 7kph operating speed.

Given:
Plow type - disk
Number of disk - 5
Field efficiency - 80%
Speed - 7 kph

Required: Effective Field Capacity

Solution:

EFC = bV ξf
= 0.1 x 0.25 cm/disk x 7 kph x 5 disks x 0.8
= 0.70 ha/hr
Problem 11

Compute the drawbar pull and the drawbar horsepower of a 4-bottom moldboard
plow on a clay loam soil having 0.3m effective width-of-cut per bottom and 0.15m
depth-of-cut. The plowing speed is 10 kph. (Soil draft for clay loam soil is assumed
at 8 psi).
Given:
Plow - moldboard
No. of Plow - 4
Width of Cut - 0.3 m
Depth of Cut - 0.15 cm
Speed - 10 kph
Soil Draft - 8 psi
Required: Drawbar Pull
Drawbar Horsepower
Solution:
F = 0.15 m x 0.30 m x 4 plows x 5818.2 kg/m3
= 1014 kg
DHP = (1014 kg x 10 kph x 1000 m/km x hr/3600 sec) / 76.2
= 36.9 hp

Problem 12

A 3-bottom plow is plowing at 8kph speed. The effective width per plow bottom is
25 cm. If the plowing efficiency is 80%, what are the theoretical and effective field
capacities of the plow?

Given:
Number of plow - 3 bottom
Speed - 8 kph
Effective width/plow - 25 cm
Plowing efficiency - 80%
Required: Theoretical Field Capacity
Effective Field Capacity
Solution:
TFC = 0.1 (3 plows)(0.25 m/plow)(8 kph) /0.80
= 0.75 ha/hr
EFC = 0.1 (3 plows) (0.25 m/plow) (8kph)
= 0.60 ha/hr
Problem 13

What is the drawbar pull required for a 3m tractor-drawn offset-disk harrow? The
implement specific draft is 89 kg/m. Also, compute the drawbar horsepower of the
implement if it is running at 20kph speed.

Given:
Harrow - offset disk
Wc - 3m
δ - 89 kg/m
V - 20 kph
Required: Drawbar Pull
Drawbar Horsepower
Solution:
F = Wc x δ
= 3 m x 89 kg/m
= 267 kg
DHP = (267 kg x 20 kph x 1000 m/km x hr/3600 sec)/ 76.2 kg-m/sec-hp
= 19.5 hp

Problem 14

A power harrow has 6m operating width. If the tractor runs at 20kph speed, what
is the theoretical field capacity of the machine? What is the effective field capacity
of the harrow if its field efficiency is 0.85?

Given:
Operating width - 6m
V - 20 kph
Required: Theoretical Field Capacity
Effective Field Capacity
Solution:
TFC = 0.1 b V
= 0.1 x 6 x 20
= 12 ha/hr
EFC = TFC ξf
= 12 x 0.85
= 10.2 ha/hr
Problem 15

The field capacity of a tractor-drawn field cultivator was measured at 2.9 ha/hr. If
the operating width and speed of the machine is 2.5 m and 13 kph, respectively,
what is the field efficiency of the tiller?

Given:
Effective Field Capacity - 2.9 ha/hr
Operating Width - 2.5 m
Travel Speed - 13 kph

Required: Field Efficiency

Solution:

ξf = 2.9 ha/hr x 100 / (0.1 x 2.5 m x 13 kph)


= (2.9 ha/hr / 3.25 ha/hr) x 100
= 89 %

Problem 16

A 16hp rotary tiller is tilling at 10cm depth. Its rotating tiller has 0.5m width. What
is the specific power output of the machine if it is traveling at 0.5mps speed?
Assume a 0.80 power transmission efficiency for the tiller.

Given:
Power Input - 16 hp
Depth-of-Cut - 10 cm
Width-of-Cut - 0.5 m
Speed of Tiller - 0.5 m/sec
Power Trans Eff - 0.80

Required: Specific Power Output

Solution:

SPO = (16 Hp x 0.80 x 746 watt/Hp) / [100 cm/m (50 cm) (10 cm) (0.5 m/sec)]
= 0.382 watts/cm3
Problem 17

A 30-disk double-offset disk harrow is travelling at 10kph speed. If the disk diameter
is 40 cm spaced at 30 cm, what is the cutting width of the harrow? What is its
theoretical field capacity?

Given:
Type of harrow - double-offset
Number of disk - 30 disks
Travelling speed - 10 kph
Disk diameter - 40 cm
Disk spacing - 30 cm
Required: Cutting Width
Theoretical Field Capacity
Solution:
Wc = [0.95 N S + 0.85 D] / 1000
= 0.95 (29 disks x 300 mm) + 0.85 (400 mm) / 1000
= 8.6 m
TFC = 0.1 W V
= 0.1 x 8.6 m x 10 kph
= 8.6 ha/hr

Problem 18

A 16hp rotary tiller is tilling at 10cm depth. Its rotating tiller has 1.2m width. What
is the specific power output of the machine if it is traveling at 0.5mps speed?
Assume a 90% power transmission efficiency for the tiller.

Given:
Power Input - 16 hp
Depth-of-Cut - 10 cm
Width-of-Cut - 1.2 m
Speed of Tiller - 0.5 m/sec
Power Trans Eff - 0.90

Required: Specific Power Output

Solution:

SPO = (16 Hp x 0.90 x 746 watt/Hp)/ (100 (120 cm) (10 cm) (0.5 m/sec)
= 0.178 watts/cm3
Problem19

Compute the drawbar pull of a plow on a power tiller with 10cm x 8cm depth- and
width-of-cut. The number of plow bottom is 2. Soil draft is equal to 5 psi. If the
plow is drawn at 10kph speed, what is the drawbar horsepower of the plow?

Given:
Width of Cut - 8 cm
Depth of Cut - 10 cm
No. of Plow - 2
Draft - 5 psi
Speed - 10 kph
Required: Drawbar Pull and Power
Solution:
A = 10 cm x 8 cm x 2 plows
= (80 cm2 /plow x 2 plows)
= 160 cm2
F = 160 cm2 (5 lb/in2 ) (in./ 2.54 cm)2 (kg/2.2 lb)
= 56.36 kg
DHP = F x V
= (56.36 kg x 10 kph x 1000 m/km x hr/3600sec)/76.2 kg-m/sec-Hp
= 2.04 Hp

Problem 20

The effective width-of-cut of a single-bottom moldboard plow drawn by a power tiller


is 10 cm. If the machine is used to plow a 2-hectare farm, what would be the total
distance to be traveled by the machine to accomplish the task?

Given:
Implement - moldboard plow
Effective width of cut - 10 cm
Area to be plowed - 2 has.

Required: Distance to be traveled by the plow

Solution:

D = Area to be plowed / Effective width-of-cut


= (2 hectares x 10,000 m2 /ha) / (10 cm x m/100 cm)
= 200 km
Problem 21

A power tiller was tested on a 10m-wide plot. During the test, the machine made
22 rounds to complete the plowing operation using two 0.3m-diameter disk plows.
What is the average swath of the power tiller? Also, compute for the disk angle of
the plow.
Given:
Machine - power tiller
Width of plot - 10 m
Number of rounds - 22 rounds
Required: Average Swath of the Power Tiller
Disk Angle of the Plow
Solution:
S = width of plot / (2 x number of rounds)
= 10 m / [2 ( 22 rounds)]
= 0.23 m
Θ = sin -1 (0.23/0.3)
= 50 deg

Problem 22

What is the required drawbar pull of a 5m tractor-drawn spike-tooth harrow with


89 kg/m specific draft? What is the drawbar horsepower of the implement if it is
running at 6kph speed?

Given:
Width of implement - 5 m
Specific draft - 89 kg/m
Speed - 6 kph
Required: Drawbar Pull
Solution:
F = Ds W
= 89 kg/m) ( 5 m)
= 445 kg
DHP = F V
= (445 kg)(6 kph)(1000 m/km) (hr/3600 sec) /76.2 kg-m/sec-hp
= 10 hp
Problem 23

A single-bottom moldboard is plowing on a clay loam soil (8 psi draft) having 0.3m
effective width-of-cut and 15cm depth-of-cut. What is the force exerted by the
plow?

Given:
Plow - single-bottom moldboard
Soil draft - 8 psi
Width of cut - 0.3 m
Depth of cut - 0.15 m

Required: Force Exerted by the Plow

Solution:

F = A ds
= (0.3 m) (0.15 m) (8 psi) (in/0.254 m)2 (kg/2.2 lb)
= 253 kg

Problem 24

What is the drawbar horsepower of the plow in Problem 23 if its plowing speed is 5
kph?

Given:
Force - 253 kg
Velocity - 5 kph

Required: Drawbar Horsepower

Solution:

DHP = F V
= 253 kg (5 kph) (1000 m/km) (hr/3600 sec)
= 4.6 hp
Problem 25

In a 200-hectare corn project, the field has to be tilled within 30 days. What size of
a heavy-duty offset-disk harrow should be selected if the tractor to be used will be
operated at 5kph speed? Assume a 75% field efficiency and an 8-hour working
day.

Given:
Area - 200 has.
Tilling period - 30 days
Implement velocity - 5 kph
Field efficiency - 75%
Operating time - 8 hours per day

Required: Size of Heavy-duty Offset-disk


Harrow

Solution:

RFC = A /[Pt x OT x Eff]


= 200 hectares/[30 days x 8hr/day x 0.75]
= 1.11 ha/hr

W = RFC / [0.1 x V]
= 1.11 ha/hr / [0.1 x 5 kph]
= 2.2 m
Problem 26

Assume that you are the Agricultural Engineer in a 500-hectare corn farm with soil
type that gives 10psi draft when plowing at 4kph speed. Compute the drawbar
horsepower required to pull a 3-disk plow with 12inch-per-disk effective cut at 6
inches plowing depth.

Given:
Area - 500 ha. corn farm
Soil draft - 10 psi
Plowing speed - 4 kph
Plow bottom - 3 disk
Width-of-cut - 12 in. per disk
Plowing depth - 6 inches

Required: Drawbar Horsepower

Solution:

DPH = [3 x 12 in x 6 in x 10 psi x 4 kph x 1000 m/km x hr/3600 sec] / 76.2


kg-m/sec-hp
= 14.3 hp

Problem 27

Referring to Problem 26, how many hectares does the tractor pulling the 3-disk
plow would be able to plow in one day (16 hours operation)? Assume a 30% total
time loss.

Given:
No. of disk - 3 disk
Width-of-cut - 12 in. per disk
Operating time - 16 hours per day
Time loss - 30%

Required: Effective Field Capacity


Solution:

EFC = 0.1 (3 disks x 12 in/disk x 0.254 m/in) (4 kph) (0.7)


= 0.36 ha/hr x 16hr/day
= 4.02 ha/day
Problem 28

Referring to Problem 26, how many days are required to plow the 500-hectare
farm using 10 units of the same kind of tractor and of plow?

Given:
Area to be plowed - 500 has.
Number of tractors - 10 units

Required: Number of Days to Finish


Plowing the Area

Solution:

Nd = 500 has. / [10 tractors x 4.02 ha/day-tractor]


= 12.5 days

Problem 29

Still referring to Problem 26, what is the amount of fuel consumed, in liters, per 16-
hour/day operation of one tractor if the specific fuel consumption is 1 lb/dhp-hr
and the specific gravity of the fuel is 0.8?

Given:
Specific fuel consumption - 1 lb/dhp-hr
Specific gravity - 0.8

Required: Fuel Consumed per


16 hours-tractor

Solution:

FC = 1 lb/dhp-hr x 14.3 hp / [0.8 x 62.4 lb/ft3 ]


= 0.286 ft3 of fuel /hr x (30 cm/ft)3 x liter/1000 cc
= 7.73 liters per hour

Vol of fuel = (7.73 liter/hr) (16hours/day) (12.5 days)


= 1546.87 liters
Problem 30

What is the cutting width of the single-disk harrow set at 18-degree working angle
containing 20 disks spaced at 30 cm? Disk diameter is 40 cm.

Given:
Number of disks - 20 disks
Disk spacing - 30 cm
Disk diameter - 40 cm

Required: Cutting Width

Solution:

W = [0.95 N S + 0.3 D] / 1000


= 0.95 (20 disks x 300 mm) + 0.3 (400 mm) / 1000
= 5.82 m

Problem 31

If the harrow in Problem 30 is a tandem disk harrow, what is its cutting width?

Given:
Type of harrow - tandem disk harrow

Required: Width-of-Cut

Solution:

Wc = [0.95 N S + 1.2 D] / 1000


= [0.95 (20 disks x 300 mm) + 1.2(400 mm)] / 1000
= 6.18 m
Problem 32

At 18-degrees working angle, a tandem disk harrow is used to pulverize a newly


plowed farm at 10kph speed. Compute the effective width of the implement, the
theoretical field capacity, and the effective field capacity of the harrow. Assume a
90% field efficiency. The harrow has 24 pieces 30cm disk spaced at 20 cm.

Given:
Harrow - tandem
Work angle - 18 deg
Speed - 10 kph
Field Eff - 90%
No. Disks - 24 units
Disk Dia - 30 cm
Spacing - 20 cm

Required: Effective Width


Theoretical Field Capacity
Effective Field Capacity

Solution:

(a) Effective Width


We = 0.95 N S + 1.2 D
= 0.95 (23) (20 cm) + 1.2 (30 cm)
= 473 cm

(b) Theoretical Field Capacity


TFC = 0.1 W V
= 0.1 x 473 cm x m/100 cm x 10 kph
= 4.73 ha/hr

(c ) Effective Field Capacity


EFC = TFC ξf
= 4.73 ha/hr x 0.90
= 4.26 ha/hr
Problem 33

Five hundred hectares of farm is to be plowed using a 5-bottom moldboard plow.


The cutting width of each plow bottom is 20 cm with 15cm cutting depth. If three
(3) tractors are pulling a plow at an average speed of 15kph, how many days will it
take to finish the whole area? Assume an 85% plowing efficiency and a 6hour-per-
day plowing operation. Also, compute the drawbar hp of the plow if the specific
draft of soil is equal to 12 psi.
Given:
Area - 500 has.
No. of Plow - 5
Width-of-Cut - 20 cm
Depth-of-Cut - 15 cm
No of Tractors - 3 units
Speed of Tractor - 15 kph
Plowing Eff - 85%
Operation - 6 hr/day
Soil Draft - 12 psi

Required: Days to Finish Plowing


Drawbar Horsepower of the Plow
Solution:
W input = 20 cm x 5 bottoms
= 100 cm
TFC = (0.1 x 100 cm x 15 kph) / (100 cm/m)
= 1.5 ha/day
TFE = 0.85 (1.5 ha/day)
= 1.275 ha/hr-unit
Capacity for 3 tractor unit = 1.275 ha/hr-unit x 3 units
= 3.825 ha/hr x 6 hrs/day
= 22.95 ha/day
No. of days to finish plowing = 500 ha / (22.95 ha/day)
= 21.78 days
Drawbar Hp = F x V
A = 20 cm/plow x 5 plows x 15 cm
= 1500 cm2
F = (1500 cm2 ) x 12 psi x (1 in/2.54 cm)2 x kg/2.2 lb
= 1268.2 kg
DHP = (1268.2 kg x 15 kph x 1000 m/km x hr/3600 sec ) / 76.2 kg-m/sec-hp
= 69 hp
Problem 34

What is the required drawbar pull for a 5m tractor-drawn spike-tooth harrow with
89 kg/m specific draft? What is the drawbar-horsepower of the implement if it is
running at 6kph speed?

Given:
Harrow - spike tooth harrow
Wc - 5m
δ - 89 kg/m
V - 6 kph
Required: Drawbar Pull
Drawbar Horsepower
Solution:
F = Wc x δ
= 5 m x 89 kg/m
= 445 kg
DHP = (445 kg x 6 kph x 1000 m/km x hr/3600 sec)/ 76.2 kg-m/sec-hp
= 9.7 hp use 10 hp

Problem 35
Determine the width-of-cut of six 800mm diameter disk plows , with 350mm disk
spacing.

Given:

W – 800 mm
No. of Disc – 6
S – 350 mm

Required: W

W = [0.95 ( 6 x 350 mm) + 800 mm] / 1000


= 2.795 m
Problem 36
What is the effective field capacity of eight 900mm-diameter disk plow running at
10kph speed. The disks are spaced at 500 mm. Assume an 80% plowing efficiency.

Given:
No. of Discs - 8 units of disc plow
Diameter of disc - 900
Disc spaces - 500 mm
Plowing Efficiency - 80%

Required: Effective field capacity

Solution:

Wc = [0.95 (8 discs x 500 mm) + 900 mm] / 1000


= 3.42 m

EFC = 0.1 [3.42 m x 10 kph x 0.8]


= 2.74 ha/hr
REFERENCES

ASAE. ASAE Standards 1997. Standard


Engineering Practices Data. American
Society of Agricultural Engineers. The
Society for Engineering in Agricultural,
Food, and Biological Systems. 2950 Niles
Road, St. Joseph, MI 49085-9659 USA. Pp.
264-275.
Campbell, J. K. 1990. Dibble Sticks,
Donkey, and Diesel. International Rice
Research Institute. Los Banos, Laguna,
Philippines. 329pp.
Hunt, D. 1982. Farm Power and
Machinery Management. Eight Edition.
Iowa State University Press. Ames, Iowa.
325pp.
Smith, H. P. and L. H. Wilkes. 1977. Farm
Machinery and Equipment. Sixth Edition.
Tata McGraw Hill Publishing Company Ltd.
New Delhi, India. 487pp.
QUESTIONS and PROBLEMS

1. Mechanical manipulation of soil for 7. Primary tillage operation includes:


any desired purpose. a. pulverizing
a. Land grading b. mulching
b. Tillage c. plowing
c. Earth moving d. All of the above
d. None of the above
8. Secondary tillage operation includes:
2. Which of the following is a primary a. listing
objective of tillage? b. subsoiling
a. To eliminate and to permanently c. harrowing
control the growth of weeds. d. All of the above
b. To create a favorable condition for
germination, emergence and growth of 9. Tillage operation traditionally
the cultivated plants. performed in preparing seedbeds for a
c. To conserve and to improve the soil as given crop commonly grown in a given
a medium for the growth of cultivated geographical area.
crops.
a. Conventional tillage
d. All of the above
b. Conservation tillage
c. Strip tillage
3. General fragmentation of soil mass
d. None of the above
resulting from the action of tillage forces.
a. Soil reaction
10. Primary tillage operation that
b. Soil shatter
manipulates the soil to a depth greater
c. Soil sliding
than 300 mm.
d. None of the above
a. Reservoir tillage
4. Which of the following is not basic to b. Deep tillage
tillage operation? c. Ridge tillage
a. Reduction of volume of the soil d. None of the above
b. Cutting of soil
c. Deformation of soil mass 11. Minimum soil manipulation necessary
d. None of the above for crop production or for meeting tillage
requirements under existing soil
5. Operation that constitutes the initial condition.
major soil working operation. a. No-tillage planting
a. Primary tillage
b. Much tillage
b. Secondary tillage
c. All of the above c. Minimum tillage
d. None of the above d. None of the above

6. Operation intended to create refined 12. Tillage or soil preparation in which


soil conditions. plant residues are specifically left on or
a. Primary tillage near the surface.
b. Secondary tillage a. Strip tillage
c. All of the above b. Mulch tillage
d. None of the above c. Optimum tillage
d. None of the above
13. Tillage system that operates with 19. Primary or secondary tillage
fewer or lesser energy compared with implement used for broadcast or for strip
that of the conventional tillage. tillage and also used as a chemical
a. Minimum tillage incorporator prior to planting.
b. No-till tillage a. Roller tiller
c. Reduce tillage b. Roller harrow
d. All of the above c. Packer
d. None of the above
14. When 30% or less of the soil surface is
tilled, the tillage operation employed is
20. Secondary tillage implement
____.
consisting of one or two in-line gang
a. minimum tillage
rollers used for crushing soil clods and for
b. strip tillage
compacting the soil.
c. ridge tillage
a. Roller tiller
d. All of the above
b. Packer
15. Which of the following affects the c. Roller harrow
energy requirement in tillage operation? d. None of the above
a. Type and condition of the soil
b. Shape, working depth and speed of 21. Tillage and land-forming operation
tool using a tool that splits the soil and turns
c.All of the above two furrows laterally in opposite
d.None of the above directions, thereby providing a ridge-and-
furrow soil configuration.
16. Equipment, which displaces and a. Listing
shatters soil, used to reduce soil strength b. Harrowing
and to bury or mix plant materials and
c. Chiseling
fertilizers in the tilled layer. d. None of the above
a. Primary tillage equipment
b. Secondary tillage equipment 22. Secondary tillage operation which
c. Seeding and planting equipment pulverizes, smoothens and packs the soil
d. None of the above during seedbed preparation and also
controls weeds.
17. Pushing or rolling of soil by a steeply
a. Harrowing
inclined blade.
b. Listing
a. Bulldozing
c. Chiseling
b. Chiseling
d. None of the above
c. Subsoiling
d. None of the above 23. Deep chiseling, below 16 inches, for
18. Soil-tool reaction in which soil slides the purpose of loosening the soil for root
along the surface of the tillage tool growth and/or for water movement.
without significant adhesion. a. Ridging
a. Sliding b. Chiseling
b. Scouring c. Subsoiling
c. Soil cutting d. None of the above
d. None of the above
24. Secondary-tillage implement used to 30. Power requirement of a rotary tiller
stir the soil using multiple number of for primary tillage.
disk blades arranged on a gang. a. 50 kW/m
a. Spike-tooth harrow b. 30 kW/m
b. Disk harrow c. 70 kW/m
c. All of the above d. All of the above
d. None of the above
31. When a material is to be inserted into
25. Tillage implement which shatters the the soil, the tillage to use is ____.
soil without complete burial or mixing of a. jointer
surface materials. b. coulter
a. Chisel plow c. injector
b. Lister plow d. All of the above
c. Disk plow
d. None of the above 32. Machine designed for use with small
plow or harrow.
26. Primary-tillage equipment used for a. Puddling-type floating tiller
small rice fields. b. Power tiller
a. Moldboard c. Hydro tiller
b. Rotary d. All of the above
c. Spiral
d. All of the above 33. A tractor pulls a 1m-wide heavy-duty
offset disk harrow at 6kph operating
27. Tillage implement consisting of two speed. What is the theoretical field
or four gangs of concave disks. capacity of the machine?
a. Disk plow a. 0.37 ha/hr
b. Disk harrow b. 0.42 ha/hr
c. Cultivator c. 0.60 ha/hr
d. None of the above d. None of the above

28. Primary-tillage plow for rice farming 34. What is the effective field capacity of
that is attached to the power tiller as a the machine in Item 33 above if field
replacement for the cage wheel. efficiency is 80%?
a. Rotary a. 0.24 ha/hr
b. Spiral b. 0.32 ha/hr
c. Disk c. 0.48 ha/hr
d. All of the above d. None of the above

29. Tool that cuts plant materials and soil 35. Secondary-tillage implement used to
ahead of another tool. stir the soil using multiple number of disk
a. Coulter blades arranged on a gang.
b. Jointer a. Spike tooth harrow
c. Injector b. Disk harrow
d. Two of the above c. Two of the above
d. None of the above
36. Hand-operated tractor, which is 41. Minimum peak transmission efficiency
suitable for both primary and secondary for power tillers.
tillage, used when the field has been a. 75 percent
soaked for at least half a day to soften b. 85 percent
the soil. c. 95 percent
a. Power tiller d. None of the above
b. Puddling-type floating tiller
c. Reaper 42. The size of the bottom of a furrower is
d. None of the above measured from ____.
a. the distance between the plow
37. Optimum puddler speed for bottoms
puddling-type floating tiller is ____. b. the left and to the right wing of
a. 100-150 rpm the share of the plow bottom
b. 200-250 rpm c. the height of the share tip to the
c. 300-350 rpm highest tip of the moldboard
d. None of the above d. None of the above
38. Tillage implement resembling a 43. Trench left when the plow bottom
double moldboard used to make ridges cuts and turns the furrow slice.
and beds for planting and trenches for a. Furrow
irrigation and drainage purposes. b. Furrow crown
a. Excavator c. Furrow depth
b. Trencher d. None of the above
c. Furrower
d. All of the above 44. Unbroken side of the furrow is called
____.
39. Special type walking-tractor with a. dead furrow
front-mounted tilling wheel and b. furrow wall
equipped with a floatation structure c. back furrow
commonly used in tilling water-logged d. None of the above
areas.
a. Float-assisted tiller 45. Raise ridge left at the center of the
b. Puddling-type floating tiller strip of land when plowing is started from
c. Hydro tiller center to side.
a. Back furrow
d. All of the above b. Dead furrow
c. Furrow crown
40. Device consisting of a single or a pair
d. None of the above
of wheels with radially-mounted tilling
blades attached to a common shaft and 46. Open trench left between the
supported and powered by the adjacent strips of land after plowing.
transmission drive of a float-assisted a. Furrow crown
tiller. b. Furrow depth
a. Drive wheel c. Dead furrow
b. Tilling wheel d. None of the above
c. Tines
d. All of the above
47. Peak of a turned furrow slice. 54. Toolbar-mounted device used to guide
a. Headland the operator in setting a furrower to
b. Furrow depth ensure uniform furrow spacing during
c. Furrow crown
operation.
d. None of the above
a. Coulter
48. Soil mass cut, lifted, pulverized, b. Jointer
inverted and thrown to one side of the c. Row marker
plow bottom. d. None of the above
a. Furrow depth
b. Furrow slice 55. Bed height, hill height, windrow
c. Furrow crown height, or height of the soil above a
d. None of the above specified soil surface.
a. Ridge height
49. Ditch depth, pat depth, trench depth
b. Furrow crown
or depth of depression below a specified
c. Headland
soil surface.
a. Furrow depth d. None of the above
b. Furrow slice
56. Soil zone which affects germination
c. Furrow crown
and emergence of seeds.
d. None of the above
a. Seedbed
50. Unplowed soil at the end of the b. Root bed
furrow strip. c. Root zone
a. Furrow d. None of the above
b. Ridge height
c. Headland 57. Soil blocks or masses that are cut,
d. None of the above sheared or broken loose by tillage tool.
a. Furrow slice
51. Headland is____. b. Clod
a. the strip of land at the ends of the c. Loose soil
field used for turning machinery d. None of the above
b. a piece of land owned and tilled by
the head of the family 58. If soil clods are needed to be inverted
c. the land at the top of the hill during plowing, the Agricultural Engineer
d. None of the above will recommend the use of a ____.
52. Part of the soil profile exploited by a. disc plow
the roots of the plants. b. moldboard plow
a. Seedbed c. spiral plow
b. Root bed d. All of the above
c. Root zone
d. None of the above 59. Type of plow equipped with plow
bodies that turn furrow slices to one side
53. Soil profile modified by tillage or
amendments for use by plant roots. only.
a. Seedbed a. One-way plow
b. Root bed b. Two-way plow
c. Root zone c. Two of the above
d. None of the above d. None of the above
60. Type of plow which has two sets of 65. Maximum friction in the moldboard
bodies mounted symmetrically on a plow is ____.
share frame which can be rotated over a. at the point of share
180 degrees along the longitudinal axis. b. between the share and the moldboard
a. Reversible plow c. at the moldboard
b. One-way plow d. None of the above
c. Lister plow
d. None of the above 66. Part of the moldboard plow which is
used to cut the soil.
61. Type of plow widely used in breaking a. Share
different types of soil for turning and b. Shin
covering crop residues. c. Moldboard
a. Rotary plow d. All of the above
b. Moldboard plow
c. Disk plow 67. Part of the plow bottom that helps
d. All of the above counteract the side pressure exerted by
furrow slices.
62. Size of a moldboard plow is a. Shin
determined ____. b. Landside
a. from the distance measured at the c. Share
wing of the share to the land side d. None of the above
b. from the width of the plow measured
at the intersection of the share and the 68. Part of the moldboard plow that
moldboard receives furrow slices from the share and
c. from the width of the moldboard turns them to break, crush, and pulverize.
d. None of the above a. Share
b. Shin
63. When plowing requires mixing the c. Moldboard
soil with grasses and weeds, what d. All of the above
implement will you recommend as an
Agricultural Engineer? 69. Type of plow used to break plow sole.
a. Disc plow a. Chisel plow
b. Rotary plow b. Lister plow
c. Moldboard plow c. Subsoiler
d. All of the above d. None of the above
64. Plow consisting of a durably-
70. If the task requires loosening of soil
constructed frame and a vertical leg
drawn to a subsoil in which the bullet- during plowing, a ____ is recommended
to use.
shape attachment creates a tube-like
a. subsoiler
cavity acting as a drainage channel.
a. Subsoiler b. field cultivator
c. chisel plow
b. Bullet plow
c. Mole plow d. None of the above
d. All of the above
71. Type of plow used primarily for 77. Implement used for seedbed
loosening the soil. preparation that crushes soil clods and
a. Lister plow smoothens and hardens soil surfaces.
b. Disk plow a. Roller harrow
c. Chisel plow b. Packer
d. All of the above c. Rotary Hoe
d. None of the above
72. Plow that combines the primary and
secondary tillage and, at the same time, is 78. Tool attached to the shank of the
used to cut and pulverize the soil. subsoiler used to cut through the soil.
a. Disk plow a. Main frame
b. Rotary plow b. Ripper point
c. Chisel plow c. Shank
d. All of the above d. None of the above

73. Structural member of a subsoiler 79. Amount of pull required to move the
commonly used in attaching a primary plow is called____.
tool to a beam. a. side draft
a. Main frame b. center of resistance
b. Ripper point c. plow draft
c. Shank d. None of the above
d. None of the above
80. Plow draft is dependent on the ____.
a. size of the plow and the depth of
74. Type of plow designed to reduce plowing
friction by making a rolling bottom instead b. climatic condition
of sliding along the furrow. c. correct use or adjustment of the
a. Moldboard implement
b. Rotary plow d. All of the above
c. Disk plow
d. None of the above 81. Metal plate attached to the shank to
reduce abrasion and to enhance
75. Moldboard plow has ____. durability of the shank.
a. lower draft than disk plow a. Ripper point
b. higher draft than disk plow b. Wear shin
c. equal draft with disk plow c. Shank cover
d. All of the above d. None of the above
76. When the center-of-load of a plow is 82. If the hitch of a plow is placed above
not on the same line with the center-of- the vertical line of pull, ____.
power, ____. a. there will be lifting action at the front
a. there will be a side draft of the plow and its bottom will tend to
b. tractor will be partly pulled sideways run shallow particularly on hard ground
c. forces on the tractor will be carried by b. the plow share will worn out easily
the rear of the tractor wheel c.the plow bottom will penetrate
d. All of the above d.None of the above
83. Line of pull that is straight from the 89. Disk angle that gives the minimum
center-of-load to the point where draft draft for a disk plow for a given width of
drawbar is attached to the tractor cut.
drawbar. a. 30-35 deg.
a. Horizontal-line draft b. 40-45 deg.
b. Vertical-line draft c. 45-50 deg.
c. Side draft d. None of the above
d. None of the above
90. Angle of the disk plow in reference to
84. Line of draft that is straight from the the vertical plane.
center-of-resistance or load to the point a. Tilt angle
where plow drawbar is attached to the b. Disk angle
tractor drawbar. c. All of the above
a. Horizontal-line draft d. None of the above
b. Vertical-line draft
c. Side draft 91. Angle on a vertical plane between the
d. None of the above centralized tool or reference axis and the
soil surface that is perpendicular to the
85. Percentage of work done in pulling direction of travel .
an average-size plow. a. Tilt angle
a. 50 % b. Lift angle
b. 18 % c. Side angle
c. 30% d. None of the above
d. None of the above
92. Angle of the disk plow in reference to
86. Percentage of work of plow used in the direction of travel of the plow.
turning a slice of soil. a. Tilt angle
a. 34% b. Disk angle
b. 25% c. All of the above
c. 60% d. None of the above
d. None of the above
93. Point in which all the horizontal and
vertical forces in the plow meet.
87. Percentage of work of plow used in
a. Side draft
cutting furrow slices.
b. Center-of-resistance
a. 43%
b. 58% c. Center-of-gravity
d. None of the above
c. 24%
d. None of the above 94. Center-of-resistance of the
moldboard plow is located ____.
88. Force that propels an implement to
a. at its center
the direction of travel.
b. at the intersection of the moldboard
a. Draft
and of the share
b. Drawbar horsepower c. at the center of the share
c. Side draft
d. None of the above
d. None of the above
95. Which of the following affect the draft 101. What is the drawbar-horsepower of
of plow? the implement in Item 100 if it is running
a. Soil condition and topography at 6kph speed?
b. Adjustment of plow and hitch of the a. 8 hp
tractor b. 10 hp
c. Depth and rate of plowing c. 23 hp
d. All of the above d. None of the above

96. Power required to pull an implement. 102. What is the theoretical field capacity
a. Wheel power of a 3m-wide tandem disk harrow pulled
b. Brake power by a 4-wheel-drive tractor running at 10
c. Drawbar power kph?
d. None of the above a. 3.0 ha/hr
b. 4.5 ha/hr
97. Drawbar horsepower is a factor of c. 5.2 ha/hr
____. d. None of the above
a. force exerted by plow
b. speed of plow 103. What is the effective field capacity
c. All of the above of a rotary tiller tilling theoretically at 0.4
d. None of the above ha/hr with 85% field efficiency?
a. 0.34 ha/hr
98. What is the force required to pull a b. 0.45 ha/hr
single-moldboard plow on a clay-loam soil c. 0.52 ha/hr
having 30cm width and 15cm depth-of- d. None of the above
cut. Soil draft is 8 psi. (Problem 3)
a. 314 kg 104. The effective width-of-cut of a
b. 253 kg single-bottom moldboard plow drawn by
c. 157 kg a power tiller is 10 cm. If the machine
d. None of the above will be used to plow a 2-hectare farm,
what will be the total distance that it will
99. If the plow in Item 98 has 5 km/hr travel to accomplish the task?
average speed, what is its drawbar a. 200 m
horsepower? b. 200 km
a. 4.6 hp c. 200 miles
b. 3.4 hp d. All of the above
c. 5.1 hp
d. None of the above 105. Type of implement that is directly
attached to the tractor and its power lift
100. What is the required drawbar pull for is dependent upon the tractor engine for
a 5m tractor-drawn spike-tooth harrow its general operation.
with 89 kg/m specific draft? a. Semi-integral
a. 345 kg b. Integral-mounted
b. 445 kg c. Trailing or pull
c. 500 kg d. None of the above
d. None of the above
106. Center-of-resistance in this type of 112. Type of implement which is
plow is at the intersection between the supported by two or three wheels and is
share and the moldboard and to the right hitched to the drawbar of the tractor that
of the shin. pulls it.
a. Disk plow a. Integral-mounted
b. Moldboard plow b. Trailing
c. All of the above c. Semi-integral
d. None of the above d. None of the above

107. Plow center-of-resistance is at the 113. Compartment provided for the


left and below the center of the disk and harrow to aid in penetration.
is closer to the furrow wall. a. Weight boxes
a. Disk plow b. Ballast
b. Moldboard plow c. Ballast compartment
c. All of the above d. All of the above
d. None of the above 114. Gang arrangement often called as
108. When a disk plow is set at vertical “tandem harrow”.
a. Double-action
position, it will ____.
a. penetrate b. Single-action
b. not penetrate c. Offset
c. remain in the position d. None of the above
d. None of the above 115. Flanged-tube mounted on a gang
axle and placed between two discs to
109. Gang arrangement in which the
prevent lateral movement of discs along
harrow is not centrally located with the shaft.
respect to the tractor position.
a. Spool
a. Double-action
b. Spacer
b. Single-action
c. Tubular washer
c. Offset
d. None of the above
d. None of the above
116. Gang arrangement on a disk harrow
110. Factor that influence the depth of consisting of two disk gangs placed end-
penetration of the disk harrow. to-end that throws the soil in opposite
a. Angle of disk direction.
b. Angle of hitch a. Double-action
c. Size of disk b. Single-action
d. All of the above c. Offset
d. None of the above
111. When a trailing harrow is hitched
too high on the drawbar of the tractor, it 117. When the disk angle is increased,
will ____. the penetration of the disk plow will
a. have too deep penetration ____.
b. have too shallow penetration a. be improved
c. have right penetration b. be the same
d. None of the above c. become poor
d. None of the above
118. Traverse distance between the top 123. Determine the width-of-cut of six
and bottom edges of the end-disks of a 800mm-diameter disk plows , with
plow . 350mm disk spacing.
a. Implement width a. 2.8 m
b. Width-of-cut b. 2.7 m
c. Disks width c. 2.6 m
d. None of the above d. None of the above
119. A rotary tiller has 2m operating 124. The effective field capacity of 2
width. If the tractor runs at a speed of 2
moldboard plows having 25cm width
km/hr, what is its theoretical field each plowing at 7 km/hr speed with 80%
capacity? field efficiency is ____.
a. 4.0 ha/hr a. 0.20 ha/hr
b. 0.4 ha/hr b. 0.28 ha/hr
c. 0.04 ha/hr
c. 0.32 ha/hr
d. None of the above d. None of the above
120. The effective field capacity of the
rotary tiller in Item 119 tilling 125. The field capacity of a rotary tiller
theoretically at 0.4 ha/hr with 85% was measured at 0.23 ha/hr. If it has a
efficiency is ____. 1.5m operating width and a 2kph speed,
a. 0.34 ha/hr what is its field efficiency?
b. 0.24 ha/hr a. 64%
c. 0.43 ha/hr b. 77%
d. None of the above c. 85%
d. None of the above
121. In a 200-hectare corn project, the
field has to be tilled within 30 days. 126. A 3-bottom plow is plowing at 8kph
What size of a heavy-duty offset-disk speed. The effective width per plow
harrow should be selected if the tractor bottom is 25 cm. If it has 80% plowing
to be used operates at 5 kph? Assume a efficiency, what is its theoretical field
75% field efficiency and an 8-hour capacity?
working day. a. 0.4 ha/hr
a. 1.5 m b. 0.6 ha/hr
b. 2.2 m c. 0.75 ha/hr
c. 2.0 m d. None of the above
d. None of the above
127. The effective field capacity of the
122. What is the theoretical field plow in Item 126 above is ____.
capacity of the tandem disk harrow in a. 0.48 ha/hr
Item 121 pulled by a 1.5m-wide tractor
b. 0.60 ha/hr
running at 10kph speed? c. 0.72 ha/hr
a. 1.5 ha/hr (1.1 ha/hr) d. None of the above
b. 2.5 ha /hr
c. 5.1 ha/hr
d. None of the above
128. What is the cutting width of the 133. A power tiller was tested on a 10m-
single-disk harrow set at 18 degree wide plot. During the test, the machine
working angle containing 20 disks spaced made 22 rounds to complete the plowing
at 30 cm? Disk diameter is 40 cm. using two 0.3meter- diameter disk
(Problem 30) plows. What is the average swath of the
a. 582 cm power tiller?
b. 454 cm a. 0.23 m
c. 541 cm b. 0.32 m
d. None of the above c. 0.41 m
d. None of the above
129. If the harrow in Item 128 above is a
tandem disk harrow, what is its cutting 134. Horizontal distance that is
width? perpendicular to the direction of travel
a. 6.18 m an implement travels in performing its
b. 5.67 m intended function.
c. 5.30 m a. Implement width
d. None of the above b. Operating width
c. Side draft
130. A 30-disk double-offset disk harrow d. None of the above
is travelling at10kph speed. If the disk
diameter is 40 cm spaced at 30 cm, what 135. A self-propelled implement ____.
is the cutting width of the harrow? a. requires a tractor to perform its
a. 860.5 cm function
b. 875.4 cm b. requires a stationary engine in order
c. 850.3 cm to do its function
d. None of the above c. does not require a tractor to operate
d. None of the above
131. What is the theoretical field capacity 136. An agricultural equipment used to
of the harrow in Item 130? carry load without power of its own.
a. 8.6 ha/hr a. Agricultural loader
b. 8.7 ha/hr b. Agricultural trailer
c. 8.5 ha/hr
c. Agricultural carrier
d. None of the above d. None of the above
132. What is the effective field capacity 137. Transport machine used in
of eight 900mm-diameter disk plows agriculture suitably designed for coupling
running at 10kph speed. The disks are to an agricultural tractor or to a self-
spaced at 500 mm. Assume an 80% propelled combine.
plowing efficiency. (Problem 36) a. Agricultural wagon
a. 2.74 ha/hr b. Agricultural loader
b. 3.05 ha/hr c. Agricultural trailer
c. 3.41 ha/hr d. None of the above
d. None of the above
138. Agricultural tractors, self-propelled 143. A part of a mechanism that may be
machines and implements designed for added to a functional power unit for the
field operations are examples of ___. purpose of enhancing the performance of
a. farmstead equipment that mechanism.
b. agricultural field equipment a. Auxillary unit
c. agricultural implement b. Spare parts
d. None of the above c. Accessory
d. None of the above
139. An implement attached to the three-
point hitch and to the hydraulic lift system 144. The actual performance of a
of the tractor. machine on land or the crop processed in
a. Trailing implement a given time based on total field time.
b. Semi-mounted implement a. Theoretical field capacity
c. Mounted implement b. Effective field capacity
d. All of the above c. Field efficiency
d. All of the above
140. The true point-of-hitch or the center-
of-pull of an implement. 145. The draft of an implement in terms
a. Center of power of force per unit area of tilled cross-
b. Center of resistance section.
c. Center of load a. Implement draft
d. All of the above b. Specific draft
c. Line draft
141. The portion of an implement d. All of the above
designed to connect to the power source.
a. Connector 146. The operating width of the
b. Clutch implement minus the overlap is ___.
c. Hitch a. implement width
d. All of the above b. effective width
c. operating width
142. The distance between lower hitch d. All of the above
point measured at the base of the lower
hitch stud, or the distance between the 147. The rate of performance obtained
inner most restraining means provided on when a machine performs its function
100% of the time at a given operating
the implement.
a. Lower hitch-point spread speed using 100% of its theoretical width.
b. Lower hitch point distance a. Theoretical field capacity
c. Lower hitch point clearance b. Effective field capacity
d. None of the above c. Field capacity
d. None of the above
148. Suppose you are the Agricultural 152. Typical diesel engine power
Engineer in-charged of a 500-hectare requirement of power tiller used for
corn farm with soil type that gives 10 psi plowing and harrowing rice field?
draft when plowing at 4 km/hr speed. a. 7 hp
Compute the drawbar horsepower b. 12 hp
required to pull a three-disk plow with c. 16 hp
12 inches effective cut per disk at 6 d. None of the above
inches plowing depth. (Problem 26)
a. 14.3 hp 153. Fuel consumption rate of diesel
b. 23.4 hp engine used by power tiller in plowing and
c. 34.5 hp harrowing operation.
d. None of the above a. 0.85 liter per hour
b. 1.0 liter per hour
149. How many hectares of the corn c. 1.5 liters per hour
farm in Item 148 a tractor can plow in d. None of the above
one day (16-hours operation) if it is 154. Which is not a safety precautionary
pulling a three-disk plow? Assume a 30% measure for power-driven implement?
total time loss. a. Never leave self-lift implements in
a. 2 ha/day raised position.
b. 4 ha/day b. Allow adequate headlands, especially
c. 6 ha/day when using trailed implements near bank,
d. None of the above river and ditches.
c. Ensure that tractor is jacked securely
when altering wheels for row-crop work.
150. How many days would be required d. None of the above
to plow the entire 500-hectare farm in
Item 148 if ten units of the same kind of 155. Which of the following is a safety
tractor and plow will be used? precaution for power-driven implements?
a. 8.3 days a. Do not make adjustments on moving
b. 12.5 days parts while the machine is running.
c. 15.4 days b. Stop the tractor engine before doing
d. None of the above repair or adjustments, even if the PTO is
disengaged.
151. Still referring to Item 148, how c. Do not work under implement held in
much fuel would the tractor consumes, raised position by the tractor hydraulic.
in liters per 16-hours day, if it has 1 d. All of the above
lb/dhp-hr specific fuel consumption and
0.8 specific gravity of the fuel? 156. Which of the following is true?
a. 1546 liters a. Always disengage the PTO drive when
b. 130.0 liters/hr work ceases even if only temporarily.
c. 95.1 liters/hr b. Always attach trailed implements to the
d. None of the above proper drawbar and never to a high
position at the rear of the tractor.
c. Always use proper draw pin with a
locking device.
d. All of the above
157. Find the effective field capacity of a 162. What is the drawbar horsepower of
5-bottom disk plow with 25cm width-of- the implement in Item 161 if it is running
cut per plow bottom. Assume a 0.8 field at 20kph speed.
efficiency and 7kph operating speed. a. 15.4 hp
a. 0.56 ha/hr b. 19.5 hp
b. 0.70 ha/hr c. 21.7 hp
c. 0.83 ha/hr d. None of the above
d. None of the above
163. A power harrow has 6m operating
158. Compute the drawbar pull of a 4- width. If the tractor runs at speed of 20
bottom moldboard plow on a clay loam km/hr, what is the theoretical field
soil having 0.3m effective field width-of- capacity of the machine?
cut per bottom and 0.15m depth-of-cut. a. 12.0 ha/hr
The plowing speed is 10 kph. The soil is b. 14.7 ha/hr
a clay loam with specific draft of 8 psi. c. 16.5 ha/hr
a. 1001 kg d. None of the above
b. 1009 kg 164. What is the effective field capacity
c. 1014 kg of the harrow in Item 163 if its field
d. None of the above efficiency is 0.85?
a. 10.2 ha/hr
159. What is the drawbar pull of the b. 12.1 ha/hr
plow in Item 158? c. 14.3 ha/hr
a. 36.9 hp d. None of the above
b. 40.1 hp
c. 45.7 hp 165. The field capacity of a tractor-drawn
d. None of the above field cultivator was measured at 2.9
ha/hr. If the operating width and speed
160. A 3-bottom plow is plowing at 8kph of the machine is 2.5 m and 13 kph,
speed. The effective width per plow respectively, what is the field efficiency
bottom is 25 cm. If the plowing of the tiller?
efficiency is 80%, what are the effective a. 78%
field capacity of the plow? b. 89%
a. 0.48 ha/hr c. 98%
b. 0.56 ha/hr d. None of the above
c. 0.60 ha/hr
d. None of the above 166. A 16hp rotary tiller is tilling at 10cm
depth. Its rotating tiller has 0.5m width.
161. What is the drawbar pull required What is the specific power output of the
for a 3m tractor-drawn offset-disk machine if it is traveling at 0.5mps
harrow? The implement specific draft is speed? Assume a 0.80 power
89 kg/m. transmission efficiency for the tiller.
a. 243 kg a. 0.382 watts/cm3
b. 267kg b. 0.453 watts/cm3
c. 286 kg c. 0.531 watts/cm3
d. None of the above d. None of the above
167. A 16hp rotary tiller is tilling at 10cm 171. The effective width-of-cut of a
depth. Its rotating tiller has 0.5m width. single-bottom moldboard plow drawn by
What is the specific power output of the a power tiller is 10 cm. If the machine is
machine if it is traveling at 0.5mps used to plow a 2-hectare farm, what
speed? Assume a 0.80 power would be the total distance to be
transmission efficiency for the tiller. traveled by the machine to accomplish
a. 0.382 watts/cm3 the task?
b. 0.453 watts/cm3 a. 100 km
c. 0.531 watts/cm3 b. 150 km
d. None of the above c. 200 km
d. None of the above
168. A 16hp rotary tiller is tilling at 10cm
depth. Its rotating tiller has 1.2m width. 172. What is the required drawbar pull of
What is the specific power output of the a 5m tractor-drawn spike-tooth harrow
machine if it is traveling at 0.5mps with 89 kg/m specific draft?
speed? Assume a 90% power a. 445 kg
transmission efficiency for the tiller. b. 478 kg
a. 0.178 watts/cm3 c. 493 kg
b. 0.190 watts/cm3 d. None of the above
c. 0.218 watts/cm3
d. None of the above 173. What is the drawbar horsepower of
the implement in Item 172 if it is running
169. Compute the drawbar pull of a plow at 6kph speed?
on a power tiller with 10cm x 8cm depth- a. 10 hp
and width-of-cut. The number of plow b. 12 hp
bottom is 2. Soil draft is equal to 5 psi. c. 14 hp
a. 45.3 kg d. None of the above
b. 50.7 kg
c. 56.36 kg 174. A single-bottom moldboard is
d. None of the above plowing on a clay loam soil (8 psi draft)
having 0.3m effective width-of-cut and
170. If the plow in Item 169 is drawn at 15cm depth-of-cut. What is the force
10kph speed, what is the drawbar exerted by the plow?
horsepower of the plow? a. 253 kg
a. 2.04 hp b. 278 kg
b. 4.57 hp c. 296 kg
c. 5.23 hp d. None of the above
d. None of the above
175. What is the drawbar horsepower of 179. Five hundred hectares of farm is to
the plow in Item 174 if its plowing speed be plowed using a 5-bottom moldboard
is 5 kph? plow. The cutting width of each plow
a. 4.6 hp bottom is 20 cm with 15cm cutting depth.
b. 4.9 hp If three (3) tractors are pulling a plow at
c. 5.2 hp an average speed of 15 kph, how many
d. None of the above days will it take to finish the whole area?
Assume an 85% plowing efficiency and a
176. At 18-degrees working angle, a 6hour-per-day plowing operation.
tandem disk harrow is used to pulverize a. 21.78 days
a newly plowed farm at 10kph speed. b. 35.12 days
The harrow has 24 pieces and 30cm disk c. 40.25 days
spaced at 20 cm. Compute the effective d. None of the above
width of the implement.
a. 473 cm 180. Compute the drawbar hp of the plow
b. 498 cm in Item 179 if the specific draft of soil is
c. 502 cm equal to 12 psi.
d. None of the above a. 69 hp
b. 74 hp
177. What is the theoretical field c. 81 hp
capacity of the implement in Item 176? d. None of the above
a. 4.21 ha/hr
b. 4.73 ha/hr
c. 4.91 ha/hr
d. None of the above

178. Referring to Items 176 & 177, what


is the effective field capacity of the
harrow assuming a 90% field efficiency.
a. 4.26 ha/hr
b. 4.91 ha/hr
c. 5.24 ha/hr
d. None of the above
IX. SEEDING AND PLANTING
EQUIPMENT

Seeding and Planting Equipment - used


to place the seeds or plant parts into the
soil and keep them from weeds from
insect pests and diseases until they are
fully grown and ready for harvesting.

Classifications

• Seeders - machines used to place


seeds, such as corn, rice, etc., in the
soil.
• Planters - machines used to place plant
parts, such as sugar cane cuttings, rice
seedlings, etc., in a prepared soil.

Methods

• Broadcasting - randomly scattering of


seeds on the surface of the field.
• Drill Seeding - randomly dropping and
covering of seeds in furrow.
• Precision Planting - accurate placing of
single seed/plant at about equal
interval in rows.
• Hill Dropping - placing of group of
seeds at about equal interval in rows.
• No-Till Planting - procedure whereby
planting is made directly into an
unprepared seedbed or untilled soil.
• Minimum Tillage - minimum soil
manipulation necessary for crop
production or for meeting tillage
requirement under existing soil
condition.
• Seed Tape Planting - planting system
wherein the seeds are deposited either
singly or in group on a water-soluble
tape (polyethylene oxide) under
controlled condition.
Rice Transplanters - machines that Air Seeders - machines which
place rice seedling at a proper spacing pneumatically place the seeds into the soil
in a well prepared paddy soil by means of a deep tillage tool.

Rice Drum Seeders - machines that are


designed for planting pre-germinated
rice seed in rows so that manually-
operated mechanical weeder can be
used.
Functions of Seeder/Planter Classifications of Seeder According to
Methods of Seeding
• To meter seeds/plant parts of different
sizes and shapes.
• To place the seeds/plant parts in the 1 Drills - machines that prepare the soil,
meter the seed, and position the seed
acceptable pattern or distribution in
in one operation.
the field.
• To place seeds/plant parts accurately 2 Field Distributors - machines that
and uniformly at the desired depth in consist of a seed box with metering
device at the bottom of the hopper, and
the soil; and drop the seed in a tube for placement
• To cover the seeds/plant parts into the furrow, followed by covering
optionally and to compact the soil and pressing of a disc and a press
around it to enhance germination and wheel. Sometimes it is provided with
emergence. mechanism to also distribute fertilizer
together with the seeds.
3 Broadcasters - machine that meter the
materials onto a revolving flange wheel
that has the highest work capacity but
is hardest to calibrate because of
uneven distribution from the flange
wheel, unequal seed weight and shape,
and difficulty to work with the wind to
obtain uniform distribution of seeds,
and uneven soil surface.

Classifications of Seeder According to


Source of Power

• Manually Operated - Machine that is


pushed or pulled by human.
• Animal Drawn - Power source is by the
use of draft animal harnessed either
singly or in multiple animals.
• Power-Tiller Operated - Seeding or
planting machine that is drawn by a
small walking-type tractor powered by
a separate engine. The machine
sometimes obtain power from the
engine through pulley-and- belt drive.
• Tractor Type - Seeding or planting
machine that is either mounted or pull-
type unit. Power source for the
machine is transmitted through the
power take-off drive of the tractor.
Basic Components of a Seeder

• Seed Box - holds and contains the seeds


in the seeder.
• Metering Device - meters the seeds or
controls the rate of delivery either in
bulk or as single seed into the seed tube.
• Seed Tube - conveys the seeds from the
metering device to the rear of a furrow
openers and drop them into the soil.
• Furrow Opener - soil-engaging device
that opens a furrow at a desired depth
for placement of the seeds and partially
covers them with soil.
• Covering Device - moves the nearby
surface soil into the furrow and then
covers the seeds.
• Press Wheel - compresses the soil
around the seed in order to create good
contact between the seeds and the soil
aggregates for improved germination.
• Ground Wheel - drives the metering
device at a proper rotational speed in
order to attain the required plant
spacing.
• Main Frame - holds and supports the
various components of the seeder and
provides the hitch attachment to the
tractor or power tiller.
Factors Affecting the Performance of a Plant Spacing and Row Spacing of Various
Seeder/Planter Crops
Crops Plant Spacing Row Spacing
• Seed-Related Factors (cm) (cm)
• Quality of seed Groundnut 5 – 10 22 – 60
• Method of seed treatment
Soybean 4–7 20 – 60
• Soil-Related Factors
Maize 20 – 25 45 – 60
• Soil bulk density
Cotton 20 – 40 50 – 80
• Soil porosity
Peas 5 – 15 45 – 60
• Soil water
• Soil temperature Rice 15 – 20 20 – 30

• Specific volume Sorghum 10 – 15 30 – 45

• Soil cohesion Sunflower 20 45 – 80


• Soil air Wheat 3-5 15 – 22
• Soil strength
• Mechanical Factors
• Seed damage during metering
• Uniformity of depth of seed
placement
• Uniformity of distribution of
seeds along rows
• Transverse displacement of
seeds from the row
• Prevention of loose soil getting
under the seeds
• Degree of soil compaction level
above the seeds
• Uniformity of soil cover over the
seeds
• Changes of mixing fertilizer with
seed during placement on the
furrow.
USEFUL FORMULA

No. of Hills Planted Distance per Hill


A 10,000
Hn = --------------------- Dph = Sr π Dg / Nc
Sr Sh where:
where: Dph - distance per hill, mm
Hn - number of hills Sr - speed ratio of ground wheel and
A - area planted, ha seed plate
Sr - row spacing, m Dg - diameter of the ground wheel, mm
Sh - hill spacing, m Nc - number of cells in the seed plate

Nominal Working Width Total Weight of Seeds


Nh Nsh Sw
W = n d TW s = --------------------------
where: 1000 E
W - working width, m where:
n - number of rows TW s - total weight of seeds needed, kg
d - row spacing, m Nh - number of hills
Nsh - number of seeds per hill
Sw - specific weight of seeds, g/seeds
E - emergence, dec.
Effective Diameter of Ground Wheel Speed Ratio of Ground Wheel and
d Metering Device
De = ---------- Nc Hs
π N R = --------------
where: Cgw
De - effective diameter of ground wheel where:
under load, m R - speed ratio
d - distance for a given N, m Nc - number of cells
N - number of revolution, rpm Hs - hill spacing, m
Cgw - circumference of ground wheel, m
Delivery Rate Delivery Rate (PTO-Driven Machine)
L 10,000 L 10,000
Q = ---------------------- Q = ----------------------
π De N W vt W
where: where:
Q - delivery rate, kg/ha Q - delivery rate, kg/ha
L - delivery for a given N, kg L - delivery for a given N, kg
De - effective diameter of ground wheel v - tractor speed, m/s
under load, m t - time for measuring delivery, s
N - number of revolution, rpm W - working width, m
W - working width, m
Effective Field Capacity Theoretical Field Capacity

EFC = A / t TFC = 0.36 w v


where: where:
EFC - effective field capacity, m2 /h TFC - theoretical field capacity, m2 /hr
A - area covered, m2 w - working width, m
t - time used during operation, hr v - speed of operation, m/s
Field Efficiency Fuel Consumption Rate

ξf = (EFC / TFC) 100 FC = V / t


where: where:
ξf ξf - field efficiency, % FC - fuel consumption, lph
EFC - effective field capacity, m2 /hr V - volume of fuel consumed, l
TFC - theoretical field capacity, m2 /hr t - total operating time, hr
Wheel Slip
No - Nl
W s = ------------------ x 100
No
where:
W s - wheel slip, %
No - sum of the revolutions of the driving
wheel without load, rev
Nl - sum of the revolutions of all driving
wheel with load, rev
SAMPLE PROBLEMS

Problem 1

A corn plantation requires a population of 37,000 plants per hectare. The average
emergence is 90% and the row spacing is 1.2 meters. If the seeds are to be drilled,
what should be the seed spacing?

Given:
Required population - 37,000 plants per ha
Emergence - 90%
Row spacing - 1.2 meters

Required: Seed Spacing

Solution:

No. of hills = No of Hills / Emergence


= 37,000 plants/ha / 0.90
= 41,111 plants /ha

Seed spacing = 41,111 plants per ha / [10,000 m2 /ha x 1.2 m]


= 0.30 m

Problem 2

Compute the number of plants required for a 2-hectare corn farm with the following
seed spacing: Plant spacing, 0.25 m and row spacing, 0.45 m.

Given:
Area - 2 ha
Sp - 0.25 m
Sr - 0.45

Required: Number of plants for the


2-hectare farm

Solution:

Np = (2 hectare x 10,000)/(0.25 m x 0.45 m)


= 177, 777.8 plants or hills
Problem 3

A 120-hectare farm is to be planted with corn. The row and hill spacings are 50 cm
and 40 cm, respectively. How many hills are to be planted in the farm? If a 12-row
drill planter is to be used at 10kph implement speed, what is the theoretical and
effective field capacities of the machine? Assume an 80% field efficiency. If the field
requires 3 seeds per hill and has a percentage emergence of 90%, how many kilos of
seeds are needed for the entire area? Assume a 0.1 g weight per seed.

Given:
Area - 120 has.
Row spacing - 50 cm
Hill spacing - 40 cm
Machine - 12- row seeder/planter
Implement Speed - 10 kph
Field efficiency - 80%
% Emergence - 90%
No. of Seeds/hill - 3 seed
Wt. per seed - 0.1 g / seed

Required:
Number of Hills
Theoretical Field Capacity
Effective Field Capacity
Weight of Seeds needed for the entire farm

Solution:

No. Hills = (10,000 x A) / (Sh x Sr)


= 10,000 x 120 ha / (0.50 m x 0.4 m)
= 6,000,000 hills

TFC = 0.1 Wi Vi
= 0.1 x 12-row x 0.5 m x 10 kph
= 6 ha/hr

EFC = TFC ξf
= 6 ha/hr x 0.80
= 4.8 ha/hr

Ws = No Hills x No. Seeds x SWs / E


= (6,000,000/hill x 3 seeds/hill x 0.1 g/seed x kg/1000 g) / 0.9
= 2000 kg
Problem 4

A 2.4m sugar planter is used to transplant sugarcane to a 150-hectare farm. The


plant spacing is 60 cm and the row spacing is 120 cm. Compute the number of hills
the machine can plant for the entire area. What is the rate of planting in hills per
hectare if the machine travels at 6kph speed? Estimate the number of units of
tractors and of planters required to finish the entire area in 10 days at 8 hours/day
operation. Assume an 80% planting efficiency.

Given:
Area - 150 ha
Plant spacing - 60 cm
Row spacing - 120 cm
Speed - 6 kph
Oper time required - 10 day (8hr/day)
Field efficiency - 80 %

Required:
No. of Hills to be planted
Rate of Planting, in hills/ha
No. of Tractor/planter unit

Solution:

150 ha x 10, 000


No. of hills =
06 m x 1.2 m

= 2, 083, 333.3 hills

Rate of planting = 0.1 x 6 km/hr x 1.2 m x 0.80


= 0.576 ha/hr

Tractor Field Capacity = 2.4 m x 6 kph x 0.80


= 1.15 ha/hr-tractor

No. of Tractors = 150 ha / (1.15 ha/hr-tractor x 8 hr/day x 10 days)


= 1.63 tractors (use 2 tractors)
Problem 5

How many grams of cotton seeds are required per row for a hectare of farm,
assuming a seeder is to be used in planting the seeds at 0.20m row spacing and
0.50m plant spacing? The seeding rate is 10 kg/ha.

Given:
Crop - cotton
Seeding rate - 10 kg/ha
Row spacing - 0.5 m
Plant spacing - 0.20

Required: Seed Required, in kilogram

Solution:

Seed Required = 10 kg/ha x 0.20 m x


0.5 m /10
= 0.1 g

Problem 6

Compute the number of hills to be planted with corn seeds on a 100-hectare farm.
The seed spacing is 50 cm and the row spacing is 40 cm.

Given:
Seeds - corn
Area - 100 has.
Hill spacing - 50 cm
Row Spacing - 40 cm

Required:
Number of Hills to be planted

Solution:
Nh = 100 ha (10000) / [0.50 m x 0.40m]
= 5,000,000 hills
Problem 7

A 10-hectare farm will be planted with soybean at hill and row spacing of 20 cm x
30 cm. If the seed has 80% emergence, how many plants are expected to emerge
during the cropping season.

Given:
Area - 10 has.
Hill spacing - 20 cm
Row spacing - 30 cm
% emergence - 80

Required: Number of plants to emerge

Solution:

Nh = 10000 x 10 ha / [0.2 m x 0.3 m]


= 1,666,666.67 hills

Np = 1,666,666.67 hills x 0.80


= 1,333,333.33 plants

Problem 8

A 5-hectare farm is to be planted with peanut at 250 kg per hectare. The hill and
row spacings are 30 cm x 50 cm, respectively. How many kilos of peanuts is to be
planted per hill?

Given:
Area - 5 has.
Seeding rate - 250 kg/ha
Hill spacing - 30 cm
Row spacing - 50 cm

Required: Weight of peanut to be


planted per hill

Solution:

Ws = [250 kg/ha x 0.3 m x 0.5 m ] /10,000


= 0.00225 kg or 2.25 grams
Problem 9

A 4m sugar planter is used to transplant sugar cane to a 150-hectare farm. The


plant spacing is 60 cm while the row spacing is 120 cm. Compute the number of
hills the machine can plant for the entire area. What is the rate of planting, in hills
per hectare, if the machine travels at a speed of 6 kph? Estimate the number of
tractors and planters required to finish the entire area in 10 days at 8 hours/day
operation. Assume an 80% planting efficiency.

Given:
Area - 250 ha
Plant spacing - 60 cm
Row spacing - 120 cm
Speed - 8 kph
Operating time required - 10 day
(8hr/day)
Field efficiency - 90 %

Required: No. of Hills to be planted


Rate of Planting, in hills/ha
No. of tractors/planter unit

Solution:

250 ha x 10, 000


No. of hills = ----------------------------
0.6 m x 1.2 m

= 3,472,222.2 hills

Rate of planting = 0.1 x 8 km/hr x 1.2 m x 0.90


= 0.864 ha/hr

No. of Tractors = 250 ha / (0.864 ha/hr x 8 hrs/day x 10 days)


= 3.61 or 4 tractor and planter units
Problem 10

A power tiller draws a two-row seeder at 30cm spacing. The seeder has a 6-pocket
metering device and is driven by a 0.4m ground wheel. If the required hill spacing is
0.3 m, what is the recommended sprocket ratio for the drive? What is the theoretical
field capacity of the implement if it runs at 4 kph? What is the effective field capacity
of the implement if its field efficiency is 90%? If the seeds per hole is required at 0.8
g/seed, how many kilos of seeds are needed to finish the 10 hectares? Assume a
90% germination percentage. How many hills will be planted for the 10-hectare
farm? How many days will it take to finish the operation? If the speed ratio to be
used is 1.5, what is the distance per hill?
Given:
No. of rows - 2
Spacing - 0.3 m
Metering device - 6 pocket
Hill spacing - 0. 3 m
Ground wheel - 0.4 m
Velocity - 4 kph
Efficiency - 90%
Weight of seed per hole - 0.8 g/seed
Germination percentage - 90%

Required: Sprocket Ratio


Theoretical Field Capacity
Effective Field Capacity
Weight of Seeds needed for 10 hectares
No. of Days to finish seeding
Distance per Hill at 1.5 speed ratio

Solution:
C ground wheel = π (0.4 m) = 1.25 m
α metering device = 360 deg/ 6 pockets = 60 deg per pocket
α ground wheel @ 0.3 m = 360 deg x 0.3 m/1.25 m = 86.4 deg @ 0.3 m travel
Ratio = 86.4 deg / 60 deg = 1.44
TFC = 0.1 x 0.3 m x 2 x 4 kph = 0.24 ha per hour
EFC = 0.24 ha per hour x 0.90 = 0.22 ha per hour
Weight of seeds in 10 ha 2
= 10 ha x 10,000 m /ha x 0.8g /hole /
[(0.3 m x 2 x 0.3 m) x 0.9]
= 493,827.2 g or 493.8 kg
No. of days = 10 ha / (0.22 ha/hr x 8 hr/day)
= 5.6 days
Distance per hill at 1.5 SR = 1.5 x 60 deg x1.25 / 360
= 0.3125 m
No of hills = 10 ha (10,000 m2/ha) / (0.30 m x 0.31 m)
= 1,075,268.8 hills
Problem 11

A five-row seeder will be used in seeding a 100-hectare farm at 5 seeds per hill. The
row spacing of the seeder is 30 cm while the hill spacing is 25 cm. Compute the number
of hills to be planted for the entire area? If the seeder travels at 15 kph, what is the
theoretical field capacity? What is the effective field capacity if it has a field efficiency of
90%. How long will it take for the seeder to finish the job if it will be operated at 8
hours per day and 20 days per month? If the seeds weigh 0.1 g/seed and requires 5
seeds per hill, what is the total weight of the seeds needed? Percent germinability of
the seeds is 98%.

Given:
No. of rows -5
Area to be planted with seeds - 100 ha
No of seeds per hill -5
Row spacing - 30 cm
Hill spacing - 25 cm
Velocity of travel - 15kph
Field efficiency - 90%
No. of seeds per hill - 5 seeds
Weight of seeds - 0.1 g/see
Germinability - 98%
Operating time per day - 8 hours per day

Required : No of hills to be planted for the entire area

Solution:

Nh = 100 ha x (10,000) / (0.30 x 0.25)


= 13,333,333.33

Theoretical field capacity = 0.1 [5 row x 0.3 m x 15 kph]


= 2.25 ha/hr

Field Capacity = 2.25 ha/hr x 0.90


= 2.025 ha/hr

No. of days = 100 ha /2.025 ha/hr x 8 hr/day x 20 days per month


= 16.17 days

Weight of seeds = [13,333,333.33 hills x 5 seeds/hill x 0.1 g/seed] / 0.98


= 6.8 tons
REFERENCES

ASAE. ASAE Standard 1997.


Campbell, J.K. 1990. Dibble Sticks,
Donkeys, and Diesel: Machine in Crop
Production. International Rice Research
Institute. Los Banos, Laguna, Philippines.
329pp
RNAM. 1991. Agricultural Machinery
Design and Data Handbook (Seeder and
Planter). RNAM/ESCAP. United nations
Building. Rajadarmnem Ave., Bangkok
10200 Thailand. 137pp.
QUESTIONS and PROBLEMS

1. Equipment used to place the seeds or 7. Method in which seeding is done


plant parts into the soil and keep them directly into previously undisturbed soil.
from weeds, insect pests and diseases until a. Much tillage
fully grown. b. No-till
a. Tillage equipment c. Minimum tillage
b. Seeding and planting equipment d. None of the above
c. All of the above
d. None of the above 8. Method in which planting is done
directly onto an essentially unprepared
2. Method of randomly scattering seeds seedbed.
on the surface of the field. a. Zero tillage
a. Broadcasting b. No-tillage planting
b. Drill seeding c. Reduced tillage
c. Seed tape planting d. None of the above
d. None of the above
9. Equipment used to place seeds in
3. Method of randomly dropping and furrow.
covering seeds in furrow. a. Planter
a. Broadcasting b. Seeder
b. Drill seeding c. Broadcaster
c. Precision planting d. All of the above
d. None of the above
10. Equipment used to place plant parts
4. Accurate placing of seeds singly at equal
to a prepared furrow.
interval in rows.
a. Planter
a. Broadcasting
b. Seeder
b. Drill seeding
c. Broadcaster
c. Precision planting
d. All of the above
d. None of the above

5. Placing of group of seeds at equal 11. Which of the following is a


interval in rows. classification of seeder according to
a. Hill dropping method of seeding?
b. No-till planting a. Drill
c. Seed tape planting b. Field Distributor
d. None of the above c. Broadcaster
d. All of the above
6. Planting method in which seeds are
deposited, either singly or in group, in a 12. Which of the following is a
water soluble tape under controlled classification of seeder according to
condition.
source of power?
a. Precision planting a. Manually operated
b. Seed-tape planting b. Animal drawn
c. Hill dropping c. Power-tiller operated
d. None of the above d. All of the above
13. Which of the following is the function 18. Which of the following is a reason
of a seeder/planter? why the broadcaster is hard to calibrate?
a. To meter seeds/plant parts of different a. Uneven distribution from the flange
sizes and shapes. wheel
b. To place the seeds/plant parts b. Unequal seed weight
c. Uneven soil surface
accurately and uniformly at the desired
d. All of the above
depth and spacing in the soil.
c. To cover the seeds/plant parts 19. Minimum area required for testing
optionally and to compact the soil around manually-operated seeder.
it to enhance germination and a. Not less than 100 m2
emergence. b. Not less than 500 m2
d. All of the above c Not less than 1000 m2
d. None of the above
14. Machine used to prepare the soil,
meter the seed and position them in one 20. Machine with power source from
operation. draft animals harnessed either singly or
a. Broadcaster with multiple animals.
b. Drill a. Tractor type
c. Field distributor b. Animal drawn
d. None of the above c. Power-tiller operated
d. None of the above
15. Machine consisting of a seed box with 21. Seeding or planting machine drawn
metering device in the bottom of the by a small walking-type tractor powered
hopper used to prepare the soil for the by a separate engine.
seed. a. Manually operated
a. Broadcaster b. Animal drawn
b. Drill c. Power-tiller operated
c. Field distributor d. None of the above
d. None of the above
22. Machine that sometimes obtain
power from the engine using pulley-and-
16. Machine that is pushed or pulled by belt drive.
human. a. Tractor type
a. Manually operated b. Animal drawn
b. Animal drawn c. Power-tiller operated
c. Power-tiller operated d. None of the above
d. None of the above
23. Power source for this machine is
17. Machine that meter the materials
transmitted through the power-take-off
onto a revolving flange wheel, less
drive of a tractor.
expensive and has the highest work
a. Manually operated
capacity but is hardest to calibrate.
b. Animal drawn
a. Broadcaster c. Tractor type
b. Drill d. None of the above
c. Field distributor
d. None of the above
24. Seeding or planting machine that is 30. Part of the seeder used to convey
either mounted or pulled-type unit. seeds from the metering device to the
a. Tractor type rear of furrow openers and drop them
b. Animal drawn into the soil.
c. Power-tiller operated a. Seed box
d. None of the above b. Metering device
25. Soil engaging device used to open a c. Seed tube
furrow at a desired depth for placement d. None of the above
of seeds and partially cover them with
soil. 31. Part of a seeder that holds and
a. Covering device contains the seed.
b. Furrow opener a. Seed box
c. Press wheel b. Metering device
d. None of the above c. Seed tube
26. Device that transfers seeds from the d. None of the above
seed box to the coulter tube at a fixed 32. Part of a seeder used to move the
rate. nearby surface soil into the furrow and
a. Seed feeder then cover the seeds.
b. Seed distributor a. Covering device
c. Seed metering device b. Furrow opener
d. All of the above c. Press wheel
d. None of the above
27. Part of a seeder that is used to meter
the seeds or to control the rate of 33. Part of a seeder that holds and
delivery, either in bulk or as single seed, supports the various components of the
into the seed tube. seeder likewise provides the hitch
a. Metering device attachment to the tractor or to the
b. Drive wheel power tiller.
c. Seed hopper a. Press wheel
d. None of the above b. Main frame
c. Ground wheel
28. Part of a seeder used to compress the d. None of the above
soil around the seed in order to create
good contact between the seed and soil 34. Minimum efficiency required for a
aggregates for improved germination. tractor-power-driven seed drill.
a. Press wheel a. 55%
b. Main frame b. 65%
c. Ground wheel c. 75%
d. None of the above
d. None of the above
29. Part of a seeder used to drive the
metering device at a proper rotational 35. Minimum efficiency required for a
speed in order to attain the required plant tractor-power-driven row-crop planter.
spacing. a. 40-50%
a. Press wheel b. 55-60%
b. Main frame c. 65-70%
c. Ground wheel d. None of the above
d. None of the above
36. Which of the following factors 41. Seed metering device with axial or
affecting seed germination and helical rod that rotates in a housing filled
emergence? with seeds and causes seeds to flow out
a. Seed damage during metering from the housing in a continuous
b. Uniformity of depth and placement of manner.
seeds a. Adjustable orifice
c. Degree of soil compaction above the b. Fluted roller
seeds c. Plate with cell
d. Uniformity of distribution of seeds d. Cup feed
along rows e. None of the above
e. All of the above
42. Metering device made of a
37. Type of metering device, wherein horizontal, vertical or inclined rotating
seed flow is regulated by changing the cylindrical metal sheet with indention to
size of opening provided at the bottom of pick up and drop the seed.
the hopper and of the agitator, used to a. Adjustable orifice
allow continuous flow of seed. b. Fluted roller
a. Adjustable orifice c. Plate with cell
b. Fluted roller d. Cup feed
c. Plate with cell e. None of the above
d. Cup feed
e. None of the above 43. Seed plate with cell metering device
is recommended for ____.
38. Metering device with spoons that
a. small seeds (e.g., mustard)
picks up seed from a feed hopper and
b. medium seeds (e.g., soybean & rice)
drops it into the seed funnel of the
c. bold seeds (e.g., ground nut)
seeder.
d. All of the above
a. Adjustable orifice
b. Fluted roller 44. Which of the following affects the
c. Plate with cell performance of a seeder/planter?
d. Cup feed a. Seed-related factors
e. None of the above b. Soil-related factors
c. Mechanical factors
39. Cup-feed-type seed metering device
d. All of the above
is recommended for ____.
a. small seeds
45. Which of the following is a seed-
b. medium seeds
related factor affecting the performance
c. bold seeds
of a seeder/planter?
d. None of the above
a. Quality of seed
40. Fluted-roller seed metering device is b. Method of seed treatment
recommended for ____. c. All of the above
a. small seeds d. None of the above
b. medium seeds
c. bold seeds
d. None of the above
46. Which of the following is a soil- 51. Type of a row-crop planter which is
related factor affecting the performance designed to deposit one or more seeds in
of a seeder/planter? a hill at equal interval (Dh ≠ Dr).
a. Soil bulk density and porosity a. Drill planter
b. Soil water and air b. Hill-drop planter
c. Soil temperature and strength c. Check-row planter
d. All of the above d. None of the above

47. Which of the following is a 52. Manual transplanting of rice is


mechanical factor affecting the advantageous over mechanical rice
performance of a seeder/planter? transplanter in terms of ____.
a. Uniformity of distribution of seed a. intelligence
along rows b. dexterity
b. Degree of soil compaction level above c. cost
the seeds d. None of the above
c. Changes of mixing fertilizer with seed
during placement on the furrow 53. In the Philippines, rice is usually
d. All of the above transplanted ____.
a. for weed control reason as the
48. Rice transplanter mechanism that seedlings have a head start over the
place the seedling properly into the soil. weeds
a. Planting arm b. to give employment to landless
b. Picker farm workers
c. Injector c. to prevent rats and birds from
d. None of the above eating the seeds
d. None of the above
49. Type of a row-crop planter designed
to continuously deposit seeds in a 54. Rate of planting or pounds of seeds
straight column. planted per acre that gives a desired
a. Drill planter quality of plants without thinning.
b. Hill-drop planter a. Seeding rate
c. Check-row planter b. Planting to stand
d. None of the above c. Planting capacity
d. None of the above
50. Type of a row-crop planter that
enables the operator to perform hill 55. Which of the crops requires 20-40 cm
planting at definite spacing and & 50-80 cm plant spacing and row
facilitates mechanical weed control and spacing, respectively?
other operations (Dh = Dr). a. Groundnut
a. Drill planter b. Sunflower
b. Hill-drop planter c. Cotton
c. Check-row planter d. None of the above
d. None of the above
56. What are the plant spacing and the 61. If the seeder in Item 60 travels at 15
row spacing required for rice? kph, what is its theoretical field capacity?
a. 20-25 cm & 45-60 cm a. 2.25 ha/hr
b. 15-20 cm & 20-30 cm b. 2.15 ha/hr
c. 10-15 cm & 30-45 cm c. 2.01 ha/hr
d. None of the above d. None of the above

57. What are the plant spacing and row 62. What is the effective field capacity of
spacing required for soybean? the seeder in Item 60 if it has a 90% field
a. 4-7 cm & 20-60 cm efficiency?
b. 5-10 cm & 22-60 cm a. 2.025 ha/hr
c. 3-5 cm & 15-22 cm b. 1.983 ha/hr
d. None of the above c. 1.869 ha/hr
d. None of the above
58. A corn plantation requires a
population of 37,000 plants per hectare. 63. How long will it take for the seeder in
The average emergence is 90% and the Item 60 to finish the job if it will be
row spacing is 1.2 meters. If the seeds operated at 8 hours per day and 20 days
are to be drilled, what should be their per month?
spacing? a. 2 days
a. 0.45 m b. 4 days
b. 0.35 m (0.30 m) c. 6 days
c. 0.25 m d. None of the above
d. None of the above
64. If the seeds weigh 0.1 g/seed and
59. Determine the number of hills in a 2- requires 5 seeds per hill, what is the total
hectare farm planted with corn having weight of the seeds needed? Percent
25cm x 45cm spacing. germinability of the seeds is 98%.
a. 150,000 hills a. 6.8 tons
b. 177,777 hills b. 6.2 tons
c. 185, 877 hills c. 7.9 tons
d. None of the above d. None of the above

60. A five-row seeder will be used in 65. A power tiller draws a two-row
seeding a 100-hectare farm at 5 seeds seeder at 30cm spacing. The seeder has
per hill. The row spacing of the seeder is a 6-pocket metering device and is driven
30 cm while the hill spacing is 25 cm. by a 0.4m ground wheel. If the required
Compute the number of hills to be hill spacing is 0.3 m, what is the
planted for the entire area? recommended sprocket ratio for the
a. 12,888,666 hills drive?
b. 13,333,333 hills a. 1.43
c. 14,121,122 hills b. 1.56
d. None of the above c. 2.01
d. None of the above
66. If the speed ratio to be used in Item 71. Referring to Items 69 & 70, how many
65 is 1.5, what is the distance per hill? days will the operation be finished?
a. 0.312 m a. 4.9 days
b. 0.314 m b. 5.7 days
c. 0.324 m c. 6.1 days
d. None of the above d. None of the above

67. What is the theoretical field capacity 72. Typical number of rows of walking-
of the implement in Item 65 if it will run type rice transplanter.
at 4 kph? a. 4 rows
a. 0.21 ha/hr b. 6 rows
b. 0.24 ha/hr c. 8 rows
c. 0.26 ha/hr d. None of the above
d. None of the above
73. Typical number of rows of riding-type
68. What is the effective field capacity of rice transplanter.
the implement in Item 65 if its field a. 4 rows
efficiency is 90%? b. 6 rows
a. 0.201 ha/hr c. 8 rows
b. 0.209 ha/hr d. None of the above
c. 0.216 ha/hr
d. None of the above 74. A 120-hectare farm is to be planted
with corn. The row spacing and hill
69. If 8 g/seed is required per hole, how spacing is 50 cm and 40 cm, respectively.
many kilos of seeds are needed to finish How many hills are to be planted in the
10 hectares? Assume a 90% farm?
germinability. a. 5,758,000 hills
a. 486 kg b. 6,000,000 hills
b. 498 kg c. 6,897,342 hill
c. 506 kg d. None of the above
d. None of the above
75. If a 12-row drill planter is to be used
70. How many hills will be planted for at 10kph implement speed, what is the
the 10-hectare farm in Item 69? effective field capacities of the machine
a. 999,999 hills assuming an 80% field efficiency.
b. 1,111,111 hills a. 4.8 ha/hr
c. 2,111,111 hills b. 5.2 ha/hr
d. None of the above c. 6.3 ha/hr
d. None of the above
75. If the field requires 3 seeds per hill 79. How many grams of cotton seeds are
and has a percentage emergence of required per row for a hectare of farm,
90%, how many kilos of seeds are assuming a seeder is to be used in
needed for the entire area? Assume a planting the seeds at 0.20m row spacing
0.1 g weight per seed. and 0.50m plant spacing? The seeding
a. 2,000 kg rate is 10 kg/ha.
b. 3,000 kg a. 0.07 g
c. 4,000 kg b. 0.10 g
d. None of the above c. 0.15 g
d. None of the above
76. A 2.4m sugar planter is used to
transplant sugarcane to a 150-hectare 80. Compute the number of hills to be
farm. The plant spacing is 60 cm and the planted with corn seeds on a 100-
row spacing is 120 cm. Compute the hectare farm. The seed spacing is 50 cm
number of hills the machine can plant and the row spacing is 40 cm.
for the entire area. a. 4,500,000 hills
a. 2,083,333.3 hills b. 4,750,000 hills
b. 2,038,333.3 hills c. 5,000,000 hills
c. 2,830,333.3 hills d. None of the above
d. None of the above
81. A 10-hectare farm will be planted
77. What is the rate of planting in hills with soybean at hill and row spacing of
per hectare if the machine travels at 20 cm x 30 cm. If the seed has 80%
6kph speed? emergence, how many plants are
a. 0.576 ha/hr expected to emerge during the cropping
b. 0.657 ha/hr season.
c. 0.765 ha/hr a. 1,333,111 plants
d. None of the above b. 1,111,333 plants
c. 1,333,333.33 plants
78. Estimate the number of units of d. None of the above
tractors and of planters required to
finish the entire area in 10 days at 8 82. A 5-hectare farm is to be planted
hours/day operation. Assume an 80% with peanut at 250 kg per hectare. The
planting efficiency. (Problem 4) hill and row spacings are 30 cm x 50 cm,
a. 2 tractor units respectively. How many kilos of peanuts
b. 3tractor units is to be planted per hill?
c. 4 tractor units a. 2.25 g
d. None of the above b. 2.52 g
c. 2.72 g
d. None of the above
83. A 4m sugar planter is used to
transplant sugar cane to a 250-hectare
farm. The plant spacing is 60 cm while
the row spacing is 120 cm. Compute the
number of hills the machine can plant
for the entire area.
a. 3,274,222.2 hills
b. 3,472,222.2 hills
c. 3,742,222.2 hills
d. None of the above

84. What is the rate of planting, in hills


per hectare, if the machine travels at a
speed of 8 kph?
a. 0.468 ha/hr
b. 0.684 ha/hr
c. 0.864 ha/hr
d. None of the above

85. Estimate the number of tractors and


planters required to finish the entire area
in 10 days at 8 hours/day operation.
Assume an 90% planting efficiency.
a. 4 tractor and planter units
b. 5 tractor and planter units
c. 6 tractor and planter units
d. None of the above

86. Depth of root zone of rice.


a. 10 – 15 cm
b. 15 – 20 cm
c. 20 – 25 cm
d. None of the above
X. CULTIVATION AND WEED
CONTROL EQUIPMENT

Cultivation - operation that requires


some kind of tool that will stir the
surface of the soil to a shallow depth in
such an manner that young weed will be
destroyed and crop growth promoted.

Operations

• Pre-Planting - being done prior to


planting after the seed bed has been
prepared.
• Post- Planting - being done in some
crops after planting before the crops
emerge.
• Post-Emergence - being done usually
after emergence of crops since weeds
generally emerge at the same time
with the crop.

Objectives of Cultivation Factors to Consider in Selecting the Type


and Size Needed for Cultivator
• Retain moisture by killing the weeds,
• Area of the farm
loose mulching on surface and by
retaining rainfall; • Kind of crop grown
• Develop plant food; • Soil type and condition
• Aerate the soil to allow oxygen to
penetrate the soil; and • Rainfall
• Promote activity of microorganism in • Type of farming practice
the soil.
• Kind of power available
Field Cultivator - Implement for seedbed Classifications of Field Cultivator
preparation, weed eradication, or follow
cultivation subsequent to some form of • Central-Forward-Tractor-Mounted
Cultivator – available in one, two, four,
primary tillage. six, and eight-row sizes. They can be
mounted on both four-wheel and
three-wheel tractors. The one row
cultivator are suitable for small farm
and terraced field with curving rows.
Two rows are suitable for medium-
sized farms and field that have rows
either straight or with gradual curves.

• Rear-Tractor-Mounted Cultivator -
They are usually unit assemblies
attached to tractors equipped with a
three point hitch as a single unit. Each
gang are provided a gauge wheel to
control the depth of cultivation. The
Rotary Hoe – Implement for dislodging gang are attached to a tool bar or
small weeds and grasses and for breaking frame bar.
soil crust and is used for fast, shallow
cultivation.
Cultivator Gangs Factors to Consider in the Selection of
Type of Shovel or Sweep
Gang or rig consists of a beam to which is
attached a shank or standard that has an 1 Type of soil
adjustable foot set at an angle so as 2 Crops
shovel or sweep can be bolted to it. 3 Weeds

• The type of gang assemblies depend on


Cultivator Speed
the following:
• Average speed of horse cultivation is 4
• kind of crops kph.
• soils
• farming practices • Tractor-mounted cultivator is at the
range of 4 to 4.8 kph
• The number and type of soil stirring
members usually determine the gang
style.

Shovels and Sweeps

• A variety of types and shapes of shovels


and sweeps are used for stirring and
killing weeds.
• Shovels are available in widths up to
about 3½ in.; whereas, sweeps can be
obtained in width ranging from 6 to 24
in.
Beet and Bean Cultivator

• Requires shovels and sweeps for


shallow cultivation. Gage wheel
control the depth of penetration of
shovel.
• Sizes of the cultivator range from two
to six rows, four rows being the most
popular in size. Attachment are
available for the application of fertilizer
as a side dressing.

Lister Cultivator

• Particularly adapted to the cultivation


of a listed crop in its early stage of
development. Listed crops are those
planted in the furrow or trench or
below the general level of the ground.
• For the first cultivation, the disks are
set to throw the soil away from the
row of the plants. For all later
cultivation, the disks are set to throw
the soil toward the plants.

Rod Weeder, Field, Sub-soil and Chisel


Cultivators

• Generally used to control weed growth


on fallow lands.

Rotary-Hoe Cultivator

• Implement used to cultivate and to


destroy weeds and grasses around
young plants.
• When rain cause a hard rust to form
over the soil an hinder the emergence
of young seedlings, the rotary hoe is an
excellent tool for pulverizing the crust.
Problems on Weeding

• Reduction of profit by reducing the


yield of the crops.
• Reduction in the quality of the crops.
• Harbors insects that damages the
crops.
•Reduce value of the land

Methods of Controlling Weeds

• Cultivation by stirring the soil


• Use of flame
• Use of chemical
• Laying of plastic strips over the row
REFERENCES

Lovegrove, H.T. 1976. Crop Production


Equipment. A practical guide for
farmers, operators, and trainees.
Hutchinson & Co. Ltd. 3 Fitzroy Square,
London Wi. 406pp.

Smith, H. P. 1977. Farm Machinery and


Equipment. Sixth Edition. Tata McGraw-
Hill Publishing Company Ltd. New Delhi.
488pp.

Stone, A. A. and H. E. Gulvin. 1967.


Machines for Power Farming. Second
Edition. John Wiley and Sons. New York.
559pp.
QUESTIONS and PROBLEMS

1. Operation that requires a kind of tool 6. Operation performed in some crops


to stir the soil surface in shallow depth to after planting but before the crop
destroy young weeds and to promote emerges.
crop growth. a. Pre-planting
a. Weeding b. Post-planting
b. Cultivation c. Post emergence
c. Cutting d. All of the above
d. None of the above
7. Farm implement used for secondary
2. Cultivation operation includes ____. tillage in which the name refers to shanks
a. pre-planting that pierce the soil as they are linearly
b. post-planting dragged through.
c. post emergence a. Cultivator
d. All of the above b. Seeder
c. Weeder
3. Which of the following is not an d. None of the above
objective of cultivation?
a. Retain moisture by killing the weeds, 8. Mechanically-operated machine that
by loosening surface of mulches, and by removes weeds and grasses from the
retaining rainfall. crops.
b. Aerate the soil to allow oxygen to a. Seeder
penetrate the soil. b. Weeder
c. Promote microbial activity in the soil. c. Cultivator
d. None of the above d. None of the above

4. Operation performed prior to planting 9. Ratio between the number of weeds


after the seed bed has been prepared. removed by the weeder to the number of
a. Pre-planting weeds present per unit area.
b. Post-planting a. Weeding index
c. Post emergence b. Weeding rate
d. None of the above c. Weeding capacity
d. None of the above
5. Operation done usually after
emergence of crops since weeds 10. Machine used to kill weed seeds by
generally emerge at the same time with microwave heating during weeding.
the crop. a. Zapper
a. Pre-planting b. Microwave oven
b. Post-planting c. Flame thrower
c. Post emergence d. All of the above
d. All of the above
11. Device used between the cultivator 16. Type of tractor-mounted cultivator
sweep and the plant row to prevent the that can be mounted on both four- and
soil from covering the young plants. three-wheeled tractors.
a. Shield a. Central-forward
b. Fender b. Rear
c. Spread bar c. Any of the two
d. None of the above d. None of the above

12. Machine that has one or more 17. One-row cultivator is suitable for
elements rotating about a vertical axis ____.
and cuts the grass by impact. a. small farms
a. Rotary mower b. terraced fields with curving rows
c. All of the above
b. Grass cutter
d. None of the above
c. Power scythe
d. All of the above 18. Two-row cultivator is suitable for
____.
13. Power-take-off rpm of a lawn mower a. medium-size farms
and garden ride-on tractors. b. fields that have rows either straight or
a. 1000 rpm with gradual curves
b. 540 rpm c. All of the above
c. 2000 rpm d. None of the above
d. None of the above 19. Gangs are attached to a tool bar or to
a frame bar and each gang is provided
14. Which of the following is a with a gauge wheel to ____.
classification of a tractor-mounted a. control the depth of cultivation
cultivator? b. have deep penetration
a. Central-forward tractor-mounted c. All of the above
b. Rear tractor-mounted d. None of the above
c. All of the above
20. Which of the following is a factor to
d. None of the above
consider in selecting the type and the size
of the cultivator to use?
15. Type of unit assemblies of tractor- a. Area of the farm and kind of crop
mounted cultivator attached to tractors grown
equipped with a three point hitch as a b. Soil type and condition
single unit. c. Rainfall and type of farming practice
a. Central-forward d. All of the above
b. Rear
c. Any of the two 21. Type of gang assemblies depends on
d. None of the above the ____.
a. kind of crop and type of soil
b. farming practice
c. All of the above
d. None of the above
22. Implement used for seedbed 28. Gauge wheels control the ____.
preparation, weed eradication, or fallow a. depth of penetration of shovel
cultivation subsequent to some form of b. size of the cultivator
primary tillage, equipped with steel c. attachment for the application of
spring shanks. fertilizer
a. Beet and bean cultivator d. None of the above
b. Lister cultivator
c. Field cultivator 29. Listed crops are those planted in the
d. None of the above ____.
a. furrow
23. Shovels and sweeps are available in b. trench
____ width. c. below the general level of the ground
a. 2½ in. and 5 to 20 in. d. All of the above
b. 3½ in. and 6 to 24 in.
c. 4½ in. and 7 to 28 in. 30. Implement used for both dislodging
d. None of the above small weeds and grasses as for breaking
soil rust, and for fast, shallow
24. Which of the following is a factor to
consider in selecting the type of shovel or cultivation before or soon after crop
plants emerge.
sweep to be used?
a. Type of soil a. Roller harrow
b. Packer
b. Crops
c. Rotary Hoe
c. Weeds
d. All of the above d. None of the above

31. For the first cultivation, disks are set


25. Average speed of a tractor-mounted
to ____.
cultivator is ____.
a. throw the soil away from the row of
a. 4 kph
plants
b. 4 to 4.8 kph
b. throw the soil toward the plants
c. 5 kph
c. Any of the above
d. None of the above
d. None of the above
26. Type of cultivator particularly
32. When soils are thrown away from the
adopted in the cultivation of listed crop in plants during cultivation, the operation is
its early stage of development. ____.
a. Beet and bean cultivator
a. hilling-up
b. Lister cultivator b. barring-off
c. Field cultivator
c. sweeping
d. None of the above d. None of the above
27. Type of cultivator that requires shovel 33. When soil is thrown to the plant
and sweep for shallow cultivation.
during cultivation, the operation is ____.
a. Beet and bean cultivator a. hilling-up
b. Lister cultivator
b. barring-off
c. Field cultivator c. listing
d. None of the above d. All of the above
34. Which of the following is used to 39. Which of the following are classified
control weed growth on fallow lands? as weeds?
a. Rod weeder a. Grasses
b. Field cultivator b. Sedges
c. Subsoil and chisel cultivators c. Broadleaf Weeds
d. All of the above
d. All of the above
e. None of the above
35. Excellent tool for pulverizing when 40. Weed that lives more than two
rain causes the formation of hard rust years.
over the soil and hinder the emergence a. Annual weeds
of young seedlings. b. Biennial weeds
a. Chisel cultivator c. Perennial weeds
b. Rotary-hoe cultivator d. All of the above
c. Field cultivator
41. Which of the following is a method
d. None of the above of controlling weeds?
a. Mechanical
36. Problems on weeds include ____. b. Crop rotation
a. reduction of profit by reducing crop c. Crop competition
yield d. Biological
b. reduction in the quality of the crops e. Fire
c. reduction in the value of the land f. Chemical
d. All of the above g. All of the above
h. None of the above
37. Which of the following is a method
for controlling weeds?
a. Cultivation by stirring the soil.
b. Use of flame and chemical.
c. Laying of plastic strips over the row.
d. All of the above

38. Any plant that is hazard, nuisance or


causes injury to man, animals or his
desired crops.
a. Weeds
b. Wild flower
c. Shrubs
d. All of the above
e. None of the above
XI. SPRAYING AND DUSTING EQUIPMENT

Primary Functions of Sprayer

• To break liquid droplets of effective size


• To distribute the droplets uniformly
over the surface or space to be
protected.
• To regulate the amount of chemical
solution in order to avoid excessive
application that might prove harmful or
wasteful.

Related Terms

Application Rate - amount of any


material applied per unit treated.

Active Ingredient Rate - amount of active


ingredient applied per unit treated,
expressed in terms of mass per relevant
unit treated. Example, kg a.i./ha, mg
a.i./m3 , mg a.i./plant or animal.

Formulation Rate - amount of chemical


formulation applied per unit treated.
Example kg/ha, mg/m3 , mg/plant or
animals.

Spray Rate - amount of spray liquid


applied per unit treated. Example,
liter/ha, ml/m3 , ml/ plant or animals

Deposit Rate - amount of any material


deposited per unit area.

Drift - movement of chemicals outside


the intended target area by air mass
transport diffusion.

Formulation - form of a chemical that is


supplied to the user, and which includes
both the active and the inert ingredients.
Classifications of Sprayer Knapsack Sprayer

1. Hand Sprayer ▪Carried at the back by means of


a. Compressed air sprayer shoulder straps.
b. Knapsack sprayer ▪Simple in design and operation and
2. Power Sprayer relatively cheap to buy and maintain.
a. Hydraulic or field sprayer ▪Capacity is up to 22.5 liters and capable
b. Hydro-pneumatic sprayer to operate at a pressure of 4.2 to 5.4
c. Blower sprayer kg/cm2 .
▪Useful for boom spraying when series of
four nozzles or more are operating at
the same time.

Compressed Air Sprayer

▪Simple in design and operation and are


relative inexpensive to buy and
maintain.
▪Particularly useful for spot and small-
area application.
▪Tank capacity usually 7.5 to 15 liters and
nozzle operating pressure up to 3.5
kg/cm2 .
Power Sprayers

Spraying apparatus that transport or


operated with power source such as
tractor, small engine, or electric motor.

• Usually use plunger or piston pumps,


and
• Usually operate at a pressure of under
40 kg/cm2 .

Types of Sprayer Nozzle

• Cone-Type Nozzle - Consists of tip and


core or swirl plate. Usually operates
between 40 to 60 psi and is lever
operated.
• Fan-Type Nozzle - The shape of the
hole is responsible for the shape of the
spray and the angle formed.
• Impact-Type Nozzle - Pressurized liquid
passes through the nozzle orifice which
Sprayer Parts Definition is relatively large and strikes the angle
face.
Agitator - rotating device located inside
the hopper of gravity flow applicators
and broadcast spreaders that enhances Spray Classifications
delivery of granules to the adjustable
orifice. • Aerosol - Distribution of droplets is less
Drop Tube - conduit either plastic, metal than or equal (≤) to 50 um.
or rubber that directs granule flow of a • Mist - Distribution of droplets is > 50
granular applicator from the metering um and ≤ 100 um.
device to the target. • Fine Spray - Distribution of droplets is >
Rotor - metering device used in positive 100 um and ≤ 400 um.
displacement granular applicators that • Medium Spray - Distribution of
regulates flow rate by displacing a fixed droplets is > 400 um and ≤ 1200 um.
rate per revolution. • Course Spray – Distribution of droplets
> 1200 um.
Methods of Application of Chemical

• Banded Application – Distribution of


chemical in a parallel bands leaving
the area between the bands free of
chemicals.
• Basal Application – Application of
chemical into the base of the plant.
• Broadcast Application – Application
of chemical over an entire area of a
field.
• Directed Application – Application of
chemical to a specific area such as
row bed or base of the plant. Duster
• Foliar Application – Application of
chemical to the stem, fruit, leaves, or
Compressed air from the fan is guided to
needles of the plant.
the bottom of the tank to agitate the
• Spot Treatment – Application of
powder or granules. Due to pressure
chemical to a small restricted area
difference at the throat in the discharge
usually to control the spread of pest.
pipe, the air with a powder or granules is
conveyed to the discharge pipe.
Calibrating Hand-Operated Sprayer

1. Prepare the sprayer


2. Determine the walking speed
3. Determine the width of spray or
swath
4. Calculate the area sprayed
5. Calculate the application rate
6. Calculate the liquid herbicides or
insecticides to mix in each spryer per
load

Aerosol Generators or Foggers


Mechanical or thermal device that
produces a liquid dispersion having a
volume median diameter of less than 50
um. A fog machine sprays liquid-type
chemicals as an aerosol. It is classified
into cold fog machine by generating
process of fog.
USEFUL FORMULA

Application Rate Piston Displacement


10000 Q π d2 L
AR = ---------------- Dp = ---------------
SV 4 (1000)
where: where:
AR - application rate, li/ha Dp - piston displacement, li
Q - delivery, lpm d - diameter of the cylinder, cm
S - swath, m L - length of actual piston travel, cm
V - travel speed, m/min
Sprayer Field Capacity Spraying Speed
S V 167 Qd
FCs = --------------- V = ---------------
10 S Q
where: where:
FCs - sprayer field capacity, ha/hr V - travelling speed, m/s
S - swath, m Qd - total discharge quantity of boom
V - travel speed, kph sprayer, lpm
S - spraying width, m
Q - spraying quantity, li / ha
Actual Sprayer Field Capacity Boom Discharge per Minute

FCa = A s / Ts Qb = Qn Nn
where: where:
FCa - actual field capacity, ha/hr Qb - boom discharge, lpm
A s - area sprayed, ha Qn - nozzle discharge, lpm
Ts - time spent, hr Nn - number of nozzle
Volumetric Efficiency Number of Sprayer Load per Hectare

ξv = (Va / Dp) 100 L = Q / Ct


where: where:
ξv - volumetric efficiency, % L - number of loads per hectare
Va - actual volume discharge, li Q - application rate, li/ha
Dp - piston displacement, li Ct - tank capacity, li per load
SAMPLE PROBLEM

Problem 1

Walking in the rice field operating a 1m-spray-swath sprayer, a person covers 220
m/min. If the nozzle discharge is 0.4 li/min, what is the application rate per
hectare?

Given:
Swath - 1m
Walking speed - 20 m/min
Nozzle discharge - 0.4 li/min

Required: Application Rate

Solution:

Area covered by spray swath per minute =


1 m x 20 m/min = 20 m2 /min

Application rate per hectare = (10, 000 m2 /ha x 0.4 li/min) / 20 m2 /min
= 200 li/ha
Problem 2

A farmer wishes to spray his vegetable farm at a rate of 300 gallons per hectare. A
16-nozzles, 18in. apart, sprayer will be used. The forward speed of the tractor is at 5
kph. Compute the field capacity of the sprayer in hectare per hour. What is the
required pump capacity for the sprayer? Also, what is the capacity per nozzle?

Given:
Spraying rate - 300 gal per ha
Number of nozzles - 16
Nozzle spacing - 18 in.
Tractor speed - 5 kph

Required: Field Capacity, in hectare per hour


Required Pump Capacity
Nozzle Capacity
Solution:
FC = 0.1 W V
= 0.1 (16 nozzles)(18 in./nozzle) (0.254 m/in)(5kph)
= 3.6 ha/hr
Qp = (300 gal/ha)(3.6 ha/hr) (hr/60min)
= 18.3 gpm
Qn = 18.3 gpm/16 nozzles
= 1.14 gpm/nozzle
Problem 3

If a boom sprayer has 6 nozzles and each nozzle discharges 0.6 li/ min, what is the
application rate if the spray swath and walking speed is 25 m/min?

Given:
No. of nozzles - 6
Nozzles discharge - 0.6 li/min
Swath - 6m
Walking speed - 25 m/min

Required: Application Rate

Solution:

Total nozzle discharge per minute = 0.6 li/min x 6


= 3.6 li/min

Area covered per min. = 3 m x 25m/min


= 75 m2 /min

Application Rate per hectare = 3.6 li/min x 10, 000 m2 /min / 75 m2 /min
= 480 li/min

Problem 4

A sprayer containing 100 liters of mixture operates at 0.2 liter per minute per
nozzle discharge. If the sprayer boom consists of 6 nozzles, after how many hours
will the mixture be changed?

Given:
Sprayer capacity - 100 liters
Discharge - 0.2 l per min/nozzle
No. of nozzle - 6

Required: Time to charge the sprayer

Time = 100 liters / (0.2 lpm x 6 x 60 min/hr)


= 1.39 hours
Problem 5

Compute the volumetric efficiency of a sprayer pump having 5cm diameter and 20 cm
stroke. The actual volume of water displaced per stroke is 0.360 liter.

Given:
Pump diameter - 5 cm
Pump stroke - 20 cm
Actual volume displaced
per stroke of the pump - 0.360 liter

Required:
Volumetric efficiency of the pump

Solution

Efficiency volumetric = 0.460 liter x 100 / [3.14/4


x (0.02 m)2 x 0.2 m x 1000 liter/m3 ]
= 0.360 liter x 100 / 0.393 liter
= 92%
REFERENCES

ASAE. ASAE Standards 1997. Standard


Engineering Practices and Data.The
Society for engineering in agricultural,
food, and biological systems. 2950
Niles Road, St. Joseph, MI USA. 978pp.

Hunt, D. 1983. Farm power and


Machinery. 8 th edition. The Iowa State
University Press. Ames, Iowa. 352pp.
QUESTIONS and PROBLEMS

1. Process of applying chemicals to 6. Primary factor that affects the


agricultural crops. performance of the sprayer.
a. Fertilizing a. Spray-particle size
b. Spraying b. Spraying time
c. Dusting c. Spray materials
d. None of the above d. None of the above

2. Equipment that breaks the liquid 7. Amount of spray liquid per unit
droplets to an effective size and distributes treated.
them uniformly over the surface or space a. Deposit rate
to be protected, and regulates the amount b. Active ingredient rate
of chemical solution to avoid excessive c. Spray rate
application that might prove harmful or d. None of the above
wasteful.
a. Duster 8. Unit for spray rate is ____.
b. Sprayer a. mg a.i./plant or animal
c. All of the above b. mg/plant or animal
d. None of the above c. ml/plant or animal
d. None of the above
3. When calibrating a sprayer, an
Agricultural Engineer ____. 9. Which of the following is a primary
a. measures the diameter of its tank function of sprayer?
b. cleans the nozzles a. Break liquid droplets to effective size.
c. measures and adjusts its application b. Distribute the droplets uniformly over
rate the surface or space to be protected.
d. None of the above c. Regulate the amount of chemical
solution to avoid excessive application
4. Amount of any material applied per that might prove harmful or wasteful.
unit treated. d. All of the above
a. Application rate
b. Active ingredient rate 10. Sprayer apparatus consisting of
c. Formulation rate pressure source as well as controlling and
d. None of the above specifically employing an over-the-crop
boom with automizer arranged in order to
5. Amount of chemical formulation provide uniform coverage of the treated
applied per unit treated. surfaces.
a. Application rate a. Boom sprayer
b. Active ingredient rate b. Knapsack sprayer
c. Formulation rate c. Compressed-air sprayer
d. None of the above d. None of the above
11. A farmer wishes to spray his cotton 16. Movement of chemicals outside the
plantation at a rate of 300 gallons per intended target area by air mass
hectare. A 16-nozzle, 18in. apart, sprayer transport diffusion.
will be used. The forward speed of the a. Deposit rate
tractor is 5 kph. Compute the field b. Formulation
capacity of the sprayer in hectare per c. Drift
hour. d. None of the above
a. 5.6 ha/hr
b. 3.6 ha/hr 17. Form of chemical supplied to the
c. 4.7 ha/hr user that includes both the active and
d. None of the above the inert ingredients.
a. Formulation
12. Type of hand sprayer that is useful for b. Drift
boom spraying when series of four c. Deposit rate
nozzles or more are operating at the d. None of the above
same time.
a. Knapsack sprayer 18. Which of the following are the
b. Compressed-air sprayer general classifications of sprayers?
c. Power sprayer a. Hand and power sprayers
d. None of the above b. Compressed-air and knapsack
sprayers
13. Mechanical or a thermal device that c. All of the above
produces liquid dispersion with less than d. None of the above
50 um volume median diameter.
a. Knapsack sprayer 19. Spraying apparatus that transports or
b. Aerosol generator operates with power such as tractor,
c. Compressed-air sprayer small engine or electric motor.
d. None of the above a. Knapsack sprayer
b. Compressed-air sprayer
14. Approximate pressure used in c. Power sprayer
spraying weedicide. d. None of the above
a. 10-25 psi
b. 25-40 psi 20. Type of hand sprayer that is
c. 40-60 psi particularly useful for spot and small
d. None of the above area application.
a. Knapsack sprayer
15. Amount of any material deposited per b. Compressed-air sprayer
unit area. c. Power sprayer
a. Deposit rate d. None of the above
b. Active ingredient rate
21. Capacity and the nozzle operating
c. Spray rate
pressure of a knapsack sprayer are____.
d. None of the above a. 7.5 to 15 liters & 3.5 kg/cm2
b. 17 liters & 3.8 to 4.0 kg/cm2
c. 22.5 liters & 4.2 to 5.4 kg/cm2
d. None of the above
22. Power sprayer usually operates at a 27. Application of chemical over the
pressure ____. entire area of a field.
a. under 40 kg/cm2 a. Basal application
b. at 40 kg/cm2 b. Banded application
c. above 40 kg/cm2 c. Broadcast application
d. None of the above d. None of the above

23. Amount of active ingredient per unit 28. Part of a typical boom-type sprayer
area treated, expressed in terms of mass that indicates the pressure of the
per relevant unit treated. chemical liquid to be released into the
a. Deposit rate sets of sprayer nozzles.
b. Active ingredient rate a. Boom gauge
c. Spray rate b. Relief valve
d. None of the above c. Boom valve
d. None of the above
24. Component of a typical boom-type
power sprayer that returns the chemical 29. Application rate can be determined
incase overpressure is developed at the using the formula ____.
tank. a. area x delivery / (swath x speed of
a. Boom gauge travel)
b. Relief valve b. area x speed of travel / (delivery x
c. Boom valve swath)
d. All of the above c. delivery x swath / (area x speed of
travel)
25. Method of chemical application in d. None of the above
which the distribution of chemical is in a
parallel bands leaving the area between 30. Factors affecting the rate of
the bands free of chemicals. application of sprayer.
a. Basal application a. Pressure of the spray tank
b. Banded application b. Size of nozzle
c. Broadcast application c. Spray swath
d. None of the above d. Speed of the sprayer
e. All of the above
26. Apparatus consisting of a hopper, a f. Three of the above
metering unit, and a distribution device
that uses either gravity, centrifugal force, 31. Lever-operated type of sprayer nozzle
or pendulum action to spread granules to consisting of a tip and a core or a swirl
the surface of the entire area to be plate that usually operates between 40 to
treated. 60 psi.
a. Broadcast spreader a. Cone-type nozzle
b. Granular applicator b. Fan-type nozzle
c. Power sprayer c. Impact-type nozzle
d. None of the above d. None of the above
32. Type of sprayer nozzle in which the 37. Spray classification with droplet
shape of the hole is responsible for the distribution of greater than 400 µm and
shape of the spray and the angle formed. less than or equal to 1200 µm.
a. Cone-type nozzle a. Course spray
b. Fan-type nozzle b. Fine spray
c. Impact-type nozzle c. Medium spray
d. None of the above d. None of the above

33. Type of spray nozzle in which 38. Spray classification with droplet
pressurized liquid passes through the distribution greater than 1200 µm
nozzle orifice which is relatively large and a. Course spray
strikes the angled face. b. Fine spray
a. Cone-type nozzle c. Medium spray
b. Fan-type nozzle d. None of the above
c. Impact-type nozzle
d. None of the above 39. Gas, liquid or solid used to propel or
to transport chemical.
34. Spray classification with greater than a. Spray material
50 µm and less than or equal to 100 µm b. Spray nozzle
droplet distribution. c. Carrier
a. Aerosol d. All of the above
b. Mist
c. Fine spray 40. To reduce the size of spray particle,
d. None of the above ____.
a. increase the operating pressure of the
35. Spray classification with droplet sprayer
distribution of greater than 100 µm and b. adjust the control valve opening of the
less than or equal to 400 µm. sprayer
a. Course spray c. adjust the opening of the spray nozzle
b. Fine spray d. None of the above
c. Medium spray
d. None of the above 41. Method of applying chemicals on the
stem, fruit, leaves, needles, or blades of
36. Spray classification in which the plant.
distribution of droplets is less than or a. Direct application
equal to 50 µm. b. Foliar application
a. Aerosol c. Space application
b. Mist d. All of the above
c. Course spray
d. None of the above
42. Application of chemical into the base of 48. Which of the following is the first
the plant. procedure when calibrating a hand-
a. Basal application operated sprayer?
b. Banded application a. Determine the walking speed
c. Broadcast application b. Calculate the application rate
d. None of the above c. Prepare the sprayer
d. None of the above
43. Method of applying chemicals to a
specific area such as row, bed or at the 49. Which of the following is the last step
base of the plant. in calibrating a hand-operated sprayer?
a. Direct application a. Calculate the liquid herbicides or
b. Spot treatment insecticides to mix in each sprayer per
c. Space application load.
d. None of the above b. Calculate the area sprayed.
c. Determine the width of spray or
44. When a sprayer is to be used, it is
swath.
important to ____.
d. None of the above
a. check the brand of the sprayer first
b. measure the dimension of the tank
50. A sprayer containing 100 liters of
c. calibrate it to meet the required rate of
mixture operates at 0.2 liter per minute
application
per nozzle discharge. If the sprayer
d. None of the above
boom consists of 6 nozzles, after how
45. Insecticide that kills insects when they many hours will the mixture be changed?
suck on plant tissues. a. 2.5 hrs
a. Contact b. 1.39 hrs
b. Stomach c. 0.8 hrs
c. Systemic d. None of the above
d. All of the above
51. Which of the following is a factor that
46. Application of chemical to a small affects the rate of application?
restricted area usually to control the spread a. Pressure of the spray tank and walking
of pest. speed.
a. Directed application b. Size of nozzle opening and spray
b. Foliar application swath.
c. Spot treatment c. All of the above
d. None of the above d. None of the above

47. Treatment method in which liquid or 52. Cold fog machine that sprays liquid-
dry particles are dispersed into the air in type chemicals as an aerosol.
such a manner that target pests are a. Hydraulic sprayer
exposed to chemicals. b. Blower sprayer
a. Spot treatment c. Aerosol generator or Fogger
b. Space treatment d. None of the above
c. Direct application
d. All of the above
53. Mechanical or thermal device that 58. Device used to hold the spray nozzle
produces a liquid dispersion having a close to the soil so the chemicals can be
volume median diameter of less than 50 sprayed on young grass in the plant row.
µm. a. Spray shield
a. Hydraulic sprayer b. Spray boom
b. Duster c. Spray holder
c. Aerosol generator or Fogger d. None of the above
d. None of the above
59. Substance used for spraying that
54. Device in which compressed-air from forms into a finely-divided solid particles
the fan is guided to the bottom of the when mixed with water.
tank to agitate the powder or granules. a. Emulsion
a. Hydraulic sprayer b. Suspension
b. Duster c. Solution
c. Aerosol generator or Fogger d. All of the above
d. None of the above
60. Amount of any material applied per
55. Type of power sprayer where the unit time treated by a sprayer.
spray pressure is built up by the direct a. Throughput rate
action of the pump on the liquid spray b. Application capacity
material. c. Application rate
a. Hydraulic sprayer d. All of the above
b. Blower sprayer
c. Hydro-pneumatic sprayer 61. Sprayer designed to charge the spray
d. All of the above droplet so that there is a powerful
attraction between the plants and the
56. Substance added to the spray charged droplet.
material that serves as sticker, stabilizer a. Electromagnetic sprayer
and conditioning agent. b. Electrolytic sprayer
a. Water c. Electrostatic sprayer
b. Petroleum gas d. All of the above
c. Gelatin
d. All of the above 62. Compute the volumetric efficiency of
a sprayer pump having 5-cm diameter and
57. Type of pump recommended for 20-cm stroke. The actual volume of water
sprayers used for abrasive spray displaced per stroke is 0.360 liter.
materials. a. 85%
a. Centrifugal pump b. 92%
b. Diaphragm pump c. 98%
c. Gear pump d. None of the above
d. All of the above
63. Minimum volumetric efficiency 69. Walking in the rice field operating a
requirement for Lever Operated 1m-spray-swath sprayer, a person covers
Knapsack Sprayer (LOKS). 220 m/min. If the nozzle discharge is 0.4
a. 80% liter/min, what is the application rate per
b. 88% hectare?
c. 95% a. 200 li/ha
d. None of the above b. 250 li/ha
c. 275 li/ha
64. Minimum capacity requirement for d. None of the above
LOKS.
a. 10 liters 70. If a boom sprayer has 6 nozzles and
b. 15 liters each nozzle discharges 0.6 li/ min, what
c. 20 liters is the total discharge of nozzle per min?
d. None of the above a. 2.7 li/min
b. 3.6 li/min
65. Maximum weight required for LOKS c. 4.1 li/min
at full load. d. None of the above
a. 12 kg
b. 22 kg 71. What is the area that can be covered
c. 32 kg by the sprayer in Item 70 per unit time?
d. None of the above a. 57 m2 /min
b. 69 m2 /min
66. Maximum pumping rate requirement c. 75 m2 /min
for LOKS. d. None of the above
a. 20 strokes per minute
b. 40 strokes per minute 72. What is the application rate of the
c. 60 strokes per minute sprayer in Item 70&71 if the spray swath
d. None of the above and walking speed is 25 m/min?
a. 480 li/min
67. Flow rate requirement for LOKS at b. 508 li/min
300 kPa mean pressure. c. 543 li/min
a. 0.3 to 1.4 lpm d. None of the above
b. 1.4 to 2.0 lpm
c. 2.0 to 2.9 lpm
d. None of the above

68. Which of the following is not a liquid


applicator?
a. Backpack or hand-pressurized sprayer
b. Hose-end sprayer
c. Boom sprayer
d. All of the above
e. None of the above
XII. HARVESTING EQUIPMENT

Harvesting - process of gathering matured


crops that is ready for processing or
consumption.

Mechanical Harvester

Machine used for mechanically harvesting


crops that uses internal combustion
engine as power source, usually a self-
propelled or pulled-type unit.

• The design of the harvester highly


depend on the crops to be harvested.
• They vary from feeding mechanism to
product separation equipment.
• Examples are:
• Grain Harvester (Rice, wheat, etc.)
• Corn Harvester
• Sugar Cane Harvester
• Cotton Harvester
• Forage Harvester
• Root Harvester
• Hay Harvester
Classifications of Rice Harvesting
Equipment

1. Manual Harvesting – using hand tools


such as scythe, sickle, and others.
Manpower requirement ranges from
60-80 persons-hr/ha.

2. Mechanical Harvesting – using power


equipment and machineries to gather
matured rice crops.

• Reaper – Only cuts the panicle


during harvesting and lay it down on
a windrow. Panicles are then
collected and bundled so that they
can be safely placed for the next
threshing operation.
• Reaper binder – Cuts the panicle and
bind them together for ease of
hauling into threshing place. Not
widely adopted in the Philippines for
cost and handling reasons.
• Stripper – Does not cut the panicle
but separates the grain instead by the
combing action of the stripper teeth
of the machine. Harvested paddy is
much easier to thresh because of the
few straws that remain intact into the
grains. One disadvantage is the
difficulty in tilling the land where the
stripper passed in harvesting paddy.
• Combine – Harvests and threshes
the grains in one operation. This is
becoming popular in the Philippines
because it harvests paddy at a faster
rate and does not need threshing
since it performs cutting of paddy
panicles and, at the same time,
separates the grains in one operation.
Rice Reaper - machine that cuts the Ways to Reduce Reaper’s Harvesting
panicles and place it on a windrow. Rice Loss
reapers are either power-tiller or tractor
operated machine. 1. Avoid sharp turn when cutting.
2. Reduce cutting speed when cutting
shattering variety.
Losses Related to Reaping 3. Clear the rice panicle off the upper
lugs and star wheels whenever
• Laying or Windrow Loss - good grains possible.
that are shattered when cut crops are
laid from horizontal conveyor to the
ground. Stripper Harvester - machine that does
• Header Loss – good grains shattered not cut the panicle during harvesting
during the cutting and transporting by instead strips-off the grain from the
horizontal flat belts towards the panicle by combing action.
discharge end.
• Crop Damage Loss – unrecoverable
good grains that are available from
standing crop that has been knocked
down or damaged by the machine
during the cutting operation.

Advantage of Stripper Harvester

• Simplicity
• Ease of construction
• Robustness of rotor which can do
varied functions, such as
• Crop lifting
• Harvesting
• Partial threshing
• Crop transport
Combine Harvester - heads the standing Losses Related to Combining of Grains
grains, threshes and cleans the grains as
they move over the field. Using • Shatter Loss – grains lying on the
combine, eliminates the use of grain ground or out of reach of the cutter bar.
binder, of header, and of stationary • Cutter Bar Loss – grains lost due to
thresher as well as the activity of rough handling by the cutter bar.
stacking and hauling of grains. • Cylinder Loss – grains lost over the
• Combine is well adapted for straw rack in the form of unthreshed
harvesting small grains, soybeans, heads.
grain sorghum, rice, and many others. • Separating Loss – grains lost out of the
• The reel-type combine (Western rear of the combine in the form of
design) is more popular than the threshed grain.
check-row type combine (Eastern • Cleaning Loss – loss in the value of the
design). crop due to presence of foreign matter
in the grain tank.

Types of Combine

1. Pull-Type Combine – drawn by a


tractor and power is transmitted
through the power take-off shaft for
smaller unit while for larger unit
through an auxiliary engine.
2. Self-Propelled Combine – powered by
60 to 150hp industrial engine and is
operated by one person.
Corn Harvester - also called as “corn Factors Affecting the Performance of
picker,” either a single- or double-row Corn Picker
machine equipped with snapping rolls to
remove the ears from the standing 1. Plant Characteristics
stalks.
Variety or hybrid suitable for machine
Type of Corn Picker According to Power harvesting, stiff stalks that stand up and
Unit do not break over and lodge, condition
of stalk, height of ears and stalks,
1. Pull-Type – driven by a tractor as toughness of the ear shanks, size of ears
separate unit and power is – large-size ears entail minimum shelling
transmitted through power take-off losses, hard shelling characteristic
drive. reduces shelling losses, thick and tight
2. Self-Propelled – driven by an husk on ear is desirable for snapping but
industrial engine as integral part of not for husking.
the machine.
2. Mechanical Factors

Type of snapping roll surface, adjustment


of snapping rollers - distance apart,
timing of snapping rollers, rate of travel,
type of wagon hitch, adjustment of
dividers to pick-up stalks which are
down.

3. Miscellaneous Factor

Timeliness of harvest - field loss is less if


Types of Corn Picker harvesting is done early, carefulness of
operator, weather condition, cleanliness
1. Snapper – snaps the ears from the of fields - i.e., freedom from tall weeds
stalk and does not remove the husks. and grasses, length of rows, row spacing
2. Picker-Husker – is equipped with a suitable for machine..
husker attachment in addition to
snapper and also removes the husks.
3. Picker-Sheller – snaps the corn and
shells it in the field.
Losses Related to Corn Harvesting Factors Affecting the Performance of
Cotton Stripper
• Loose Ear Loss – Ears separated from
the stalk that lay on the ground beyond 1. Plant characteristics
the reach of the gathering mechanism. 2. Thickness of plant in row
• Picker Ear Loss – Ears left on the ground 3. Cultural practices
still attached to the stalks but escaped 4. Pick-up fingers or limb lifters
the snapping rolls while being picked up 5. Design of stripping unit
by the gathering points. 6. Conveying system
• Shelled Corn Loss – Shelled corn lying 6. Rate of travel
on the ground as a result of shelling
occurring at the snapping roll.
Forage Harvester . Machine used for
• Shelling Loss – Corn kernel lost over gathering silage crops such as grasses or
rack that were not shelled from the cob. straws for feeds.
• Separating Loss – Individual corn kernel
lost over the rack that were not It performs harvesting and chopping
separated from husks, silks, etc. green row crops consisting of plant-
cutting unit and a chopping unit.

Cotton Harvester – also called as cotton


stripper, harvests cotton by combing
action of rolling pickets.

Types of Cotton Harvester

• Double Roller – Machine with


alternate rows of nylon brush bristles
and flexible rubberized strip. They be
either centrally mounted on the
tractor or self-propelled.
• Multiple-Finger or Comb-Type –
Either centrally mounted on the Advantages of Field Forage Harvester
tractor or pulled behind the tractor.
1. Eliminates the drudgery of lifting and
loading heavy green bundles of corn
per hectare.
2. Provides ensilage at lower cost.
3. Provides more tons of fee per silo.
4. Permits filling the silo when the crop is
at the right stage.
5. Make ensilage with greater feeding
value.
6. Provides more uniformity of feeding
value from any part of silo.
7. Provides more uniform, slid pack Operating Speed
without air pockets, thus preventing Reaping or Binding 1.6 – 3.5 kph
molds.
8. Causes no wilting of leaves or loss of
previous moisture. Combining 1.6 – 4.8 kph
9. Leaves no mud or contaminating soil Mowing 1.6 – 4.8 kph
bacteria on butts.
Raking 2.4 – 5.0 kph
10. Avoids soggy material which often
occurs when the silo is filled too early Baling 2.4 – 5.0 kph
with green, immature corn. Field Chopping 3.0 – 5.0 kph

Root Harvester - Machine used to dig and


separate the roots of the crop from the Specific Draft and Power Requirement
soil. It is commonly applied for potatoes,
beets, peanuts, sweet potato, and many Reaping or Binding 1 – 2 kg/row
others.
Combining 2 – 4 kg/row
Mowing 0.5 – 0.8 hp/ft
Raking 0.2 – 0.6 hp/ft
Baling 1 – 3 hp/ton
Field Chopping 1 – 3 hp/ton

Field Efficiency Data


Reaping or 60 – 80%
Binding

Combining 50 – 75%
Mowing 50 – 80%
Raking 60 – 85%
Baling 50 – 75%
Field Chopping 40 – 70%
USEFUL FORMULA

Flat-Belt Conveyor Velocity Pitch of Flat-Belt Lugs

Vb = Vwo P N / π P < D sin (π / N)


where:
Vb = 1.4 Vf
P - pitch of the flat belt lugs, m
where:
D - diameter of star wheel, m
Vb - flat-belt conveyor velocity, m/s
N - number of star wheels
Vwo - velocity of the outer tip of star wheel
lugs, m/s
P - pitch of the flat-belt lugs, m
N - number of star wheel lugs
Vf - machine forward velocity, m/s
Star Wheel Velocity Velocity Ratio

Vw = Vf / cos α K = Vk / Vf
where: k falls 1.3 to 1.4
Vw - average star wheel velocity, m/s where:
Vf - machine forward velocity, m/s K - velocity ratio
α - angle of inclination of star wheel, Vk - average knife velocity, m/s
22 deg Vf - average forward velocity, m/s
Theoretical Field Capacity Effective Field Capacity

TFC = 0.1 W i Vi EFC = TFC ξf


where:
where: EFC - effective field capacity, ha/hr
TFC - theoretical field capacity, ha/hr TFC - theoretical field capacity, ha/hr
W i - width of implement, m ξf - field efficiency, dec
Vi - harvester speed, kph
Field Efficiency

Fe = (efc / tfc) 100


where:
Fe - field efficiency, %
efc - effective field capacity, ha/hr
tfc - theoretical field capacity, ha/hr
SAMPLE PROBLEMS

Problem 1

A combine harvester, with 3m-wide effective width, passes the field at a speed of 4
kph. If the field efficiency of the harvester is 70%, what is its field capacity?

Given:
Width of harvester - 3m
Velocity of harvesting - 4 kph
Field efficiency - 70%

Required: Effective Field Capacity

Solution:

TFC = 0.1 x 3 m x 4 kph


= 1.2 ha per hr

EFC = 1.2 ha per hr x 0.70


= 0.84 ha per hr

Problem 2

A rice reaper is designed to run at a speed of 20 km per hour. What is the design
velocity of the knife of the reaper? If the reaper is 1.0 m wide, what is its
theoretical field capacity?

Given:
Speed of the reaper - 20 kph
Reaper width of cut - 1.0 m
Field efficiency - 70%

Required: Knife Velocity


Theoretical Field Capacity
Effective Field Capacity

Solution:

Knife Velocity = 20 kph x 1.35 x 1000 m/km x 1 hr /3600 sec


= 7.5 m/s
TFC = 0.1 x 20 kph x 1.0 m
= 2 ha/hr
EFC = 0.70 x 2 ha/hr
= 1.4 ha/hr
Problem 3

In a field demonstration of a rice reaper, with 1.2m cutter bar, performance


showed that the average travel speed of the machine is 3 kph. The average width
of cutter bar actually utilized is 1.0 m. Time lost in crossing the dike is 5 min/ha
and 10 min/ha for repair. Turning and other idle time is 10% of the effective
operating time while removing field obstruction and clogging is 15 min/ha. What is
the theoretical and effective field capacity of the machine? What is the time lost in
turning and other idle time per hectare of the reaper?

Given:
Width of implement - 1.2 m
Velocity - 3 kph
Width effective - 1.0 m
Time lost in crossing the dike - 5 min/ha
Time lost in repair - 10 min/ha
Turning and idle time - 10% of effective operating time
Lost in removing obstruction and
clogging - 15 min/ha

Required: Theoretical Field Capacity


Effective Field Capacity
Time Lost in turning and other idle time per hectare

Solution:

TFC = 0.1 (1.2 m) (3 kph)


= 0.36 ha/hr

EFC = 0.1 (1.0 m) (3 kph)


= 0.30 ha/hr

Tl = [1/(0.3 ha/hr)] [0.1]


= [3.33 hr/ha ][0.1]
= 0.333 hr/ha x 60 min/hr
= 20 min/ha
Problem 4

A five-row reaper, with 30cm-spacing per row, is harvesting at a speed of 2 kph.


The field efficiency of the reaper is 70%. If the specific draft of the machine is 2 kg
per row, what is the power output of the reaper?

Given:
No. of row - 5
Row spacing - 30 cm
Speed of harvesting - 2 kph
Field efficiency - 70%
Specific draft - 2 kg/row

Required: Power of the Reaper


Effective Field Capacity

Solution:

Power = Specific draft x No. of row


= 2 kg/row x 5 rows
= 10 kg x 2 kph x 1000 m/km x hr/3600 sec
= 5.56 hp

TFC = 0.1 x (5 rows x 30 cm/row) x 2 kph


= 0.1 x 1.5 m x 2 kph
= 0.30 ha/hr

EFC = 0.30 ha/hr x 0.70


= 0.21 ha/hr
Problem 5

A 6m-wide harvester is harvesting corn at 100 kg per minute. The effective width
of the machine is 90% of its width. If it runs at 2.5kph speed, what are the
effective field capacity and the harvesting rate of the machine in tons per hectare?
Assume a 70% machine field efficiency.

Given:
Width of harvester - 6m
Effective width - 90% of machine width
Harvesting rate - 100 kg per minute
Speed of harvesting - 2.5 kph
Field efficiency - 70%

Required:
Effective Field Capacity
Harvesting Rate, in ton per ha

Solution:

TFC = 0.1 x (6 m x 0.90) x 2.5 kph


= 1.35 ha/hr

EFC = 1.35 ha/hr x 0.70


= 0.945 ha/hr

Harvesting Rate = (100 kg/min x 1 ton/1000 kg x 60 min/hr)/(0.945 ha/hr)


= 6.3 tons per ha
REFERENCES

ASAE. ASAE Standards 1997. Standard


Engineering Practices Data. American
Society of Agricultural Engineers. The
Society for Engineering in Agricultural,
Food, and Biological Systems. 2950 Niles
Road, St. Joseph, MI 49085-9659 USA.
Pp. 264-275.

Hunt, D. 1982. Farm Power and


Machinery Management. Eight Edition.
Iowa State University Press. Ames, Iowa.
325pp.

Smith, H. P. and L. H. Wilkes. 1977. Farm


Machinery and Equipment. Sixth Edition.
Tata McGraw Hill Publishing Company
Ltd. New Delhi, India. 487pp.
QUESTIONS and PROBLEMS

1. Process of gathering matured crops 6. Method of grain harvesting that uses


that are ready for processing or power equipment and machinery.
consumption. a. Manual harvesting
a. Harvesting b. Mechanical harvesting
b. Reaping c. Both a & B
c. Threshing d. None of the above
d. None of the above
7. Mechanical harvesting equipment
2. Which of the following is not a that only cuts the panicle during
classification of grain harvesting harvesting and places them on a
equipment? windrow.
a. Manual harvesting a. Reaper
b. Mechanical harvesting b. Stripper
c. Both a & b c. Combine
d. None of the above d. None of the above

3. Method of grain harvesting that uses 8. Harvesting equipment that cuts the
hand tools such as scythe, sickle and panicle and binds them together for ease
others. of hauling to the threshing place.
a. Manual harvesting a. Combine
b. Mechanical harvesting b. Stripper
c. Any of a & b c. Reaper/binder
d. None of the above d. None of the above

4. Manpower requirement in manual 9. Ratio of the knife velocity to the


harvesting of rice ranges from ____. forward velocity of a rice reaper.
a. 20 to 40 persons-hr/ha a. 1.35:1
b. 30 to 50 persons-hr/ha b. 1.55:2
c. 60 to 80 persons-hr/ha c. 1.55:0
d. None of the above d. None of the above

5. Machine that cuts rice stalks and 10. Good grains shattered during the
places them on a windrow where they cutting and transporting by horizontal
are gathered and bound into sheaves and flat belts towards the discharge end is
carried to a thresher. referred to as ____.
a. laying or windrow loss
a. Scythe
b. header loss
b. Stripper harvester c. crop damage loss
c. Reaper d. All of the above
d. Combines
e. None of the above
11. Losses related to reaping include: 17. Stripper harvesting is more
a. Laying or windrow loss advantageous to use because of its ____.
b. Header loss a. simplicity and ease of construction
c. Crop damage loss
d. All of the above b. robustness of rotor
c. both a & b
12. Unrecoverable good grains that are d. None of the above
available from standing crop that has
been knocked down or damaged by the 18. Which of the following functions can
machine during cutting is referred to as be done by stripper harvesting?
____. a. Harvesting and partial threshing
a. laying or windrow loss b. Crop lifting and transport
b. header loss c. All of the above
c. crop damage loss d. None of the above
d. All of the above

13. Good grains that are shattered when 19. Mechanical harvesting equipment that
cut crops are laid from a horizontal harvests, threshes and cleans the grains as
conveyor to the ground. it moves over the field.
a. Laying or windrow loss a. Combine
b. Header loss b. Stripper
c. Crop damage loss c. Reaper/binder
d. All of the above d. None of the above

14. Ways to reduce reaper ’s harvesting 20. Harvesting equipment that eliminates
losses include: the use of grain binder, header, stationary
a. Avoid sharp turn when cutting thresher as well as stacking and hauling of
b. Reduce cutting speed when cutting grains.
shattering variety a. Reaper
c. Clear the rice panicle off the upper b. Stripper
lugs and star wheels whenever possible. c. Combine
d. All of the above d. None of the above
15. Mechanical harvesting equipment 21. An equipment that is well-adopted for
that does not cut the panicle rather strips harvesting small grains, soybeans, grain
off grains from the panicles by combing sorghum, rice, and many others.
action. a. Reaper
a. Combine b. Stripper
b. Stripper c. Combine
c. Reaper/binder d. None of the above
d. None of the above
22. Which of the following is a type of
16. Machine used for combing the grain combine harvester?
from the plant while the plant remains a. Pull-type combine
anchored to the soil. b. Self-propelled combine
a. Reaper c. All of the above
b. Stripper harvester d. None of the above
c. Combine harvester
d. All of the above
23. Machine powered by 60 to 150hp 28. Portion of a reaper comprising the
industrial engine and is operated by one mechanism for gathering crops.
person. a. Crop gatherer
a. Self-Propelled Combine b. Star wheel
b. Pull-Type Combine c. Header
c. Any of the above
d. All of the above
d. None of the above
29. Portion of a combine comprising the
24. Device at the header of a combine
mechanism for gathering, cutting and
used for severing the plant stalk.
picking the crop.
a. Cutting mechanism
a. Reel
b. Star wheel
b. Header
c. Pickup
c. Cutter
d. None of the above
d. None of the above
25. Machine drawn by a tractor and power
30. Which of the following is not a basic
is transmitted through the power-take-off
function of a combine?
shaft in the case of a smaller unit and
a. Cuts standing grains and feeds the cut
through an auxiliary engine in the case of a
grains to the cylinder.
larger unit.
b. Threshes grains from the stalk and
a. Self-propelled combine
separates them from straw.
b. Pull-type combine
c. Cleans the grains by removing the
c. Any of the above
chaffs and other foreign matters.
d. None of the above
d. None of the above
26. Revolving slats or arms, with battens
31. Device for gathering crops in the
arranged parallel to the cutter bar of a
windrow of a combine.
combine, used to hold the crop in place as
a. Header
it is being cut by the knife and to guide the
b. Pickup
crop as it is conveyed to the platform of
c. Star wheel
the feeder auger.
d. None of the above
a. Star wheel
b. Reel
32. Harvest residue discharged from the
c. Header
cleaning devices of a combine.
d. None of the above
a. Foreign matters
b. Cut straws
27. Assembly of two or more racks which
c. Chaffs
agitates the straw and separates the
d. None of the above
remaining grains from the straw.
a. Straw stripper
33. Which of the following are losses
b. Straw walker
related to combining of grains?
c. Shaker
a. Shatter, cutter bar and cylinder losses
d. None of the above
b. Separating and cleaning losses
c. All of the above
d. None of the above
34. Grains lying on the ground or out of 40. Level of moisture content of paddy
reach of the cutter bar is referred to as during dry harvest season.
____. a. 18%
a. cylinder loss b. 21%
b. cutter bar loss c. 25%
c. shatter loss d. None of the above
d. None of the above
41. A single- or double-row machine
35. Grains lost due to rough handling by equipped with snapping rolls to remove
the cutter bar is referred to as ____. corn ears from standing stalks.
a. cylinder loss a. Corn harvester
b. cutter bar loss b. Corn picker
c. shatter loss c. Both a & b
d. None of the above d. None of the above

36. Grains lost over the straw rack in the 42. Which of the following is not a type of
form of unthreshed heads is referred to as corn picker?
____. a. Snapper
a. cylinder loss b. Picker-husker
b. cleaning loss c. Picker-sheller
c. separating loss d. None of the above
d. None of the above
43. Type of corn picker that snaps the corn
37. Grains lost out of the rear of the and shells it in the field.
combine in the form of threshed grains is a. Picker-husker
referred to as ____. b. Snapper
a. cylinder loss c. Picker-sheller
b. cleaning loss d. None of the above
c. separating loss
d. None of the above 44. Corn picker equipped with a husker
attachment in addition to the snapper and
38. Loss in the value of the crop due to the also removes the husks.
presence of foreign matters in the grain a. Picker-husker
tank is referred to as ____. b. Snapper
a. cylinder loss c. Picker-sheller
b. cleaning loss d. None of the above
c. separating loss
d. None of the above 45. Type of corn picker that snaps the ears
from the stalk and does not remove the
39. Level of moisture content of paddy husks.
during wet harvest season. a. Picker-husker
a. 18% b. Snapper
b. 21% c. Picker-sheller
c. 25% d. None of the above
d. None of the above
46. What are the factors affecting the 52. Ears separated from the stalk and
performance of a corn picker? lying on the ground out of reach of the
a. Plant characteristics gathering mechanism.
b. Mechanical and miscellaneous factors a. Picker ear loss
c. All of the above b. Loose ear loss
d. None of the above c. Shelled corn loss
d. None of the above
47. Which of the following is a
characteristic of corn variety or hybrid that 53. Ears left on the ground that were
is suitable for machine harvesting? attached to the stalks but escaped the
a. Stiff stalks that stand up and do not snapping rolls while being picked up by
break over. the gathering points.
b. Large ear size to reduce shelling losses. a. Picker ear loss
c. Hard shelling characteristics b. Loose ear loss
d. All of the above c. Shelled corn loss
d. None of the above
48. Which of the following is a mechanical
factor that affects the performance of a 54. Shelled corn lying on the ground as a
corn picker? result of shelling occurring at the
a. Type of snapping roll surface snapping roll.
b. Distance adjustment of snapping rollers a. Shelled corn loss
c. Timing of snapping rollers b. Separating loss
d. All of the above
c. Shelling loss
49. Which is not a factor affecting the d. None of the above
performance of a corn picker?
a. Rate of travel 55. Corn kernel loss over rack that was
b. Type of wagon hitch not shelled from the cob.
c. Adjustment of dividers picking up stalks a. Shelled corn loss
that are down b. Separating loss
d. None of the above c. Shelling loss
d. None of the above
50. The following are miscellaneous factors
affecting the performance of a corn picker, 56. Individual corn kernels lost over the
except ____. rack that were not separated from husks,
a. toughness of the ear shanks
silks, etc.
b. timeliness of harvest
c. length of rows and row spacing a. Shelled corn loss
d. None of the above b. Separating loss
c. Shelling loss
51. Which of the following are losses d. None of the above
related to corn harvesting?
a. Loose ear and picker ear losses 57. Machine that harvests cotton by the
b. Shelled corn losses combing action of the rolling pickets.
c. Shelling and separating losses a. Cotton harvester
d. All of the above b. Cotton stripper
c. Both a & b
d. None of the above
58. Which of the following is a type of 63. Machine used for gathering silage
cotton stripper? crops such as grasses or straws for feeds.
a. Double roller a. Cotton harvester
b. Multiple finger or comb-type b. Forage harvester
c. Both a & b c. Corn harvester
d. None of the above d. None of the above

59. Equipment with alternate rows of 64. Which of the following does not
nylon brush bristles and flexible rubberized describe the advantage of using field
strips which is either self-propelled or forage harvesters?
centrally-mounted on a tractor. a. Eliminates drudgery of lifting and
a. Double roller loading heavy green bundles of corn per
b. Multiple finger or comb-type hectare.
c. Both a & b b. Permits filling the silo when the crop is
d. None of the above at the right stage.
c. Provides more uniform, slid-pack
60. Equipment that is either centrally without air pockets, thus preventing
mounted on the tractor or pulled behind molds.
the tractor. d. None of the above
a. Double roller
b. Multiple finger or comb-type 65. Machine that performs harvesting
c. Both a & b and chopping green raw crops which
d. None of the above consists of a plant-cutting unit and a
chopping unit.
61. Which of the following is a factor a. Forage harvester
affecting the performance of a cotton b. Sugarcane harvester
stripper? c. Cotton harvester
a. Plant characteristics and cultural d. None of the above
practices.
b. Thickness of plants in row. 66. Machine used to dig and separate the
c. Pick-up fingers or limb lifters. roots of crops from the soil.
d. All of the above a. Sugarcane harvester
b. Root harvester
62. Which of the following is a factor not c. Citrus harvester
affecting the performance of a cotton d. None of the above
stripper?
a. Design of stripping unit 67. Common applications of a root
b. Conveying system harvester are for ____.
c. Rate of travel a. beets and peanuts
d. None of the above b. potatoes and sweet potatoes
c. All of the above
d. None of the above
68. Based on field data, percent efficiency 73. What is the effective field capacity of
of reaping/binding is ____. the machine in Item 72?
a. 60-80 % a. 0.25 ha/hr
b. 50-75 % b. 0.30 ha/hr
c. 40-70 % c. 0.35 ha/hr
d. None of the above d. None of the above

69. Percent efficiency of combining is 74. What is the time lost in turning and
____. other idle time per hectare of the reaper
a. 50-75 % in Item 72?
b. 50-80 % a. 10 min
c. 60-85 % b. 17 min
d. None of the above c. 20 min
d. None of the above
70. Based on field data, operating speed
when reaping/binding ranges from ____. 75. Prevailing custom rate in combine
a. 1.6-3.5 kph harvesting per 100 sacks of harvested and
b. 2.4-5.0 kph threshed paddy.
c. 3.0-5.0 kph a. 2 to 9 sacks
d. None of the above b. 9 to 12 sacks
c. 12 to 17 sacks
71. Operating speed when combining d. None of the above
ranges from ____.
a. 1.6-3.5 kph 76. In combine harvesting, which of the
b. 1.6-4.8 kph following sequence is true and correct?
c. 2.4-5.0 kph a. Rice panicle is threshed first before
d. None of the above standing grains are cut.
b. Standing grains are cut first before
72. During the field demonstration of a cleaning.
rice reaper with 1.2-m cutter bar, c. Standing grains are cut first and are fed
performance showed that the machine into the threshing unit to separate the
travels at 3-kph average speed. The grains from straws followed by cleaning
average width of the cutter bar actually and collection of clean threshed grains.
utilized is 1.0 m. Time lost for crossing the d. None of the above
dike is 5 min/ha and for repair is 10
min/ha. Turning and other idle time is 10% 77. Part of the combine harvester that
of the effective operating time while properly holds the standing grains against
removing field obstruction and clogging is the knife while the stalks are cut.
15 min/ha. What is the theoretical field a. Reel
capacity of the machine? b. Divider
a. 0.36 ha/hr c. Cutter bar
b. 0.25 ha/hr d. None of the above
c. 0.18 ha/hr
d. None of the above
78. Reciprocating serrated-knife of the 83. A rice reaper is designed to run at a
combine harvester that cuts the standing speed of 20 km per hour. What is the
grains. design velocity of the knife of the reaper?
a. Reel a. 7.5 m/s
b. Divider b. 9.2 m/s
c. Cutter bar c. 10.1 m/s
d. None of the above d. None of the above

79. Typical rice combine harvester in the 84. If the reaper in Item 83 is 1.0m wide,
Philippines is powered by ____hp diesel what is its theoretical field capacity?
engine. a. 1.5 ha/hr
a. 40 b. 2.0 ha/hr
b. 60 c. 2.5 ha/hr
c. 100 d. None of the above
d. None of the above
85. What is the effective field capacity of
80. Diesel fuel consumption of typical the reaper in Items 83 if the field
rice combine harvester being used in the efficiency is 70%?
Philippines. a. 1.4 ha/hr
a. 10 liters per hour b. 1.6 ha/hr
b. 15 liters per hour c. 1.9 ha/hr
c. 20 liters per hour d. None of the above
d. None of the above
86. A five-row reaper, with 30cm-spacing
81. A combine harvester, with 3meter- per row, is harvesting at a speed of 2 kph.
wide effective width, passes the field at a The field efficiency of the reaper is 70%.
speed of 4 kph. What is the theoretical If the specific draft of the machine is 2 kg
field capacity of the harvester? per row, what is the power output of the
a. 1.2 ha/hr reaper?
b. 1.4 ha/hr a. 5.56 hp
c. 1.6 ha/hr b. 6.55 hp
d. None of the above c. 7.12 hp
d. None of the above
82. If the field efficiency of the harvester
In Item 81 is 70%, what is its field 87. What is the theoretical field capacity
capacity? of the reaper I Item 86?
a. 0.81 ha/hr a. 0.30 ha/hr
b. 0.84 ha/hr b. 0.43 ha/hr
c. 0.87 ha/hr c. 0.49 ha/hr
d. None of the above d. None of the above
88. What is the effective field capacity of
the reaper Item 86?
a. 0.18 ha/hr
b. 0.20 ha/hr
c. 0.21 ha/hr
d. None of the above

89. Specific draft and power requirement


for reaping/binding and combining are
____, respectively.
a. 0.5-0.8 and 0.2-0.6 hp/ft
b. 1-3 hp/ton and 1-3 hp/ton
c. 1-2 and 2-4 kg/row
d. None of the above

90. A 6m-wide harvester is harvesting


corn at 100 kg per minute. The effective
width of the machine is 90% of its width.
If it runs at 2.5-kph speed, what is the
harvesting rate of the machine in tons
per hectare? Assume a 70% machine
field efficiency.
a. 5.7 t/ha
b. 5.9 t/ha
c. 6.3 t/ha
d. None of the above
XIII. THRESHING AND SHELLING
EQUIPMENT

Threshing - process of detaching the


grains from the panicle by means of
impact or by rubbing action. Usually
applied to rice, wheat, and other cereal
grains.

Shelling - removal of seeds from the


cobs, pods, or husks after separation
from the plant. Usually applied to corn,
peanuts, and similar crops.

Methods of Threshing

1. Manual Threshing - with the use of a


threshing frame, a flail, or by foot
trampling either by human or by
animals.
2. Mechanical Threshing - by subjecting
the grains to the rotating parts of the
thresher. Capacity ranges from 500
to 2000 kg/hr.

Classifications of Thresher
Classifications of Through Flow
A. According to Operation
• Single Cylinder – Type of thresher which
consists of only one threshing cylinder.
1. Through Flow – Machine whereby
• Double Cylinder – Type of thresher
harvested products are threshed while
which consists of two cylinders
the cut plants are wholly fed into the
arranged in series or one after the
machine through an opening
other.
perpendicular to the cylinder and
directed to an outlet which is also
perpendicular to it. Usually the
diameter-to-length ratio of the
threshing cylinder is 1.
Hold-On Thresher – Mechanical thresher
wherein during the operation, the stalks
are held mechanically by the operator
until all the grains are detached from the
panicle.

2. Axial Flow – Mechanical thresher


whereby the stalk with the grains is
fed into one of the thresher feed tray
located in one side and the materials
moved axially in the the threshing
cylinder until the grains are C. According to the Threshing Units
separated and the straws are thrown
out at the other end of the straw 1. Rasp Bar – Kind of threshing cylinder
outlet. wherein equidistant bar-like protrusion
in parallel orientation are laid on the
B. According to the Method of Feeding periphery of the cylinder.
2. Wire Loop – Kind of threshing cylinder
1. Throw-In Type – Mechanical thresher wherein wire loop of the same arch
whereby the whole cut plants are fed and of equal sizes are attached around
into the machine and major portion the periphery of the cylinder in
of the grains is threshed by the initial tandem arrangement.
impact of the bars or spikes in the 3. Peg-Tooth - Kind of threshing cylinder
cylinder and further threshing is wherein spikes or pegs of equal length
accomplished as the moving panicles are attached around the periphery of
hit the spikes or the bars of the the cylinder in tandem fashion.
concave.
2. Portable Thresher with Oscillating
Screen

• Equipped with oscillating screen,


centrifugal, cross-flow blower for
winnowing screen.
Rasp Bar • Does not require winnowing after the
grains are discharge from the thresher.
• Power requirement is larger than that of
portable threshers.

3. Mobile Axial-Flow Thresher

• Similar but larger in size than the


portable thresher with oscillating
Wire Loop screen.
• Usually equipped with engine as power
unit and a support wheel for ease of
transport.
• They can be easily moved in the field
with the use of a power tiller or of
animals during operation.

4. Self-Propelled Axial-Flow Thresher


Peg-Tooth
• Large-size thresher complete with
Classifications of Axial Flow Thresher oscillating screen that is with power unit
for threshing operation as well as for
1. Portable Axial-Flow Thresher transport.
without Oscillating Screen • Has the ability to move in paddy field
not needing a separate power unit.
• Simple and light weight for ease of • Capacity is much higher than pull-type
movement of machine in the field. mobile threshers.
• Usually designed to be modular in
which major components can be
separated to facilitate mobility and
easy to assemble for use.
• Size are usually small with power. unit
of 16 hp and below.
Components of Axial Flow Thresher

1. Threshing Cylinder – Thresher part


that rotates about the axis equipped
with either pegs, rasp bar, or wire
loop on its periphery. It is the main
part of the thresher that separates
Axial Flow Thresher with the grains from the panicle.
No Oscillating Screen
2. Concave – Thresher part that encloses
the threshing cylinder and allows the
separation of the grains from the
panicle.
a. Lower Concave – Semi-circular
shape wire mesh or bars covering the
lower portion in which the cylinder
rubs and separates the grains from the
Axial-Flow Thresher with panicle and through which grains fall.
Oscillating Screen b. Upper Concave – Semi-circular
shape top cover of the threshing
cylinder consisting of a metal sheet
with louvers and/or grills which assist
threshing and axial movement of the
straw.

Mobile Axial-Flow
Thresher

Self-Propelled Axial-Flow
Thresher
Typical Conventional Rasp Bar Cylinder
Settings for Various Crops
Crop Peripheral Clearance Clearance
Speed Front Rear (mm)
(m/s) (mm)
Barley 27-34 10-18 3-10
Beans 7-20 20-35 10-18
Maize 10-20 25-30 15-20
Oats 27-35 12-20 3-10
Peas 7-18 20-30 10-18
Rapeseed 15-24 20-30 10-20
Rye 25-35 12-20 3-10
Rice 20-30 14-18 3-6
Wheat 24-35 12-20 4-10

Factors Affecting Threshing Capacity

3. Straw Outlet – Straws are 1. Moisture content of the grain - Feed


discharged as they are pushed by material with high content will reduce
the rotating paddles of the rotating the capacity of the thresher.
cylinder of the thresher.
2. Straw-to-grain ratio – Higher straw to
4. Feed Tray – Rice panicles are placed grain ration will reduce the capacity of
for feeding into the threshing the thresher.
cylinder. The length must be more
than the arm length of the operator 3. Length of straw cut - The longer the
for safety. straw, the lesser the capacity of the
thresher.
5. Oscillating Screen – Used to
separate chopped straws that goes 4. Variety – Japonica variety of rice,
with the paddy as it slides into the which is bold in shape, is difficult to
oscillating perforated screen. thresh and reduces the capacity of a
thresher.
6. Blower – Provides the air needed to
aspirate empty grains and other light 5. Operational skill – Knowledgeable
materials from paddy. person in operating the thresher can
maximize the output capacity of a
7. Engine – Provides the power needed thresher.
to propel the thresher.
Factors Affecting the Power 5. Cylinder Diameter – Effective
Requirement of a Thresher diameter of the circle generated by
the outermost points of the cylinder
1. Feeding Rate – The more panicle is threshing elements.
feed into the thresher, the higher is 6. Cylinder Peripheral Speed –
the power requirement. Equivalent linear speed of the
2. Moisture Content of Crops – The outermost point of the cylinder
higher the moisture content of the during threshing.
feed material, the more power is 7. Grain Content – Weight ratio of grain
required. to harvested crop under the same
3. Length of Straw or Stalks – The moisture content of 20%, expressed
longer the straw of the material in percent.
being fed into the thresher, the more
power is required.

Testing Thresher

1. Actual Capacity – Threshing output


collected per unit time.
2. Blower Loss – Ratio of the weight of
collected grains blown with the
straw to the total grain input,
expressed in percent.
3. Clean Threshed Grains – Threshed
grains with 100% purity exclusive of
empty grains.
4. Cracked Grains – Grains whose
endosperm are partially fractured.
REFERENCES

NGA. Harvesting, threshing, and cleaning


of Paddy. Technical Research and
Services Directorate. National Grains
Authority. 32pp.

PCARRD. 1978. The Philippine


Recommends for Rice Postproduction
Operation. PCARRD Technical Bulletin
Series No. 63. PCARRD, Los Banos,
Laguna, Philippines. 130pp.
QUESTIONS and PROBLEMS

1. Process of detaching the grains from 7. According to threshing units,


the panicle by means of impact or by mechanical threshers classifications are:
rubbing action. a. Rasp bar, wire loop & peg tooth
a. Harvesting b. Through-in and through flow
b. Threshing c. Both a & b
c. Drying d. None of the above
d. None of the above
8. Kind of threshing cylinder where
equidistant bar-like protrusions in parallel
2. Methods of threshing could be ____. orientation are laid on the periphery of
a. manual threshing the cylinder.
b. mechanical threshing a. Wire loop
c. both a & b b. Peg tooth
d. None of the above c. Rasp bar
d. None of the above
3. Method of threshing that uses a
threshing frame, a flail or accomplished by 9. Kind of threshing cylinder wherein wire
foot trampling. loop of the same arch and of equal sizes
a. Manual threshing are attached around the periphery of the
b. Mechanical threshing cylinder in tandem arrangement.
a. Wire loop
c. Both a & b b. Peg tooth
d. None of the above c. Rasp bar
d. None of the above
4. Method of threshing which is
accomplished by subjecting grains to the 10. Kind of threshing cylinder wherein
rotating part of a thresher. spikes or pegs of equal length are attached
a. Manual threshing around the periphery of the cylinder in
b. Mechanical threshing tandem fashion.
c. Both a & b a. Wire loop
d. None of the above b. Peg tooth
c. Rasp bar
5. Machine used to detach grains from d. None of the above
the panicle. 11. According to operation, mechanical
a. Reaper threshers are classified as follows:
b. Thresher a. Through flow
c. Sheller b. Axial flow
d. All of the above c. Both a & b
d. None of the above
6. Type of thresher which consists of two
cylinders arranged in series or one after 12. Mechanical threshers are generally
the other. classified according to ____.
a. Double cylinder a. operation and method of feeding
b. Single cylinder b. threshing units
c. Both a & b c. All of the above
d. None of the above d. None of the above
13. According to method of feeding, 18. Mechanical thresher in which the
classifications of mechanical threshers whole cut plants are fed into the machine
are: and major portion of the grains is threshed
a. Throw-in type by the initial impact of the bars or spikes in
b. Hold-on thresher the cylinder and further threshing is
c. Both a & b accomplished as the moving panicles hit
d. None of the above the spikes or the bars of the concave.
a. Through flow
14. Mechanical thresher wherein the b. Throw-in type
stalks with grains are fed into one of the c. Hold-on type
threshing chambers and the materials d. None of the above
moved axially until the straw is thrown
out at the other end when all the grains 19. Mechanical thresher in which the
stalks are held manually by the operator
are removed.
during the operation until all the grains are
a. Axial flow detached from the panicle.
b. Through flow a. Through flow
c. Mobile axial-flow b. Throw-in type
d. None of the above c. Hold-on type
d. None of the above
15. Axial-flow thresher which is simple
and light weight for ease of movement in 20. Machine in which the harvested
the field.
a. Portable thresher with oscillating product is threshed while the cut plants
screen are wholly fed into the machine in an
b. Portable axial-flow thresher without opening perpendicular to the cylinder and
oscillating screen directed to an outlet which is also
c. Mobile axial-flow thresher perpendicular to it.
d. None of the above a. Portable axial-flow thresher
b. Single- and double-cylinder thresher
16. Axial-flow thresher equipped with
oscillating screen and centrifugal blower c. Through-flow thresher
for screen winnowing. d. None of the above
a. Portable thresher with oscillating
screen 21. Type of thresher which consists of only
b. Portable axial-flow thresher without one threshing cylinder
oscillating screen a. Double cylinder
c. Mobile axial-flow thresher b. Single cylinder
d. None of the above c. Both a & b
17. Axial-flow type which is similar with d. None of the above
the portable thresher with oscillating
screen but larger in size usually equipped 22. Grain-straw ratio requirement for
with support wheel for ease of transport. testing a rice thresher ranges from ____.
a. Portable thresher with oscillating a. 0.35 to 0.50
screen b. 0.50 to 0.65
b. Portable axial-flow thresher without c. 0.65 to 0.80
oscillating screen d. All of the above
c. Mobile axial-flow thresher
d. None of the above
23. Commonly used threshing unit for 29. Component of an axial-flow rice
axial-flow rice thresher. thresher that causes the rice straw to
a. Peg-tooth move from one end to the other end
b. Rasp bar during threshing.
c. Wire loop a. Concave
d. None of the above b. Louvers
c. Peg tooth
24. Commonly used threshing units for d. None of the above
throw-in type thresher.
a. Peg tooth 30. Semi-circular shape wire mesh or bars
b. Wire loop covering the lower portion of the cylinder
c. Rasp bar at which the cylinder rubs and separates
d. None of the above the grains from the panicle and through
which grains fall.
25. Part of a thresher that rotates about a. Upper concave
the axis and is equipped with either peg, b. Concave
rasp bar or wire loop on its periphery. c. Lower concave
a. Threshing cylinder d. None of the above
b. Concave
c. Both a & b 31. Semi-circular shape top cover of the
d. None of the above threshing cylinder consisting of a metal
sheet with louvers and/or grills that assist
26. Part of the thresher which encloses in threshing and in axial movement of the
the threshing cylinder and allows the straw.
separation of grains from the panicle. a. Upper concave
a. Threshing cylinder b. Concave
b. Concave c. Lower concave
c. Both a & b d. None of the above
d. None of the above
32. In terms of rasp bar rear clearance,
27. Main part of the thresher that the recommended cylinder setting for rice
separates the grains from the panicle. is ____.
a. Threshing cylinder a. 3 – 6 mm
b. Concave b. 10 – 18mm
c. Both a & b c. 15 – 20 mm
d. None of the above d. None of the above
28. Which of the following is a factor 33. Which of the following separates clean
affecting threshing capacity? threshed grain from straw?
a. Variety and moisture content of the a. Feeding device
grains. b. Threshing cylinder
b. Straw-to-grain ratio and length of c. Concave
straw cut. d. Blower
c. All of the above e. Sieve-shaker/straw walker
d. None of the above f. None of the above
34. Ratio of the weight of clean threshed 40. Diameter of the cylinder of a
grains to the grain input. mechanical rice thresher is the measured
a. Threshing efficiency distance from ____.
b. Threshing recovery a. the base of the threshing element
c. Threshing output passing through the shaft axis
d. All of the above b. the tip of the threshing element
passing though the shaft axis
35. In designing a rice thresher, the most c. the outlet of the cylinder cover
important factor to consider that affects d. None of the above
threshing efficiency is the ____.
a. rpm of the cylinder 41. Which of the following is a factor
b. peripheral speed of the cylinder affecting the power requirement of a rice
c. diameter of the cylinder thresher?
d. None of the above a. Feeding rate
36. Threshing efficiency is basically b. Moisture content
c. Length of straw
determined based on the weight of
d. All of the above
grains obtained at the ____ to the grain
input fed at the feeding trays.
42. Which of the following is a parameter
a. straw outlet
b. oscillating screen outlet considered in testing a thresher?
a. Actual capacity and blower loss
c. blower outlet
b. Clean threshed grains, cracked grains
d. All of the above
and grain content
37. Recommended rpm for the rice c. Cylinder diameter and peripheral speed
thresher cylinder. d. All of the above
a. 400-600 rpm
b. 800-1200 rpm 43. Ratio of the weight of the shelled corn
c. 2000-3000 rpm kernel collected from all the outlets of the
d. None of the above sheller to the total weight of the corn
kernel input of the sheller, expressed in
38. Shaft rpm of a rice thresher needs to percent.
be ____ if the diameter of the threshing a. Shelling efficiency
cylinder is reduced. b. Shelling recovery
a. increased c. shelling index
b. reduced d. None of the above
c. remain the same
d. None of the above 44. Ratio of the weight of collected grains
blown with the straw to the total grain
39. Typical power requirement of an input, expressed in percent.
axial-flow rice thresher. a. Actual capacity
a. 25-hp diesel engine b. Blower loss
b. 16-hp gasoline engine c. Grain content
c. 8-hp diesel engine d. None of the above
d. All of the above
45. Threshed grains with 100% purity 51. Machine used in separating the
exclusive of empty grains. kernels from the cob.
a. Clean threshed grains a. Grader
b. Grain content b. Sheller
c. Cracked grains c. Thresher
d. None of the above d. None of the above
46. Grains with partially fractured 52. Threshing output collected per unit
endosperm.
time.
a. Clean threshed grains
a. Actual capacity
b. Grain content
b. Blower loss
c. Cracked grains
c. Grain content
d. None of the above
d. None of the above
47. Effective diameter of the circle
generated by the outermost points of 53. Based on the typical conventional
the cylinder threshing elements. rasp bar cylinder settings for various
a. Grain content crops, rice has ____ peripheral speed.
b. Cylinder peripheral speed a. 10 – 20 m/s
c. Cylinder diameter b. 20 – 30 m,s
d. None of the above
c. 25 – 35 m/s
48. Equivalent linear speed of the d. None of the above
outermost point of the cylinder during
threshing. 54. Which of the following parts is
a. Grain content responsible in separating the grains from
b. Cylinder peripheral speed the panicle?
c. Cylinder diameter a. Feeding device
d. None of the above
b. Threshing cylinder
49. Weight ratio of grains to be c. Concave
harvested crops under the same d. Blower
moisture content of 20%, expressed in e. Sieve-shaker/straw walker
percent. f. None of the above
a. Grain content
b. Cylinder peripheral speed 55. Which of the following removes
c. Cylinder diameter chaffs and light materials from clean
d. None of the above
threshed grain?
50. Recommended front rasp bar a. Feeding device
clearance for maize is ____. b. Threshing cylinder
a. 12 – 20 mm c. Concave
b. 20 – 30 mm d. Blower
c. 25 – 30 mm e. Sieve-shaker/straw walker
d. None of the above f. None of the above
56. Which of the following is not a part
of a thresher?
a. Feeding device
b. Threshing cylinder
c. Concave
d. Blower
e. Sieve-shaker/straw walker
f. None of the above

57. Axial-flow rice thresher basically are


powered by ____ gasoline engine.
a. 16 hp
b. 22 hp
c. 24 hp
d. None of the above

58. Rate of gasoline consumption of a


16-hp gasoline engine in threshing rice.
a. 1.5 liters per hour
b. 2.0 liters per hour
c. 2.5 liters per hour
d. None of the above

59. Typical field capacity of an axial-flow


rice thresher.
a. 1.0 ha harvest per 6 hours
b. 1.5 ha harvest per 6 hours
c. 2.0 ha harvest per 6 hours
d. None of the above
XIV. CROP DRYING EQUIPMENT

Introduction • Recirculating-Type Dryer – batch dryer


equipped with conveyors to circulate or
Drying - process of removing the excess mix granular product during drying
moisture from the product that is above operation.
its equilibrium moisture content by • Continuous Flow Dryer – type of
applying heat from combustible fuel to mechanical dryer wherein the granular
minimize deterioration or spoilage. product is passed continuously through
a drying chamber utilizing agitator until
Terminologies the product reaches the specified
moisture content reduction per pass
• Dryer – device for removing excess and subsequent cooling is effected
moisture generally by forced through the tempering bins before
ventilation with or without addition of subjecting to another pass.
heat, to prevent the development of
favorable environment for the growth
of molds and insects that normally
cause spoilage.
• Heated-Air Mechanical Dryer –
mechanical dryer used to remove
moisture from a product by blowing
artificially heated air until the required
moisture content is attained.
• Unheated-Air Mechanical Dryer –
mechanical dryer used to remove
moisture from a product by blowing
ambient air until the required moisture
content is attained.
• Flat-Bed Dryer – batch-type dryer with
horizontal holding bin for granular
product and is used for drying at a
maximum drying layer thickness of 45
cm.
• Vertical-Bin Dryer – batch-type dryer
with vertical holding bin and is used for
drying at the same layer with the flat-
bed dryer.
• Deep-Bed Dryer – batch-type dryer
normally used for drying granular
products at a depth more than 45 cm.
Drying time for this type of dryer are
normally longer.
Advantages of Mechanical Drying

• Drying can be done in a shorter period


of time than the natural method by
sundrying.
• Drying can be done regardless of the
weather condition.
• Programming of farm activities can be
more convenient and more profitable.
• Procurement program can be better
managed and can provide more
opportunities to procure more stocks,
especially during rainy season when
farmers are willing to sell their products
at a lower price.
• More product can be dried per unit
space.
• Good quality product can be produced
due to controlled drying procedure.
• Low labor requirement.

Factors Affecting the Choice of the Type


of Drying Equipment for Specific
Installation

• Quantity to be dried
• Drying rate required
• Weather conditions
• Economics
Types of Mechanical Crop Dryer

Batch or Bin Dryer – Materials to be dried


are placed in a bin or container, and air is
forced through the mass until the materials
are dried. This system is simple, moderately
inexpensive, and serves as storage units
after drying is accomplished.

Continuous-Gravity-Flow Dryer – Materials


are allowed to flow from a vertical column
where heated air is forced across the
column bed. This type of dryer is
moderately expensive due to the need of
material handling devices to cause the
movement of the product.

Rotary Dryer – Materials are not free


flowing. This is applicable for chopped
forage, fruits and vegetables, livestock feed,
and fertilizer components. It has high initial
cost and requires more floor spaces per unit
of capacity the batch of column dryer.

Tray Dryer – Materials that cannot be dried


in the previous types are applicable for this
type of dryer. Fruits and vegetables are the
best example of product that is suitable for
this type of dryer. In this type, the materials
are placed in a shallow trays which are
stacked inside the dryer drying chamber.
Parts of Mechanical Dryer 2 Heat Exchanger – converts heat from
the flue gas into clean air. It is used for
1 Burner – device that supplies the the dryer that is heated by indirect
needed heat for drying. method, especially those that employ
biomass burner.
Methods:

• Direct method – flame heats directly


the drying air and is forced through
the drying chamber by means of a fan
or blower.
• Indirect method – drying air is being
heated through a heat exchanger in
order that the product of combustion
will not mix with the drying air and the
product.

3 Fan or Blower - supplies the required


amount of air and pressure needed by
the system. A fan may either be an
axial, vane-axial, or tube-axial. On the
other hand, a blower could be either
radial, backward, forward-curved, or a
mixed- flow type.

4 Drying Chamber – product undergoes


drying during operation. It may be
inside a bin, a cylinder, a tray, or a
cyclone. The product inside the
chamber are allowed to be either
stationary, mixed, float, or fluidized
during the drying process to enhance
drying.
5 Plenum Chamber – dryer component
where the velocity pressure of the air
is converted into static pressure to
obtain a much uniform air distribution
within the chamber.
6 Safety Control Network – includes
accessories such as thermometer,
thermostat, manometer, gas valve,
switches and others to monitor and
control the conditions during drying.
7 Accessories – include conveying
equipment, receiving bin, cleaning
device, tempering bins, bagging or
packaging, auto weigher and others.
Bulk Density of Various Products

Crop Bulk Crop Bulk Density


Fuels for Crop Dryers Density (lb/ft3 )
(lb/ft3 )
• Electricity Barley 38 Peanuts
• LPG Beans Unshelled 14-17
• Kerosene
Lima, dry 45 Popcorn
• Biomass (rice husks, corn cobs, sugar
baggasse, wood, etc.) Lima,
26 Earcorn 28
unshelled
Corn Shelled 45
Drying Parameters Ear, husked 28 Rice, rough 36

Shelled 45 Sorghum 40-45


• Bin Size - dimension of the bin that
contains the product during drying. It Green
28 Soybean 48
is influenced by the bulk density of the sweet
product, as given below. Cottonseed 26 Sunflower 19-26
• Drying Air Temperature – temperature Cowpeas 48 Wheat 48
of the air entering the product to be
dried.
• Air Flow Rate – volume of air in cubic • Burner Efficiency – ratio of the heat
meters delivered to the product per supplied to the heat available expressed
unit time in minutes. in percent.
• Drying Rate – rate of removal of • Heat Utilization Efficiency – ratio of the
moisture from the product. Different total heat requirement to the heat
products require different drying rate. supplied by the burner.
Temperature Requirement of Different
Type of Dryer Drying Rate
Crops
Paddy on Re- 1-2% per hour
Crop Moisture Temperature circulating Dryer
Content, %wb ( °C )
Paddy on Up to 10%
Seed Corn Over 25% 32-43 Conduction-Type
Under 25% 43-50 Rotary Dryer
Commercial For wet milling 55-57
Corn
Small grain Seed 50
Commercial 82
Soybean Commercial 82
Hay Feeding 66
Paddy For seeds 45
Fruits Sliced 60-70
Fish Split and whole 60-70

Approximate Airflow Requirement of the


Various Types of Dryer
Type Approximate Airflow
(m3 /min-ton)

Batch-In-Bin, Small 50

Batch-In-Bin, Large 23

Recirculating Bin, Small 60 – 80

Recirculating Bin, Large 70 – 100

Continuous-Flow, Small 85 – 115

Continuous-Flow, Large 60 - 80
SAMPLE FORMULA

Drying Capacity Final Weight of Dried Material

Cd = (Wi / Td) Wi (100 – Mci)


where: Wf = ------------------------
Cd - drying capacity, kg/hr (100 – MCf)
Wi - initial weight of material, kg where:
Td - drying time, hr Wf - final weight of dried material, kg
Wi - initial weight of material, kg
Mci - initial moisture content, %
MCf - final moisture content, %
Drying Rate Heat Supplied to the Dryer

Wi – Wf 60 (h2-h1) AR
DR = -------------- Qsd = ------------------------
Td γ
where: where:
DR - drying rate, kg/hr Qsd - heat supplied to the dryer, KJ/hr
Wi - initial weight, kg H2 - enthalpy of drying air, KJ/kg da
Wf - final weight, kg H1 - enthalpy of ambient air, KJ/kg da
Td - drying time, hr AR - airflow rate, m 3/min
γ - specific volume, m 3/kg da
Moisture Reduction Rate Heat System Efficiency

%MCf – MCi ξhs = (Qsd / Qaf) 100


MRR = -------------------- where:
Td ξhs - heating system efficiency, %
where: Qsd - heat supplied to the dryer, KJ/hr
MRR - moisture reduction rate, %/hr Qaf - heat available in the fuel, KJ/hr
Wi - initial weight, kg
Wf - final weight, kg
Td - drying time, hr
Heat Available in the Fuel Heat Utilization Efficiency
THU
Qaf = FCR HVf ξhu = ------------ x 100
where: Qsd
Qaf - heat available in the fuel, KJ/hr where:
FCR - fuel consumption rate, kg/hr ξhu - heat utilization efficiency, %
HVf - heating value of fuel, KJ/hr THU - total heat utilized, KJ/hr
Qsd - heat supplied to the dryer, KJ/hr
SAMPLE PROBLEMS

Problem 1

A 2ton-capacity flat-bed dryer dries paddy from 20 to 14 percent in 10 hours.


Compute the moisture reduction from the paddy grains per hour.

Given:
Initial moisture content - 20%
Final moisture content - 14%
Drying Time - 10 hours

Required: Moisture Reduction per hour

Solution:

MR = (20% - 14%) / 10 hours


= 6% /10 hours
= 0.6 percent / hour

Problem 2

If the amount of heat used in removing the moisture from the grains in a
continuous-flow dryer is 3000 kJ/hr, what is the heat utilization efficiency of the
dryer if the burner uses 6000 kJ/hr of fuel?

Given:
Heat utilized - 3000 kJ/hr
Heat supplied - 6000 kJ/hr

Required: Heat Utilization Efficiency

Solution:

HUE = (3000 kJ/hr) x 100 / (6000 kJ/hr)


= 50 %
Problem 3

A 2ton-capacity flat-bed dryer dries paddy from 20 to 14 percent in 10 hours.


What is the drying capacity of the dryer? Also, compute the moisture reduction
from paddy grains per hour.

Given:
Capacity of the bin - 2 tons
Drying time - 10 hours
Initial MC - 20%
Final MC - 14 %

Required: Drying Capacity


Moisture Reduction per Hour
Solution:
Cd = 2 tons/ 10 hours
= 0.2 tons per hour

MR = (20% - 14%) 10 hours


= 6% / 10 hours
= 0.6 percent/ hour
Problem 4

A 2ton mechanical dryer dries paddy from 24 to 14% in 8 hours. What is the final
weight of the product after drying?

Given:
Capacity - 2 tons
MCi - 24%
MCf - 14%
Time - 8 hrs

Required: Final Weight

Solution:

Wf = Wi (1-MCi) / (1 – MCf)
= 2 tons (1-0.24) / (1-0.14)
= 1.767 tons

Problem 5

What is the percentage moisture reduction per hour in Problem 4?

Given:
MC initial - 24 %
MC final - 14 %
Drying time - 8 hrs

Required: % Moisture Reduction per hr

Solution:

% moisture = (24% – 14%) / 8


= 1.25 % per hr
Problem 6

Referring to Problem 4, what is the drying capacity of the dryer?

Given:
Wt initial - 2 tons
Drying time - 8 hrs

Required: Drying Capacity

Solution:

Cd = Wi / Td
= 2 tons / 8 hrs
= 0.25 ton / hr

Problem 7

Still referring to Problem 4, what is the rate of moisture reduction per hour?

Given:
Wt initial - 2 tons
Wt final - 1.767 tons
Drying time - 8 hrs

Required: Moisture Reduction per hr

Solution:

Wt. of Moisture Removed


= 2 tons – 1.767 tons
= 0.233 ton

Moisture Reduction per hr


= 0.232 ton / 8 hrs
= 0.029 ton/hr
REFERENCES

Djokoto, I. K. Crop Dryers. Agricultural


Engineering Training Courses. The
International Rice Research Institute. Los
Banos, Laguna.

Henderson S.M. and R. L. Perry. 1976.


Agricultural Process Engineering. Third
Edition. The AVI Publishing Company, Inc,
Wesport, Connecticut. 442pp.

Hunt, D. 1977. Farm Power and


Machinery Management. IOW State
University Press. Ames, Iowa. 365pp.
QUESTIONS and PROBLEMS
1. Process of removing excess moisture 6. Batch-type dryer with horizontal
from the product above its equilibrium holding bin for granular products and is
moisture content by applying heat. used for drying at a maximum drying
a. Tempering layer thickness of 45 cm.
b. Drying a. Heated-air mechanical dryer
c. Dryaeration b. Flat-bed dryer
d. None of the above c. Unheated-air mechanical dryer
d. None of the above
2. Mechanical dryer wherein granular
products pass continuously through a 7. Batch-type dryer with vertical holding
drying chamber utilizing an agitator until bin and is used for drying at the same
the products reach the specified moisture layer with that of the flat-bed dryer.
content reduction per pass and the a. Flat-bed dryer
subsequent cooling is effected by the use b. Deep-bed dryer
of tempering bins before subjecting them c. Vertical-bin dryer
for another pass. d. None of the above
a. Flat-bed dryer
b. Recirculating-type dryer 8. Temporary holding of grains between
c. Continuous-flow dryer drying passes to allow the moisture in
d. None of the above the center of the grain to equalize with
that on its surface.
3. Drying is important to ____ the a. Tempering
product. b. Static drying
a. minimize deterioration or spoilage of c. Dryaeration
b. safely store d. None of the above
c. Both a & b
d. None of the above 9. Batch-type dryer normally used for
drying granular products at a depth more
4. Mechanical dryer used to remove than 45 cm and normally has longer
moisture from a product by blowing drying time.
artificially-heated air until the required a. Flat-bed dryer
moisture content is attained. b. Deep-bed dryer
a. Heated-air mechanical dryer c. Vertical-bin dryer
b. Flat-bed dryer d. None of the above
c. Unheated-air mechanical dryer
d. None of the above 10. Mechanical dryer used to remove
moisture from a product by blowing
5. Usual depth of grains in a deep-bed ambient air until the required moisture
batch dryer. content is attained.
a. 18 inches a. Heated-air mechanical dryer
b. 2 to 8 ft b. Flat-bed dryer
c. 8 to 10 ft c. Unheated-air mechanical dryer
d. None of the above d. None of the above
11. Recommended depth of grains in a 16. Type of mechanical dryer in which the
shallow-bed dryer. material is allowed to flow along a vertical
a. Up to 12 inches column where heated-air is forced across
b. Up to 18 inches the column bed.
c. Up to 24 inches a. Continuous-gravity-flow dryer
d. All of the above b. Batch or bin dryer
c. Rotary dryer
12. Which of the following statements is d. None of the above
true in relation to the use of mechanical
dryers? 17. Batch dryer equipped with conveyors
a. Drying can be done regardless of to circulate or mix granular products
weather condition. during drying.
b. More produce can be dried per unit a. Flatbed dryer
space. b. Recirculating-type dryer
c. Good quality product can be produced c. Continuous-flow dryer
due to controlled drying procedure. d. None of the above
d. All of the above
18. This system is simple, moderately
13. Which of the following factors affect inexpensive and serves as storage unit
the choice of the type of drying after drying is accomplished.
equipment for a particular installation? a. Batch or bin dryer
a. Economics and weather conditions b. Rotary dryer
b. Quantity of product to be dried c. Continuous-gravity flow dryer
c. Required drying rate d. None of the above
d. All of the above
19. This type of dryer is moderately
14. Type of dryer suitable for chopped expensive because of material-handling
forage, fruits and vegetables, livestock devices needed to cause the product
feeds, and fertilizer components and the being dried to move.
materials being dried are not flowing. a. Batch or Bin dryer
a. Continuous-gravity-flow dryer b. Rotary dryer
b. Batch or bin dryer c. Continuous-gravity flow dryer
c. Rotary dryer d. None of the above
d. None of the above
20. Type of mechanical dryer in which
15. This dryer has higher initial cost and materials to be dried is placed in a bin or
requires more floor spaces per unit in a container, and air is forced through
capacity per batch. the mass until the material is dried.
a. Rotary dryer a. Continuous-gravity-flow dryer
b. Tray dryer b. Batch or bin dryer
c. Spray dryer c. Rotary dryer
d. All of the above d. None of the above
21. In this type of dryer, materials are 27. Device that supplies the needed heat
placed in shallow trays which are stacked for drying.
inside the drying chamber. a. Burner
a. Fluidized-bed dryer b. Fan or blower
b. Spray dryer c. Drying chamber
c. Tray dryer d. None of the above
d. None of the above
28. Type of burner in which the drying air
22. Type of dryer best suitable for fruits is heated with the use of heat exchanger
and vegetables. so the product of combustion does not
a. Spray dryer mix with the drying air and with the
b. Tray dryer product itself.
c. Fluidized-bed dryer a. Direct method
d. None of the above b. Indirect method
c. Any of the above
23. Mechanical dryers can be fueled by d. None of the above
____.
a. electricity 29. Type of burner wherein the flame
b. LPG and/or kerosene heats directly the drying air which is
c. biomass forced through the drying chamber with
d. All of the above the use of a fan or a blower.
a. Direct method
24. Type of dryer suitable for materials b. Indirect method
that cannot be dried in the other types of c. Any of the above
dryer. d. None of the above
a. Spray dryer
b. Tray dryer 30. Ratio of the heat supplied by the
c. Both a & b burner/furnace to the heat released by
d. None of the above the fuel.
a. Furnace/Burner efficiency
25. Device used to supply the required b. Combustion efficiency
amount of air and pressure needed by c. Dryer efficiency
the drying system. d. All of the above
a. Heat exchanger
b. Fan or blower 31. Mechanical grain dryer that is
c. Drying chamber equipped with collapsible plenum
d. None of the above chamber on top of the drying bin to allow
the flow of hot air from top to bottom at
26. Approximate airflow for small the mid of drying operation to eliminate
continuous-flow dryer. mixing of grains in order to attain uniform
a. 85-115 m3 /min-ton drying.
b. 70-100 m3 /min-ton a. Removable-plenum dryer
c. 60-80 m3 /min-ton b. Reversible-flow dryer
d. None of the above c. Two-way-flow dryer
d. All of the above
32. Part of a dryer where the products 37. Include conveying equipment,
undergo drying inside a bin, a cylinder, a receiving bin, cleaning device, tempering
tray, or a cyclone during the process. bins, bagging or packaging, auto weigher,
a. Drying chamber and others.
b. Plenum chamber a. Safety control network
c. Heat exchanger b. Accessories
d. None of the above c. Plenum chamber
d. None of the above
33. Component of a drying system that
converts heat from the flue gas into a 38. Product inside the ____ is allowed to
clean gas and is used for indirect heated be stationary, mixed, float, or fluidized
dryer, particularly those that employ during drying process to enhance drying.
biomass burner. a. plenum chamber
a. Fan or blower b. drying chamber
b. Plenum chamber c. safety control network
c. Heat exchanger d. None of the above
d. None of the above
39. Includes accessories such as
34. Component of a dryer in which the thermometer, thermostat, manometer,
velocity pressure of the air is converted gas valve, switches and others to monitor
into static pressure to obtain a much and to control the conditions during
uniform air distribution in the chamber. drying.
a. Fan or blower a. Safety control network
b. Plenum chamber b. Accessories
c. Heat exchanger c. Plenum chamber
d. None of the above d. None of the above

35. Air chamber maintained under 40. Fan used for a mechanical dryer is
pressure and is usually connected to one a/an ____ type.
or more distribution ducts in a drying a. axial
system. b. vane-axial
a. Drying bin c. tube-axial
b. Plenum chamber d. All of the above
c. Air duct
d. None of the above 41. Pressure build-up in the plenum
chamber of the dryer which maintains
36. Blower used for a mechanical dryer is uniform distribution of air flow through
a ____ type. the grain mass.
a. radial a. Velocity pressure
b. backward- or forward-curved b. Static pressure
c. mixed flow c. Dynamic pressure
d. Any of the above d. Any combination of the above
42. Which of the following biomass 48. Minimum heating efficiency
cannot be used as fuel for a dryer? requirement for a direct-fired biomass-
a. Rice husks fueled grain dryer.
b. Corn cobs a. 75%
c. Sugar bagasse b. 65%
d. None of the above c. 50%
d. None of the above
43. Which of the following is a drying
parameter? 49. Minimum heat utilization efficiency
a. Bin size and air flow rate requirement for a direct-fired petroleum-
b. Drying air temperature based fuel mechanical dryer, based on
c. All of the above the standard performance criteria .
d. None of the above a. 90%
b. 80%
44. Volume of air, in cubic meters, c. 70%
delivered to the product per unit time, in d. None of the above
minutes.
a. Drying air temperature 50. Suggested drying temperature for
b. Bin size sliced fruits as well as for split and whole
c. Airflow rate fish is ____, respectively.
d. None of the above a. 43°-50°C and 60°-70°C
b. 55°-57°C and 60°-70°C
45. Minimum drying efficiency c. 60°-70°C and 60°-70°C
requirement based on the standard d. None of the above
performance criteria for mechanical
dryers. 51. Ratio of the total heat utilized for
a. 90% drying to the heat available in the fuel,
b. 75% expressed in percent.
c. 50% a. Furnace/Burner Efficiency
d. None of the above b. Combustion Efficiency
c. Dryer Efficiency
46. Minimum heating efficiency d. All of the above
requirement for a direct-fired biomass-
fueled grain dryer. 52. Mechanical grain dryer wherein
a. 75% grains pass intermittently in cycles or in
b. 65% stages through a drying chamber either
c. 50% by mechanical means or by gravity with
d. None of the above subsequent cooling and tempering until
the grains reach the desired moisture
47. Typical power requirement of a 6ton content.
flat-bed dryer. a. Batch-In Bin Dryer
a. 7-hp electric motor b. Multi-Pass Dryer
b. 5-hp electric motor c. Recirculating Dryer
c. 3-hp electric motor d. All of the above
d. None of the above
53. Ratio of the heat supplied to the heat 59. Ratio of the total heat requirement to
available, expressed in percent. the heat supplied by the burner.
a. Burner efficiency a. Burner efficiency
b. Heating efficiency b. Heating efficiency
c. Heat utilization efficiency c. Heat utilization efficiency
d. None of the above d. None of the above

54. Rate of moisture removal from the 60. Total amount of heat utilized to
product. vaporize moisture in the material,
a. Drying efficiency expressed in kJ/kg of H2 O.
b. Drying rate a. Moisture reduction
c. Heating efficiency b. Heat utilization
d. None of the above c. Fuel consumption
d. All of the above
55. Amount of water removed from the
product per unit of time, expressed in 61. Axial fan of a flat-bed dryer is driven
kilogram per hour. by a 10-hp diesel engine. If gasoline
a. Moisture reduction engine will be used to drive the fan
b. Drying rate instead of diesel engine, what would be
c. Moisture removal rate the size of the gasoline engine?
d. All of the above a. 9.5 hp
b. 10 .2 hp
56. Recommended drying rate for a c. 11.1 hp
conduction-type rotary dryer is ____. d. None of the above
a. up to 8% per pass
b. up to 10% per pass 62. Continuous-flow dryer in which the
c. up to 15% per pass drying bin is similar to columnar drying
d. None of the above bin except that it includes louvers causing
mixing to occur as the grains flow through
57. Ratio of the average percent moisture the system.
content removed from the grain, to a. Mixing Type
drying time, expressed in percent per b. Stirring Type
hour c. Rotating Type
a. Moisture reduction d. All of the above
b. Heat utilization
c. Fuel consumption 63. A dryer in which the material being
d. All of the above dried moves through the drying chamber
and is subsequently discharged without
58. A 45°C drying temperature is equal to being recirculated.
____. a. Moving-Bed Type
a. 113°F b. Flow-through Type
b. 131°F c. Continuous Type
c. 121°F d. All of the above
d. None of the above
64. Continuous-type intermittent dryer 69. Continuous-flow dryer in which grains
basically consists of ____. in the drying bin flow through the column
a. a series of dryers in a straight path.
b. a drying and series of temporary a. Columnar Type
holding bins b. Non-Mixing Type
c. a holding bins c. Non-Stirring Type
d. All of the above d. All of the above

65. Conventional recirculating-type dryer 70. LSU-type dryer is an example of ____


with tempering bin is an example of ____ type dryer.
type dryer. a. mixing
a. mixing b. non-mixing
b. non-mixing c. reversible
c. reversible d. All of the above
d. All of the above
71. Minimum drying efficiency required
66. Measurement of the dryer holding for heated-air mechanical grain dryers.
capacity based on PAES should be based a. 50%
on ____ of the sample. b. 75%
a. weight c. 95%
b. moisture content d. All of the above
c. purity
d. All of the above 72. Minimum heating system efficiency
for direct-fired petroleum-based heated-
67. Mechanical grain dryer wherein grains air mechanical dryers.
in fixed volume are held in the drying a. 70%
chamber until the grains reach the b. 80%
desired moisture content. c. 90%
a. In-bin Dryer d. All of the above
b. Batch-Type Dryer
c. In-Store Dryer 73. Minimum heating system efficiency
d. All of the above for indirect-fired biomass-fuelled heated-
air mechanical dryers.
68. A conventional 6ton-capacity flatbed a. 50%
dryer basically consists of ____ sheets of b. 60%
1.2 m x 2.4 m perforated sheets as drying c. 70%
floor. d. All of the above
a. 5
b. 7 74. Typical size of engine used in a 6ton
c. 9 flatbed dryer in the Philippines.
d. None of the above a. 10 hp diesel engine
b. 12 hp diesel engine
c. 14 hp diesel engine
d. None of the above
75. Typical fuel consumption rate of a
12hp diesel engine used in a 6-ton
flatbed dryer.
a. 0.9 liter per hour
b. 1.0 liter per hour
c. 1.2 liters per hour
d. None of the above

76. Overall size of an electric motor used


for a 6ton-capacity recirculating- type
paddy dryer in the Philippines.
a. 8 kW
b. 10 kW
c. 12 kW
d. None of the above

77. Power consumption of a 10kW


electric motor in drying 6 tons of paddy
in a recirculating-type paddy dryer.
a. 11.6 to 15.0 kW-hour/ton
b. 15.1 to 19.0 kW-hour/ton
c. 19.1 to 24.0 kW-hour/ton
d. None of the above

78. A 2ton-capacity flat-bed dryer dries


paddy from 20 to 14 percent in 10 hours.
Compute the moisture reduction from
the paddy grains per hour.
a. 0.6 % per hour
b. 0.7 % per hour
c. 0.8% per hour
d. None of the above
XV. RICE MILLING EQUIPMENT

Milling - process of converting paddy into


rice. It is to be carried out with outmost
care to prevent breakage of the kernel
and improve the recovery of the paddy.

Milling Process

• Cleaning - removing foreign materials


such as rice straws, stones, seeds, etc.
from the paddy.
• Dehusking and Husk Separation -
removing the husk from the paddy
with minimum damage to the grain,
and separating the husk from paddy.
• Paddy Separation - separating
dehusked paddy from the remaining
grains. Most dehusker remove about
90% of the husk.
• Bran Removal - removing all or part of
the bran layer from the grain to
produce polished rice.
• Grading - separating or grading broken
grains from unbroken rice. Broken
grains are separated into different
sizes.
• Sifting – separating of brewer ’s grains,
impurities, etc. from milled rice.
• Bagging Bins – collecting milled rice.
Dehusking Equipment n Rubber-Roll Paddy Husker - Consists
of two rubber rolls rotating in
n Steel huller - Combines the opposite direction at different speed.
dehusking and polishing processes in One roll moves slower than the other
one operation. Husking is by 25%. The difference in the
accomplished due to friction peripheral speed causes the shearing
between grains and steel parts of the action which dehusked the grains.
huller which cause the husk and the Faster rubber rolls worn out quicker
bran to be scraped off. This machine than the other. Wearing out rubber
has lower head rice recovery and roll reduces the diameter and capacity
high power requirement. Steel of the huller. The advantage is the
huller operates at 800-900 rpm for high milling recovery while the
smaller mill and 600-800 rpm for disadvantage is the cost of
larger mill. replacement of the rubber roller. The
average capacity of the pair of rubber
is 100-200 tons paddy per pair. The
optimum age of rubber begins 2-3
months after manufacture and
decreases rapidly when the rubber is
6-9 months old.

n Under-Runner Disc Huller - Consists


of two horizontal iron discs partly
coated with abrasive layer. The top
disc is fixed to the housing while the
bottom disc rotates. Paddy is fed at
the center and moves outward by
centrifugal force which causes the
grains to dehusk due to the friction.
Its main advantage is that it is simple
and low running cost.
Capacity and Power Requirement of Paddy Separation Equipment
Under-runner Disc Huller
Disc Capacity Horsepower • Compartment-Type Paddy Separator -
Diameter (Kg Requirement made of steel or wood consisting of
(mm) paddy/hr) number of compartments in one or
750 450 – 600 3 more decks. The number of
1000 700 – 1000 3.5 compartments determine the capacity.
1250 1000 – 1400 4
Each compartment has a capacity of
40kg brown rice per hour for long
1400 1600 – 2100 5.5
grains and 60 kg/hr for short grains.

• Tray Separator - consists of several


Capacity and Power Requirement of indented trays mounted one above the
Different Sizes of Rubber-Roll Huskers other about 5 cm apart all attached to
an oscillating frame. Tray moves up
Sizes Dimension Rolls Capacity (t/h) Horse
and forward causing the brown rice
(mm) power
Require and paddy to be separated. Capacity
Length Diam- Long Short ranges from 1.2 to 9.5 tons per hour.
eter Grains Grains ments

4 100 220 0.9 1.25 2.5

6 150 220 1.2 1.9 4

10 254 254 2.2 3.8 6

Number of Bags Processed by a


Pair of Rubber Rolls
Rubber Roll Size Number of Bags
(in.) (50 kg paddy
processed per
pair of rubber roll)
2 -1/2 200 – 250

3 300 – 350

4 400 – 600

6 850 – 900

8 950 – 100

10 1500 – 2000
Bran Removal • Horizontal Abrasive Whitener - More
compact than the vertical abrasive
• Vertical Abrasive Whitener - whitener. It consists of an abrasive roll
Dehusked paddy enters the top center (emery stone to steel shaft) operating
and moves outward by centrifugal in a cylindrical metal perforated screen
force to the edge of metal cone. The mounted horizontally. Brown rice
cone has abrasive surface and turn enter at one end and discharges at the
inside a cylinder covered with screen. other end. Bran removal is
Clearance between the cone and accomplished similar to the vertical
screen is adjusted at 10 mm by raising abrasive whitener.
and lowering the cone. Vertical rubber
brakes (30 to 50mm wide) is placed at • Horizontal Friction Whitener - Often
an interval around the cone. Abrasive called jet or pneumatic pearler. Each
action takes place on brown rice as it machine uses a friction process in
moves down between the cone and which the bran is peeled off by friction
the screen. Peripheral speed of the of the rice grains. Steel hullers are also
cone is about 13 m/sec. Air aspiration used as friction-type whitener. Air is
through the whitener reduces used to remove bran.
breakage caused by heating and keeps
the dust out of the mill. • Rice Polishers - Sometimes called
pearlers or refiners to make the rice
more glossy and highly polished. They
are made of vertical or horizontal
abrasive whitener wherein cones are
covered with leather strips and
operated at lower rpm. The leather
strips roll the whitened rice against the
screen which make the remaining bran
to be removed and rice become shinier
and glossier. The machine produces
few broken grains and power
Air Quantities consumption is lesser than by 30 to
40% than that of the whitener.
Cone Size (mm) Air Quantities
(m3 /min)
500 8 Grading Equipment
600 10
800 12 • Vibrating or rotary sieves - used for
1000 15
separating small grains
• Trieurs, rotating cylinder, or drum
1250 20
grader - used for large broke grains
1500 20
Terminologies Milling Degree - extent or degree by which
the bran covering of the brown rice is
Abrasive-Type Whitener - consists of an removed as result of the whitening
abrasive-coated cylinder or cone which process.
rotates inside a perforated steel housing. Head Milled Rice - percentage mass of
head milled rice recovered from the mass
Frictional-Type Whitener - consists of a of the corresponding input paddy to the
fluted roller rotating inside a hexagonal rice whitener.
chamber with slot-type perforations. Head Milled Rice Recovery Index - ratio of
the head milled rice recovery from the test
Maximum Input Capacity - maximum rice mill to that from the laboratory
quantity of paddy that the rice mill can whitener.
process to a specified quality over a Percentage Broken Milled Rice -
given period, expressed in kg/h. percentage of milled rice grains measuring
less than ¾ the size of its full head rice
Output Capacity - mass of milled rice per from the mass of the corresponding input
unit of milling time, expressed in kg/h. paddy to the rice husker then to the
whitener.
Coefficient of Husking - ratio of the mass Broken Milled Rice Index - ratio of the
of brown rice to the total mass of paddy percentage broken milled rice recovery
input to the husker. from the test rice mill to that from the
laboratory whitener.
Coefficient of Wholeness - ratio of the
Total Milling Recovery - percentage total
mass of head brown rice grain to the mass of milled rice (head and broken rice)
total mass of brown rice (head broken) to the mass of the corresponding input
produced by the husker. paddy to the rice mill (husker or huller and
whitener).
Husking Efficiency - product of
Total Milling Recovery Index - ratio of the
coefficient of husking and the coefficient
milling recovery from the test rice mill to
of wholeness. that from the laboratory whitener.

Percentage Cracked Brown Rice -


number of grains showing cracks of
fissures per 100 grains of hand-husked
paddy.

Hardness of Grain - compressive


strength of a full head, uncracked and
hand-husked, brown rice as measured by
a hardness tester.
USEFUL FORMULA

Hulling Coefficient Wholeness Coefficient

Ch = W br / W p Cw = W wbr / W br
where: where:
Ch - hulling coefficient, dec Cw - wholeness coefficient, dec
W br - weight of brown rice, g W wbr - weight of whole brown rice, g
W p - weight of paddy, g W br - weight of brown rice, g
Hulling Efficiency Percentage Brown Rice Recovery

ξh = Ch Cw %BRR = (W brr / W p ) x 100


where: where:
ξh - hulling efficiency, dec %BRR - percentage brown rice recovery, %
Ch - hulling coefficient, dec W brr - weight of brown rice, kg
Cw - wholeness coefficient, dec Wp - weight of paddy, kg
Percentage Broken Milled Rice Throughput Capacity

%BR = (W br / W mr) 100 Ct = 0.2 W p / To : brown rice


where: Ct = [W p MR]/To : milled rice
%BR - percentage broken rice, % where:
W br - weight of broken rice, kg
Ct - throughput capacity, kg/hr
W mr - weight of milled rice, kg W p - weigh t paddy input, kg
To - operating time, hr
MR - milling recovery, dec 0.60 to 0.69
Percentage Brewer ’s Rice Head Rice Recovery

%BrR = (W brr / W mr ) 100 %HR = (W hr / W mr ) 100


where: where:
%BrR - percentage brewer’s rice, % %HR - head rice recovery, %
W brr - weight of brewer’s rice, kg W hr - weight of head rice, kg
W mr - weight of milled rice, kg W mr - weight of milled rice
Milling Recovery Speed of Low Speed Rubber Roller

%MR = ( W mr / W p ) 100 Ns = Nh - [0.25 / Nh]


where: where:
% MR - milling recovery, % Ns - speed of slower rubber roller, rpm
W mr - weight of milled rice, % Nh - speed of faster rubber roller, rpm
Wp - weight of paddy, kg
Number of Compartments for Paddy Number of Brake for Vertical-Abrasive
Separator Whitener

NC = Cb / 40 : long grain NB = [D / 100] : Germany


NC = Cb / 60 : short grain NB = [D / 100] : Italy
where: where:
NC - number of compartments NB - number of brakes, unit or pc
Cb - throughput capacity, kg brown rice D - cone diameter, mm
per hr
Rubber Roll Peripheral Velocity

Vp = 𝝅 D N / 60

Where:
Vp - rubber roll huller peripheral velocity,
m/s
D - diameter of rubber roll, m
N - speed of rubber roll, rpm
SAMPLE PROBLEM

Problem 1

A single-pass rice mill was tested for 30 min. and gave the following results: input
paddy, 1000 kg; milled rice produced, 650 kg; broken rice produced, 150 kg; and
weight of head rice, 200 kg. Compute the following: (a) Throughput capacity, (b) %
Milling recovery, (c) % Broken grains, and (d) % Head rice recovery.

Given:
Wp - 1000 kg
W mr - 650 kg
W hr - 500 kg
W br - 150 kg
Time - 30 min.

Required: Throughput capacity


% Milling Recovery
% Broken Grains
% Head Rice Recovery

Solution:

Throughput Capacity = Wp x MR / To
= 650 / 30 min/60 min/hr
= 1300 kg milled rice per hr

% Milling recovery = W mr 100 / W p


= 650 kg x 100 / 1000 kg
= 65%

% Broken rice = Wbr 100 / Wmr


= 150 kg x 100 / 650 kg
= 23%

% Head rice recovery = Whr 100/Wmr


= 500 kg x 100 / 650 kg
= 77%
Problem 2

Five tons of paddy milled in 6 hours produces 3950 kg of brown rice and 3250 kg of
milled rice. What is the milling recovery?

Given:
Wt of paddy - 5 tons
Milling time - 6 hrs
Wt of brown rice - 3950 kg
Wt of milled rice - 3250 kg

Required: Milling Recovery

Solution:

MR = Wt milled rice x 100 / wt of paddy


= 3250 kg x 100 / 5000 kg
= 0.65 x 100
= 65%

Problem 3

What is the hulling coefficient of the huller in Problem 2?

Given:
Weight of brown rice - 3950 kg
Weight of paddy - 5000 kg

Required: Hulling Coefficient

Solution:

Coef. of hulling = Wt of brown rice/Wt of paddy


= 3950 kg / 5000 kg
= 0.79
Problem 4

A rubber-roll husker was tested to determine


its performance. The husker ran for 2 hours
at a rate of 1000 kg of input paddy. The
weight of brown rice obtained after passing
the husker was 805 kg. The weight of whole
brown rice obtain per kg of sample was 900
grams. What is the hulling coefficient,
wholeness coefficient, and hulling efficiency
of the huller? What is the throughput
capacity of the husker, in kilogram per hour of
brown rice?

Given:
Weight of paddy - 1000 kg
Operating time - 2 hrs
Weight of brown rice - 805 kg
Weight of whole brown rice - 900 g per
kg of brown rice

Required:
Hulling Coefficient, Wholeness Coefficient,
Husking Efficiency, and Throughput Rate

Solution:

Hulling Coefficient = 805 kg/1000 kg


= 0.805

Wholeness Coefficient = (900 g brown rice


x 1kg/1000 g) x 805 kg brown rice /
805 kg brown rice
= 0.9

Hulling Efficiency = (0.805 x 0.90) x 100


= 72.45%

Throughput Capacity = 805 kg / 2 hrs


= 402.5 kg/hr
Problem 5

A disk huller with 600mm diameter will be installed as a return huller for a rice
milling plant. As an Agricultural Engineer, determine the diameter of the pulley
required for the huller. The motor for the huller will have 4in.-diameter pulley and
is expected to run at 1740 rpm.

Given:
Diameter of huller - 600 mm
Motor pulley - 4 in.
Motor speed - 1740 rpm
Required: Huller Pulley Diameter
Solution:
Huller rpm = (14 m/s ) (60 s/min) /[ 3.14 (0.6 m)]
= 445.8 rpm
Huller Pulley D = 4 in. (1740 rpm) / 445.8 rpm
= 15.6 in. use 16 in.
REFERENCES

Camacho, I. R. Rice Milling Systems. Ag.


Engineering Training Program. The
International Rice Research Institute. Los
Baños, Laguna. 29pp.

NAPHIRE. Improving Rice Milling


Efficiency. pp.101-117

RNAM. Test Code and Procedure for Rice


Mills. ESCAP/RNAM. c/o UNDP, PO Box
7285. Domestic Airport Office Lock Box.
1300 Domestic Road, Pasay City, metro
Manila. 56pp. October 1990.

The Rice Postproduction Operations


Committee. The Philippines
Recommends for Rice Postproduction
Operations, Los Baños, Laguna: PCARRD;
NAPHIRE, 1987. 130p. (PCARRD
Technical Bulletin Series No. 63). 130pp.

Wimberly, J.E. 1983. Technical Handbook


for the Paddy Rice Postharvest Industry in
Developing Countries. The International
Rice Research Institute. Los Baños,
Laguna, Philippines. 188pp.
QUESTIONS and PROBLEMS

1. Hydrothermal treatment of paddy 7. Broken rice is a quality deterioration


before milling is called ____. index during ____.
a. parboiling a. drying
b. drying b. threshing
c. water treatment c. milling
d. None of the above d. None of the above

2. Extent by which the bran layer of the 8. Fraction of grain with its length equal
brown rice is removed as a result of to or greater than an average of three-
whitening. fourths.
a. Milling degree a. Large broken grain
b. Degree of whitening b. Small broken grain
c. Milling recovery c. Head grain
d. All of the above d. None of the above
9. Minimum hulling efficiency
3. Machine that converts paddy to milled requirement for rice mill, based on
rice. standard performance criterion.
a. Paddy husker a. 90%
b. Rice whitener b. 80%
c. Rice mill c. 70%
d. All of the above d. None of the above

4. Kind of rice mill that employs only one 10. If five tons of paddy is milled in 6
whitening machine. hours produces 3950 kg of brown rice
a. “Kiskisan” rice mill and 3250 kg of milled rice, what is the
b. Single-pass rice mill milling recovery?
c. Rubber-roll rice mill a. 60%
d. None of the above b. 65%
c. 70%
5. Ratio of the weight of milled rice to the d. None of the above
weight of paddy multiplied by one 11. Referring to Item 10, what is the
hundred. hulling coefficient of the huller?
a. Percentage head rice recovery a. 0.79
b. Percentage milling recovery b. 0.89
c. Percentage milling capacity c. 0.99
d. None of the above d. None of the above

6. Maximum percentage of milling 12. If the head rice recovery of paddy in


recovery from rice mills. Item 10 is equal to 85%, what is the
a. 69% amount of broken grains?
b. 75% a. 487.50 kg
c. 80% b. 478.50 kg
d. None of the above c. 475.85 kg
d. None of the above
13. Referring to Item 10, what is the 19. If the end-product in milling is brown
amount of rice hull produced during rice, the grain has just passed through a
milling? ____.
a. 1000 kg a. paddy cleaner
b. 1050 kg b. Rubber-roll huller
c. 1100 kg c. whitener
d. None of the above d. sifter
e. None of the above
14. Multi-pass rice mill is characterized by
____. 20. Recommended peripheral speed of a
a. having two or three separate rubber roller for hulling operation.
hullers in the system a. 10 m/s
b. having a series of two or three b. 14 m/s
whitening machines c. 18 m/s
c. having the paddy loaded in the rice mill d. None of the above
several times 21. Percentage speed difference
d. None of the above between rubber rollers.
a. 15%
15. Rice mill huller that has high milling b. 20%
recovery. c. 25%
a. Steel huller d. None of the above
b. Rubber huller
c. Stone disk 22. Speed of the fixed rubber roll in a
d. All of the above rubber-roll huller is slower by ____
compared with that of the adjustable
16. Example of a rice huller. roller.
a. Steel-fluted huller a. 20 %
b. Rubber-roll b. 25 %
c. Disk c. 30 %
d. Centrifugal d. None of the above
e. All of the above
23. Husking ratio of a rubber-roll huller is
17. The measure of the machine’s ability about ____.
to remove hulls with minimum breakage. a. 60-70 %
a. Coefficient of hulling b. 80-90 %
b. Coefficient of breakage c. 95-98 %
c. Coefficient of wholeness d. None of the above
d. All of the above 24. If the rubber-roll huller wears out,
the peripheral speed of the roller will
18. Dehulled paddy but with bran layer ____.
remaining intact. a. decrease
a. Rough rice b. increase
b. Milled rice c. be the same
c. Brown rice d. None of the above
d. All of the above
25. A rice processing plant operates at 5 30. Recommended clearance for a disk
tons per hour. The milling and head rice huller in order to achieve high brown-
recoveries of the rice mill are 69% and rice recovery with less breakage.
95%, respectively. What is its input a. ¼ of the length of the paddy grain
capacity? b. ½ of the thickness of the paddy grain
a. 7.24 tons per hour c. ¾ of the length of the paddy grain
b. 3.45 tons per hour d. None of the above
c. 4.75 tons per hour
d. None of the above 31. Machine used to remove the bran
from brown rice.
26. Referring to Item 25, what is the a. Polisher
throughput rate of the rice mill in ton b. Husker
broken grains per hour? c. Whitener
a. 0.172 ton per hour d. All of the above
b. 0.250 ton per hour
c. 0.350 ton per hour 32. Type of whitener consisting of a
d. None of the above flutted-roller rotating inside a hexagonal
chamber with slot-type perforations.
27. If a rice huller has 0.90 hulling
a. Abrasive-type whitener
coefficient and 0.80 wholeness
b. Frictional-type whitener
coefficient, what is the output per hour of
c. All of the above
the rice huller in Item 25 in tons of brown
d. None of the above
rice?
a. 5.21 tons per hour
33. Manufacturers’ recommended speed
b. 4.50 tons per hour
for the rotating cone in a vertical-
c. 4.00 tons per hour
abrasive whitening cone must not exceed
d. None of the above
____.
28. In a disk huller, the stationary disk is a. 13 m/s
located at the ____. b. 14 m/s
a. top of the huller c. 15 m/s
b. bottom of the huller d. None of the above
c. side of the huller
d. None of the above 34. Auxillary device in a rice mill that
receives the fine bran particles from
29. Manufacturers’ prescribed peripheral milled rice and gives a glossy appearance
velocity for a disk huller is ____. to the product.
a. 10 m/s a. Whitener
b. 14 m/s b. Grader
c. 18 m/s c. Polisher
d. None of the above d. All of the above
35. Manufacturers’ recommended speed 41. A one-ton-per-hour rice mill is to be
for a horizontal-abrasive whitener is ____. designed. The milling recovery is 68%
a. 800 rpm while the hulling efficiency is 90%. What
b. 1000 rpm size of paddy separator will you
c. 1200 rpm recommend?
d. All of the above a. 26 compartments
b. 32 compartments
36. Adjustment clearance for rubber c. 40 compartments
brakes in a vertical-abrasive whitening d. None of the above
cone is ____ from cone coating.
a. 1 to 2 mm 42. Roller dimension of No. 4 rubber-roll
b. 2 to 3 mm huller.
c. 3 to 4 mm a. 100 mm long x 220 mm diameter
d. None of the above b. 150 mm long x 220 mm diameter
c. 254 mm long x 254 mm diameter
37. Machine that smoothens the surfaces d. None of the above
of milled rice after removing the bran
making it shiny. 43. Roller dimension of No. 6 rubber-roll
a. Whitener huller.
b. Polisher
a. 100 mm long x 220 mm diameter
c. Sifter b. 150 mm long x 220 mm diameter
d. Grader c. 254 mm long x 254 mm diameter
e. All of the above d. None of the above
38. Machine used to segregate various
fractions/sizes of milled rice. 44. Roller dimension of No. 10 rubber-
roll huller.
a. Whitener
a. 100 mm long x 220 mm diameter
b. Grader
b. 150 mm long x 220 mm diameter
c. Whitener
c. 254 mm long x 254 mm diameter
d. None of the above
d. None of the above
39. Machine that separates paddy from
brown rice. 45. Throughput capacity of No. 10
a. Screen sifter rubber-roll huller.
b. Paddy separator a. 0.9 to 1.2 tons per hour
c. Paddy husker b. 1.2 to 1.9 tons per hour
d. Rice whitener c. 2.2 to 3.8 tons per hour
e. None of the above d. None of the above

40. Capacity of a compartment-type 46. Throughput capacity of No. 6 rubber-


separator is ____. roll huller.
a. 20-30 kg. brown rice per hour a. 0.9 to 1.2 tons per hour
b. 40-60 kg. brown rice per hour b. 1.2 to 1.9 tons per hour
c. 70-80 kg. brown rice per hour c. 2.2 to 3.8 tons per hour
d. None of the above d. None of the above
47. Throughput capacity of No. 4 rubber 53. How many 50kg bags of paddy
roll huller. processed before a pair of No. 10 rubber-
a. 0.9 to 1.2 tons per hour roll huller is to be replaced?
b. 1.2 to 1.9 tons per hour a. 850 to 900 bags
c. 2.2 to 3.8 tons per hour b. 950 to 1000 bags
d. None of the above c. 1500 to 2000 bags
d. None of the above
48. Horsepower requirement of No. 4
rubber-roll huller. 54. Capacity of 750mm disc huller.
a. 2.5 hp a. 450 to 600 kg paddy per hour
b. 4.0 hp b. 700 to 1000 kg paddy per hour
c. 6.0 hp c. 1600 to 2100 kg paddy per hour
d. None of the above d. None of the above

55. Capacity of 1000mm disc huller.


49. Horsepower requirement of No. 6
a. 450 to 600 kg paddy per hour
rubber-roll huller.
b. 700 to 1000 kg paddy per hour
a. 2.5 hp
c. 1600 to 2100 kg paddy per hour
b. 4.0 hp
d. None of the above
c. 6.0 hp
d. None of the above 56. Capacity of 1400mm disc huller.
a. 450 to 600 kg paddy per hour
50. Horsepower requirement of No. 10 b. 700 to 1000 kg paddy per hour
rubber-roll huller. c. 1600 to 2100 kg paddy per hour
a. 2.5 hp d. None of the above
b. 4.0 hp
c. 6.0 hp 57. Horsepower requirement of the
d. None of the above 750mm disc huller.
a. 3.0 hp
51. How many 50kg bags of paddy b. 3.5 hp
processed before a pair of No. 6 rubber- c. 5.5 hp
roll huller is to be replaced? d. None of the above
a. 850 to 900 bags
58. Horsepower requirement of the
b. 950 to 1000 bags
1000mm disc huller.
c. 1500 to 2000 bags
a. 3.0 hp
d. None of the above
b. 3.5 hp
c. 5.5 hp
52. How many 50kg bags of paddy
d. None of the above
processed before a pair of No. 8 rubber-
roll huller is to be replaced? 59. Horsepower requirement of the
a. 850 to 900 bags 1400mm disc huller.
b. 950 to 1000 bags a. 3.0 hp
c. 1500 to 2000 bags b. 3.5 hp
d. None of the above c. 5.5 hp
d. None of the above
60. In a tray-type paddy separator, ____ 66. Husk aspirator is located after the
moves upward of the tray. ____.
a. brown rice a. precleaner
b. rough rice b. huller
c. milled rice c. paddy separator
d. All of the above d. whitener
e. polisher
61. In a try-type paddy separator, ____ f. None of the above
moves downward of the tray.
a. brown rice 67. Typically, the husk from the multiple-
b. rough Rice pass rice mill is collected at ____.
c. milled rice a. silo
d. None of the above b. bin
c. bag house
62. In a compartment-type paddy d. None of the above
separator, ____ moves downward of the 68. Milling recovery of the multiple-pass
compartment. rice mill is ____ single-pass rice mill.
a. brown rice
a. lesser than
b. rough Rice b. higher than
c. milled rice c. the same with
d. None of the above d. None of the above
63. In a compartment type paddy 69. Brown rice milling machine ____.
separator, ____ moves upward of the a. requires a whitener
compartment. b. requires a polisher
a. brown rice c. does not require a whitener and a
b. rough Rice polisher
c. milled rice d. All of the above
d. None of the above
70. Compartment-type paddy separator
64. In a multiple-pass rice mill, rough rice separates paddy and brown rice through
obtained from the separator directly goes ____ of the compartment assembly.
to the ____. a. vibration
a. whitener b. oscillation
b. huller c. upward-forward motion
c. polisher d. None of the above
d. None of the above 71. Tray-type paddy separator separates
brown rice from paddy through ____ of
65. The more the paddy passes the the trays.
husker, the ____. a. vibration
a. higher is the milling recovery b. oscillation
b. lesser is the milling recovery c. upward and forward motions
c. same is the milling recovery
d. None of the above
d. All of the above
72. A single-pass rice mill was tested for 76. A rubber-roll husker was tested to
30 min. and gave the following results: determine its performance. The husker
input paddy, 1000 kg; milled rice ran for 2 hours at a rate of 1000 kg of
produced, 650 kg; broken rice produced, input paddy. The weight of brown rice
150 kg; and weight of head rice, 200 kg. obtained after passing the husker was
What is the throughput capacity of the 805 kg. The weight of whole brown rice
rice mill? obtain per kg of sample was 900 grams.
a. 1000 kg milled rice per hour What is the hulling coefficient?
b. 1300 kg milled rice per hour a. 0.800
c. 1500 kg milled rice per hour b. 0.805
d. None of the above c. 0.905
d. None of the above
72. What is the milling recovery of the rice
mill in Item 72? 77. What is the wholeness coefficient of
a. 60% the hiller in Item 76?
b. 65% a. 0.90
c. 69% b. 0.94
d. None of the above c. 0.98
d. None of the above
73. What is the percentage broken grains
of the rice mill in Item 72? 78. Referring to Item 76, what is the
a. 20% hulling efficiency of the huller?
b. 23% a. 65%
c. 25% b. 72%
d. None of the above c. 85%
d. None of the above
74. Still referring to Item 72, what is the
percentage head rice recovery of the rice 79. What is the throughput capacity of
mill? the husker in Item 76, in kilogram per
a. 71% hour of brown rice?
b. 77 a. 402 kg/hr
c. 80% b. 457 kg/hr
d. None of the above c. 497 kg/hr
d. None of the above
75. A 5ton-per-hour rice mill plant is to be
designed. The design milling recovery
and hulling efficiency of the rice mill are
69% and 95%, respectively. How many
compartments are needed for the paddy
separator of the rice mill? Assume a 50kg
br/hr-comp.
a. 124 compartments
b. 138 compartments
c. 148 compartments
d. None of the above
XVI. FEED MILLING AND MIXING EQUIPMENT

Commonly Used Feed Milling Machines

1. Hammer Mill – reduction caused by


impact of metal hammers into the
grains being processed.

a. Fixed Hammer
b. Swinging Hammer

2. Burr Mill or Plate mill – reduction


caused by twisting pressure of a
rotating and stationary discs

Hammer Mill

• Consist essentially of a rotating beater


and a heavy-perforated screen.
• Material is introduced into the housing
and the beater that rotates 1500 to
4000 rpm beats and pounds the
material until it is small enough to pass
through the screen.
• Fineness of the material is controlled
mainly by the size of holes of the
screen.
• Hammers are rigidly fixed to the shaft
or to the swing.
• Striking edge of the hammer is
designed in great many ways.
• Swinging hammer is usually reversible.

Advantages of Hammer Mill Disadvantages of Hammer Mill

• Simplicity • Inability to produce uniform grind


• Versatility • High power requirement
• Freedom from significant damage due to
foreign objects
• Freedom from damage when operating
empty
• Hammer wear does not materially
reduce its efficiency
Burr, or Plate Mill Performance Characteristics

• Consists essentially of two roughened • Uniform product as to size


plates, one stationary and other • Minimum temperature rise during
rotating. reduction
• Material is fed between the plates and
• Minimum power requirement
is reduced by crushing and shearing.
• Over feeding reduces the • Trouble free operation
effectiveness of the grinder and
excessive heating results. Uniformity of the Product
• Operating speed is usually less than
1200 rpm. • Attrition mill does not significantly
• Fineness of reduction is controlled by produce a more uniform product than
the type of plates and by the spacing. hammer mill.
• Heavy–duty precision used for
preparation of feed and food.
• Each burr rotates and is driven
independently, speed is much higher, Product Size
and design and construction are more Hammer Varies the size of screen
precise. Mill opening
Burr Mill Varies spring pressure on
Advantages of Burr Mill the burrs

• Low initial cost


• Product may be relatively uniform Power Requirement
• Power requirement are very low
• Affected by the type of materials,
Disadvantages of Burr Mill moisture content, fineness of grinding,
rate of feeding, type and condition of
• Foreign objects may cause breakage mill, etc.
• Operating empty may cause excessive • Fibrous materials require more power
burr wear than crystalline materials.
• Worn burrs yield poor results. • Moist grains are difficult to grind than
dried grains.
• Power required to operate empty
increases swiftly as the speed of the mill
increases.
• Capacity of hammer mill is limited by
the power available.
Energy Requirement of Feed Grinding Material Temperature Rise During
(kW-hr/ton) Grinding

Material Hammer Mill Burr Mill Material Type of Fineness Temp


Grinder Modulus Elevation
Shelled Corn 7.4 3 – 5.8
Oats Burr 2.73 50
Oats 11.5 – 14 5 – 14 Hammer 2.70 18
Barley 9 – 14 4 – 10 Barley Burr 3.66 8
Ear Corn 6–8 - Hammer 3.66 2
Hay 8 - 16 - Shelled Burr 3.96 7
Corn
Hammer 3.13 10
Temperature Rise Ear Corn Burr 3.07 14
Hammer 3.05 13
• Energy for grinding is dissipated as
heat energy raises the temperature of
the ground product, the mill, and the
ambient air.
• Some heat is lost in vaporizing
moisture.
• Temperature may rise higher when
grinding fibrous materials, particularly
when producing fine materials.
• Hammer mill produces a cooler
product because it uses air for
conveying the materials.
• High temperature contributes to
decomposition of ground materials,
especially if the moisture content is
high.
Feed Mixers

• Mixing of materials is a very


extensive processing operation.
• Difficulty of mixing may result if the
solid has the same size but of
different specific gravity or if they
have different sizes or shapes.
• Heavier particles tend to remain near
the bottom of the container during
mixing.
• Round or small particles tend to
move toward the top of the
container.

Indicator of Satisfactory Mixing

• Produces uniform mixture


• Less time in mixing the materials
• Minimum cost for overhead, power,
and labor

Batch Mixer

• Used for moderate to small


operations where overhead cost must
be low and labor cost is not critical.
• Ingredients are weighed or measured.
• Can be adopted to semi-continuous
process by using a number of batch Horizontal Mixer - consists of a horizontal
mixers which empty into common cylindrical or U-shaped vessel in which a
conveyor or storage. shaft with mixing blades or paddles
• Concurrent filling, mixing, and rotates (approx. 25 rpm). It is filled from
emptying facilitate the use of labor the top and discharged through the valves
and equipment. in the bottom. Compared with the
vertical mixer the installed power is
relatively high. Mixing time is much
shorter in this type of mixer.
Advantages and Disadvantages of
Horizontal Mixer

• Short mixing time that results in high


capacity.
• Rapid discharge.
• Mixing of liquids is possible.
• Low building height.
• Incomplete cleanout when outlet
valves are small.
• Higher investment because of more
complex construction and higher
installed power.
• Filling level influences mixing
efficiency.

Continuous Mixer

• Most satisfactory for large, extensive Advantages and Disadvantages of


operations. Vertical Mixer
• Ingredients are usually added
volumetrically by either an auger, start • Relatively low installed motor power.
wheel, or other device to a screw • Low investment.
conveyor. • Flexible capacity utilization as the
• Continuous mixing operation is filling rate does not influence mixing
carried out in a screw conveyor that quality.
may have special flights to insure • Relatively long mixing time.
thorough mixing job. • Slow discharging which can reduce
feed mill capacity.
Vertical Mixer - consists of either • Relatively high building height
cylindrical, rectangular, or hexagonal bin required.
with a conical-shaped bottom. Inside
the vessel is a vertical screw-conveyor
which moves the material upward and
subsequently flows downward at the
wall of the vessel. Filling is on top or
through a screw conveyor into dump-
hopper at the bottom of the mixer. After
mixing the material is discharged
through an outlet at the bottom or at the
side of the vessel. Mixing time varies
from 15 minutes for normal feeds and 30
minutes for production of premixes.
REFERENCES

Henderson, S. M, and R. L. Perry. 1976.


Agricultural Process Engineering. Third
Edition. The AVI Publishing Company, Inc.
Westport Connecticut. 442pp.

Hunt, D. Farm Power and Machinery


Management. Iowa State University
Press. Ames, Iowa. 365pp
Questions

1. Raw materials for feeds are 5. Size-reduction machines are


processed primarily using a size- characterized based on ___.
reduction equipment to ___. a. capacity
a. increase the surface area of b. power required unit material
contact of the material to facilitate reduce
taking place of reaction c. size and shape of the product after
b. make it palatable when used as reduction
feed for animals d. range of size and shape of resulted
c. facilitate conveyance operation product
at the feed mill e. All of the above
f. All of the above
6. The power requirement in reducing
2. Size-reduction machines are the size of grains is ___ for moist
characterized based on ___. grains than for dried grains.
a. capacity a. lesser
b. power required unit material b. the same
reduce c. higher
c. size and shape of the product d. None of the above
after reduction
d. range of size and shape of 7. Performance characteristics of size-
resulted product reduction equipment are based on
e. All of the above ___.
a. size uniformity
3. The power requirement in b. temperature rise
reducing the size of grains is ___ for c. power requirement
moist grains than for dried grains. d. trouble-free operation
a. lesser e. All of the above
b. the same
c. higher 8. Hammer mills are designed
d. None of the above purposely to ___.
a. chop forage materials
4. A trade term used relative to the b. reduce the sizes of granular
reduction of grains into meal or products
flour. c. compact powdered product
a. Shredding d. None of the above
b. Crushing
c. Milling
d. None of the above
9. The size-reduction machine usually 14. The factor affecting the power
used in feed milling to break dried requirement of grinding mill.
grains into suitable size for feeds. a. Type of material
a. Hammer mill b. Moisture content of material
b. Roller mill c. Fineness of grinding of
c. Burr mill material
d. None of the above d. Rate of feeding the material
e. Type and condition of mill
10. In hammer milling, the fineness f. All of the above
of a material is determined by the
___. 15. The process of combining
a. number of hammers different materials until a certain
b. speed of hammer degree of homogeneity is
c. size of hole of the screen achieved.
d. None of the above a. Milling
b. Mixing
11. The energy requirement of a c. Feeding
hammer mill in grinding shelled corn. d. None of the above
a. 7.4 kw-hr/ton
b. 5.8 kw-hr/ton 16. The assembling and
c. 2.3 kw-hr/ton measuring of the needed amount
d. None of the above of every solid raw feed material in
the formulation of the desired
12. The speed of hammer mills. composition of a mixture.
a. 200 to 1000 rpm a. Mixing
b. 1500 to 4000 rpm b. Blending
c. 4500 to 6000 rpm c. Pelleting
d. None of the above d. All of the above

13. During grinding, the temperature 17. Basically, the purpose of


of the material ___ when grinding mixing is ___.
fibrous than granular material. a. to promote the transfer of
a. decreases heat
b. increases between hot and cold products
c. does not change b. to obtain good contact
d. None of the above between
materials being mixed
c. to promote reactions between
reactants
d. All of the above
e. Any two of the above
18. A mixer with U-shaped bin 23. The auger that is centrally
containing a central mixing blade or a located in a vertical feed mixer
ribbon mounted on a rotating shaft. and usually rotates at ___ speed.
a. Horizontal mixer a. 100-250 rpm
b. Inclined mixer b. 250-400 rpm
c. Vertical mixer c. 400-650 rpm
d. All of the above d. None of the above

19. A feed mixer characterized by high 24. The type of mixer suitable for
capacity, short mixing time, and high moderate to small operation in
power requirement. which overhead cost is low and
a. Horizontal mixer labor cost is not critical.
b. Inclined mixer a. Batch-type mixer
c. Vertical mixer b. Semi-continuous-type mixer
d. All of the above c. Continuous-type mixer
d. None of the above
20. Horizontal feed mixer is not
advantageous for feed mixing as
compared with vertical feed mixer for
the reason that ___.
a. it has lower capacity
b. it has high power requirement
c. it has shorter life span
d. None of the above

21. A feed mixer characterized by low


capacity, longer mixing time and low
power requirement.
a. Horizontal mixer
b. Inclined mixer
c. Vertical mixer
d. All of the above

22. A feed mixer with an auger that


elevates the feed to the top of the
mixing bin and spreads them evenly
throughout by gravity for another
mixing cycle.
a. Horizontal mixer
b. Vertical mixer
c. Inclined mixer
d. All of the above
XVII. CHOPPING, SHREDDING, AND
DECORTICATING EQUIPMENT

Size Reduction for Fibrous Materials

Cutting – separation or reduction by


pushing or forcing a thin, sharp knife
through the materials to be reduced.
Chipper usually operates by cutting action
of knife into the material being reduced.

Shearing – combination of cutting and


crushing. It is done either singly or in
combination. Chopper for reduced sizes
of forages usually operates in this
method.

Crushing – reduction by applying force to


the unit to be reduced in excess of its
strength. Shredders and decorticators
usually operate by crushing fibrous
materials.

Cutting - minimum deformation and Shearing - used for reducing size of


rupture of the reduced particles. New materials having tough fibrous nature
surfaces produces by sharp knife are where some crushing may be
relatively undamaged. Well adopted for advantageous and the resulting units are
fruits and vegetables. Knife of extreme of large uniform size. For best
sharpness and as thin as possible is the performance, the clearance between the
most satisfactory cutting device. Knife bar and the knife should be as small as
with edge that has sawing component possible and the knife as sharp and thin as
provides a smoother cut with probably possible.
less energy.
Crushing - characteristics of the new
surfaces are irregular in size and shape
that are dependent upon the type of
material and the method of force
application. Crushing by means of rigid
roll or bed are examples of static force
application. Hitting the product with
hammer is an example of dynamic force
application.

Forage Chopper - machine used to cut


the crop into short parallel length.

Classifications:

1. According to Feeding Mechanism


Madre de Cacao Twigs

a. Precision-Cut
b. Semi-precision Cut
c. Random Cut

2. According to Chopping Mechanism

a. Cylinder cutterhead
b. Flywheel cutterhead

3. According to Discharging Mechanism

a. Let-fall type
b. Throw-away
c. Blow-up type

Rice Straw
According to Feeding Mechanism According Chopping Mechanism

1. Precision-cut forage chopper – uses a 1. Cutterhead or cutting rotor – device


feeding mechanism consisting of four intended to cut the crop into short
or more feed rolls to partially orient lengths with reasonable consistency
and advance the crop at a consistent within the range of optional settings.
rate into the cutting mechanism
producing the shortest and most 2. Cylinder cutterhead – knives on
uniformly cut particles. cylindrical mountings such that the
cutting edges of the knives are
2. Semi-precision-cut forage chopper – essentially parallel to the axis of
uses a feeding mechanism consisting rotation.
of two feed rolls, or other means such
as an auger to advance the crop to the 3. Feedroll – cylindrical roll generally
cutting mechanism. Particle lengths with protrusions or flutes, used to
and uniformity are intermediate gather, compress and advance the
between those obtained with crop into the cutterhead.
precision and random-cut forage
choppers. 4. Stationary knife or chear bar – fixed
plate providing a stationary edge
3. Random-cut forage chopper - no against which the cutterhead knives
distinct mechanism, usually employing shear the crop.
flails to impact-cut and chop the crop
directly into shorter pieces. This type
usually produces the longest mean According to Discharging Mechanism
particle length and the least uniformly
cut particles. 1. Blow-up type – chopped materials are
blown up through the blow-up pipe.

2. Let-fall type – chopped materials are


dropped down to the bottom of the
machine.

3. Throw-away type – chopped


materials are thrown away to the
front area of the machine.
Parts of Forage Chopper

Cutter Mill - best for reducing the size of


fibrous materials like forages, straws, and
weeds.
USEFUL FORMULA

Fiber Recovery Extraction Efficiency

%Fr = Fp x 100 / Fw ξe = Wef x 100 / Wpf


where: where:
%Fr - percentage fiber recovery, % ξe - extraction efficiency, %
Fp - weight of the plant Wef - weight of dried extracted fiber, kg
Fw - weight of the decorticated fiber Wpf - weight of potential fiber content, kg
Output Capacity Extraction Loss
Wp
Co = ---------- ξ l = 1 - Wefx 100 / Wpf
TO where:
where: ξ l - extraction loss, %
Co - output capacity, kg/hr Wef - weight of dried extracted fiber, kg
Wp - weight of product, kg Wpf - weight of potential fiber content, kg
TO - time of operation, hr
Chopping Efficiency Amount of Unshredded Biomass Material

Wi - Wuc Wus + Wp
ξc = ---------------- x 100 A(Ubm) = ---------------- x To
Wi Tc
where: where:
ξc - chopping efficiency, kg/h A(Ubm) - amount of unshredded biomass, kg
Wuc - weight of uncut materials, kg Wus - weight of unshredded biomass
Wi - weight of input materials, kg materials, kg
Wps - weight of partially shredded biomass
materials, kg
Tc - collection time of sample from
output chute, hr
Percent Unshredded Biomass Material Shredding Efficiency

Wus + Wps ξs = 100 – Ubm


% (Ubm) = ---------------- x 100
Wi where:
where: ξs - shredding efficiency, %
Ubm - percent unshredded biomass Ubm - percent unshredded biomass materials,
materials, % %
Wus - weight of unshredded biomass
materials, kg
Wps - weight of partially shredded biomass
materials, kg
Wi - weight of total input biomass material,
kg
REFERENCES

Henderson, S. M, and R. L. Perry. 1976.


Agricultural Process Engineering. Third
Edition. The AVI Publishing Company, Inc.
Westport Connecticut. 442pp.

Hunt, D. Farm Power and Machinery


Management. Iowa State University
Press. Ames, Iowa. 365pp
QUESTIONS and PROBLEMS

1. Size reduction machine either power- 6. Machine used to cut biomass materials
or manually-operated which is used to cut into strips.
or slice root crops or banana into small a. Biomass stripper
thin pieces called chips. b. Biomass chopper
a. Shredder c. Biomass shredder
b. Slicer d. All of the above
c. Chipper
d. All of the above 7. Machine used to cut the crop into
short parallel lengths.
2. Amount of material that can be a. Chopper
processed by a chipping machine per unit b. Slicer
time. c. Cutter
a. Chipping throughput rate d. All of the above
b. Chipping capacity
c. Chipping efficiency 8. Forage chopper which uses a feeding
d. All of the above mechanism consisting of four or more
feed rolls to partially orient and advance
3. Device intended to slice the crop into the crop at a consistent rate into the
chips with reasonable consistency within a cutting mechanism.
range of optional settings. a. Precision-cut forage chopper
a. Cutter-head b. Random cut forage chopper
b. Chipping plate c. Semi-precision-cut forage chopper
c. Chipping disc d. None of the above
d. All of the above
9. Forage chopper which uses a feeding
4. Type of a chipper where knives on mechanism consisting of two feed rolls, or
cylindrical mountings are parallel with the other means such as an auger, to advance
axis of rotation of the cutter-head. the crop to the cutting mechanism.
a. Flywheel type a. Precision-cut forage chopper
b. Disc type b. Random cut forage chopper
c. Drum type c. Semi-precision-cut forage chopper
d. All of the above d. None of the above

5. Type of chipping machine with knives 10. Forage chopper without a distinct
mounted radially with the cutting edges feeding mechanism, usually employing
that are in perpendicular to the axis of flails to impact cut and chop crop directly
rotation of the cutter-head. into shorter pieces.
a. Flywheel type a. Precision-cut forage chopper
b. Disc type b. Random cut forage chopper
c. Drum type c. Semi-precision-cut forage chopper
d. All of the above d. None of the above
11. Type of forage chopper usually 16. Type of forage chopper where the
produces the longest mean particle chopped materials are dropped down the
lengths and the least uniformly bottom of the machine.
cut particles. a. Front-fall type
a. Precision-cut forage chopper b. Let-fall type
b. Random cut forage chopper c. Blow-up type
c. Semi-precision-cut forage chopper d. None of the above
d. None of the above
17. Type of forage chopper where the
12. Type of forage chopper where the chopped materials are forced by air-blast
mean particle lengths and particle through the duct or pipe outlet.
uniformity are intermediate between a. Front-fall type
those obtained with precision-cut and b. Let-fall type
random-cut forage choppers. c. Blow-up type
a. Precision-cut forage chopper d. None of the above
b. Random cut forage chopper
c. Semi-precision-cut forage chopper 18. Cylindrical roll generally with
d. None of the above protrusions or flutes used to gather,
compress and advance the crop into the
13. Type of forage chopper that is cutter-head.
capable of producing the shortest and a. Feed roll
most uniformly-cut particles. b. Crop roll gatherer
a. Precision-cut forage chopper c. Roller
b. Random cut forage chopper d. None of the above
c. Semi-precision-cut forage chopper
d. None of the above 19. Ratio of the weight of the fresh
chopped materials collected at all outlets
14. Fixed support providing a stationary to the total fresh weight of the input of
edge against which the cutter-head the chopper, expressed in percent.
knives shear the crop. a. Chipping efficiency
a. Shear bar b. Chopping efficiency
b. Shear plate c. Chopping index
c. Shear disc d. All of the above
d. All of the above
20. Machine used to crush matured
15. Type of forage chopper where the coconut husk through impact or beating
chopped materials are thrown away to action to separate coco fiber and coconut
the front area of the machine. peat from the husk.
a. Throw-away type a. Coconut crusher
b. Front-throw type b. Coconut mill
c. Outlet-throw type c. Coconut coir decorticator
d. None of the above d. All of the above
21. Part of the forage chopper where the 26. Ratio of fresh weight of fiber
forage to be chopped is loaded. extracted and the total fresh weight of
a. Feed tray input abaca tuxies to the abaca stripper,
b. Crop table expressed in percent.
c. Feeding table a. Potential fiber recovery
d. All of the above b. Product recovery
c. Fiber extraction recovery
22. Ratio of the weight of the input d. None of the above
coconut husk partially and totally
undecorticated coconut husk to the total 27. Mechanical device used for
weight of the input coconut husk to the extracting fibers by crushing, beating and
decorticator, expressed in percent. scraping actions effected by the rotating
a. Crushing efficiency cylinder with equally-spaced blades and
b. Decorticating efficiency breaker or scraper block.
c. Milling efficiency a. Fiber decorticator
d. All of the above b. Fiber crusher
c. Fiber shredder
23. Mechanical device used for extracting d. All of the above
primary fibers from abaca by scraping
action from blade and stripping block. 28. Part of the decorticator where raw
a. Abaca stripper materials are beaten up and crushed.
b. Abaca fiber decorticating machine a. Scraper block
c. Abaca fiber making machine b. Cutter-head
d. All of the above c. Shear bar
d. None of the above
24. Ratio of the total dry weight
(moisture content of 14%) of the fiber 29. Outside diameter generated by the
extracted to the potential fiber content outermost point of the cylinder
of abaca tuxies, expressed in percent. decorticating elements.
a. Extraction efficiency a. Effective cylinder diameter
b. Fiber efficiency b. Actual cylinder diameter
c. Stripping efficiency c. Rated cylinder diameter
d. All of the above d. All of the above

25. Ratio of the dry weight of fiber


extracted and total fresh weight of stalks
/ leaves, expressed in percent.
a. Fiber recovery
b. Fiber decortication efficiency
c. Fiber efficiency
d. None of the above
XVIII. WATER PUMPING EQUIPMENT

Pump - device used to lift or transfer


water from one source to the other. It
uses either reciprocating or rotational
motion to move water from a source to
delivery point like tanks, reservoir, or
directly to the field. Sources of power
maybe manual, animal, electricity, or
mechanical using fossil (gasoline, diesel,
etc.), or any form of renewable energy
available.

Applications

• To supply water for domestic and


group of households.
• To supply water for farm animal.
• To supply water for conveyance canal
by gravity irrigation.
• To provide pressure for sprinkler and
drip irrigation.
• To provide pressure in applying
chemicals into irrigation system.
• To supply water for fishponds and
remove it.
General Classifications of Pump

Power Drive for Water Pumps • Rotary – uses gears, vanes, lobe, or
screw to trap and convey fluid from inlet
• Electric motor to the outlet of the pump.
• Gasoline engine • Reciprocating – uses the back-and-forth
• Diesel engine motion of mechanical parts such as
• Human piston or diaphragms to pressurize the
• Animal fluid.
• Solar • Centrifugal – uses the centrifugal force
• Wind imparted to the fluid by one or more
rotating elements called impellers to
increase the kinetic energy and pressure
energy of the fluid.
Types of Pump Commonly Used for Types of Impeller for Centrifugal Pumps
Pumping Irrigation Water
• Open – used to pump water with
• Centrifugal Pump – pump with considerable amount of small solids.
impellers rotating inside a closed • Semi-Open or Semi-Enclosed – used to
casing which draws water into the pump water having some amount of
pump through a central inlet opening suspended sediments.
and forces water out through a • Enclosed – designed to pump clear
discharge outlet at the periphery of water.
the housing by means of centrifugal
force.
Types of Suction Inlet for Centrifugal
• Diffuser Pump – centrifugal pump Pumps
wherein the impeller is surrounded by
diffuser vanes. The diffuser vanes • Single-Action – has an impeller which
have small openings near the impeller has suction cavity on one side only.
and enlarge gradually to their outer • Double Action – has an impeller which
diameter where the water flows into has suction cavity on both sides.
the chamber and around to the pump
discharge. This is sometimes called a
turbine pump. Axis of Rotation of Centrifugal Pumps

• Volute Pump – centrifugal pump with • Horizontal – has a vertical impeller


a casing made in the form of a spiral mounted on a horizontal shaft.
or volute curve. The casing is • Vertical – has a horizontal impeller
proportioned to reduce gradually the mounted on a vertical shaft.
velocity of water as it flows from the
impeller to the discharge, thus
discharging velocity heat to pressure
head.
Methods of Priming Centrifugal Pumps

• Non-Self-Priming – centrifugal pump


that needs to be manually primed. The
system has to be filled initially by
pouring water into the pipes from a
bucket and thereafter the foot valve
will keep water in the system even
after the pump is not used for some
time.

• Self-Priming – centrifugal pump that


develops a vacuum sufficiently enough
for atmospheric pressure to force the
liquid to flow through the suction pipe
into the pump casing without priming
the pump.
Axial-Flow Pump - type of pump which
develops most of the suction and
discharges head by propelling or lifting
action of the impeller vanes on water.

Mixed-Flow Pump - type of pump which


combines some of the features of both
centrifugal and the axial-flow pump and
in which head is developed partly by the
centrifugal force and partly by the lift of
the vanes on the water.

Lift pump – type of pump that moves


water by the upward and downward
motions of the plunger. This type of
pump can be used for lifting water on a
deep well. They are commonly driven
manually or with the use of a windpump.
Pumps Terms

Capacity – amount of discharge of a


pump at maximum efficiency.

Discharge – volume of water pumped


per unit time.

Cavitation – formation of cavities filled


with water vapor due to local pressure
drop and collapse as soon as the vapor
bubbles reach regions of high pressure.

Priming – filling up of the pump with


water to displace or evacuate the
entrapped air through a vent and create
a liquid seal inside the casing.

Human and Animal Daily Water Requirement


Type Water Requirement
gpd m3 /day
Each household 50 - 250 0.189 - 0.945
(min)
Each horse 12 0.045
Each cow 40 0.151
producing milk
Each dry cow or 12 0.045
steer
Each hog 4 0.015
Each sheep 2 0.008
Each 100 6 0.023
chickens
Each 100 18 0.068
turkeys
Crop Water Requirement
Various Crops (mm/day)
Corn 6.15
Cabbage 3.75
Eggplant 4.76
Mungbean 5.33
Pechay 6.00
Soybean 5.23
Tomato 4.35
Water Melon 4.87

Head – amount of energy required to lift


and move the water from any arbitrary
datum.

• Friction Head - equivalent head


required to overcome the friction
caused by the flow through the pipe
and pipe fittings.
• Static Suction Head - vertical distance
from the free suction water level to the
center line of the pump.
• Static Discharge Head - vertical
distance from the centerline of the
pump to the discharge water level.
Overall System Efficiency of Water Pump
Pump Type Overall System
Efficiency
(%)
New electric motor 95
and water pump
Reconditioned pump 60
and motor units
New Engine and water 85
pump
Surplus Engine and 65
reconditioned pump

Pump Performance Curve - represents the


interrelationship between capacity, head,
power, NPSH, and efficiency of the pump.
Factors to Consider in the Selection of
Pump

• Head-Capacity of the Well


• Initial Cost
• Space requirement
• Type of power unit and pump
characteristics
• Storage Capacity, rate of
replenishment, and well diameter Factors to consider in the Selection of
• Other possible uses of pump Power Unit for Pumps

• Amount of power required


• Initial cost
• Availability and cost of energy fuel
• Duration and frequency of pumping
• Maintenance and convenience of
operation
• Durability and dependability of unit
• Labor availability and quality
USEFUL FORMULA
A. General
Engine Horsepower Water Horsepower
QH
EHP = BHP / ξm Pw = -------------
where: 102
EHP - engine horsepower, hp where:
BHP - pump brake horsepower, hp Pw - water horsepower, hp
ξm - engine efficiency, decimal 80% for Q - discharge, lps
diesel and 70% for gasoline H - total head, m
Total Pump Head Hydraulic Efficiency
H Q
Ht = Hs + (HLsp + HLf) ξh = -------------- x 100
where: P 33000
Ht - total head loss, ft where:
Hs - head loss due to elevation, ft ξh - hydraulic efficiency, %
HLsp - friction loss on straight pipe, ft H - head, ft
HLf - head loss on fittings, ft Q - mass flow rate, lb/min
P - power input, hp
Overall System Efficiency Input Power Delivered to Pump

ξos = ( Pw / MHP ) x 100 Pi = 9.8 q h / ξp


where: where:
ξos - overall system efficiency, % Pi - power input delivered to pump, kW
Pw - water horsepower, hp q - discharge rate, m3 /s
MHP - motor horsepower, hp h - total heat, m
ξp - pump efficiency, 0.20 to 0.75
Pump Brake Horsepower Pump Motor Horsepower

BHP = Pw / ξp MHP = BHP / ξm


where: where:
BHP - pump brake horsepower, hp MHP - motor horsepower, hp
Pw - water horsepower, hp BHP - pump brake horsepower, hp
ξp - pump efficiency, dec ξm - motor efficiency, dec
Pump Discharge Requirement Pump Specific Speed
A D
Q = 183.4 --------- Ns = C N q ½ / h ¾
F H where:
where: Ns - specific speed
Q - pump discharge requirement, gpm C - 51.65
A - design irrigable area, ha N - impeller speed, rpm
D - depth of irrigation, in. q - flow rate, m3 /s
F - number of days permitted for irrigation, h - head, m
day
H - average number of hours of operation,
hrs per day
B. Pump Laws
Capacity vs Speed and Diameter Head vs Speed and Diameter

q1 / q2 = (N1 / N2 ) (D1 3 / D2 3 ) H1 / H2 = (N1 2 / N2 2 ) (D1 2 / D2 2 )


where: where:
q1 - pump capacity, gpm H1 - pump head, ft
q2 - pump capacity, gpm H2 - pump head, ft
N1 - pump speed, rpm N1 - pump speed, rpm
N2 - pump speed, rpm N2 - pump speed, rpm
D1 - pump diameter, in. D1 - pump diameter, in.
D2 - pump diameter, in. D2 - pump diameter, in.
Horsepower vs Speed and Diameter Impeller Diameter vs Capacity

Hp1 / Hp2 = (N1 3 / N2 3 ) (D1 5 / D2 5 ) D1 3 / D 2 3 = q 1 / q 2


where: where:
Hp1 - pump power, hp D1 - pump diameter, in.
Hp2 - pump power, hp D2 - pump diameter, in.
N1 - pump speed, rpm q1 - pump capacity, gpm
N2 - pump speed, rpm q2 -pump capacity, gpm
D1 - pump diameter, in.
D2 - pump diameter, in.
Speed vs Capacity Speed vs Head

N1 /N2 = q1 /q2 N1 2 / N2 2 = H1 / H2
where: where:
N1 - pump speed, rpm N1 - pump speed, rpm
N2 - pump speed, rpm N2 - pump speed, rpm
q1 - pump capacity, gpm H1 - pump head, ft
q2 - pump capacity, gpm H2 - pump head, ft

Speed vs Power

N1 3 / N2 3 = Hp1 / Hp2
where:
N1 - pump speed, rpm
N2 - pump speed, rpm
Hp1 - pump head, ft
Hp2 - pump head, ft
SAMPLE PROBLEMS

Problem 1

One-thousand gallons of water is to be pumped from a river per minute (1000


gpm) to a farm 150 feet above the river. What is the required horsepower to drive
the pump, considering a 70% pump efficiency and 20% friction losses and static
head?

Given:
Pumping rate - 1000 gpm
Pumping head - 150 ft
Pump efficiency - 70%
Friction and other loss - 20% of the static head

Required: Pump Horsepower

Solution:

PHP = Q H γ /ξ [7.48 x 33000


= (1000 gpm)(150ftx 1.20)(62.4 lb/ft3 ) / 0.70 (7.48 gal/ft3 x 33000 ft-lb/min-hp
= 64.8 hp

Problem 2

If a diesel engine will be used to drive the pump in Problem 1, what is the
recommended horsepower for the engine? Engine efficiency is 80%.

Given:
Pump horsepower - 64.8 hp
Engine efficiency - 80%

Required: Engine Horsepower

Solution:

EHP = PHP / ξe
= 64.8 hp / 0.80
= 80.9 hp
Problem 3

Compute the friction head from a 6m-depth suction pipe of a centrifugal pump
which discharges 0.012 m3 /sec of water. The pipe nominal diameter is 4 inches.

Given:
Length of pipe - 6 m
Discharge - 0.012 m3 /sec
Pipe diameter - 4 in.

Required: Friction Head

Solution:
L x Q2
Hf = k
C2 x D 2

6 m x (0.012 m3 /sec)2
= 10
(1.0)2 x (4 in. x 2.54 cm/in. x m/100 cm)2

= 0.864 m

Problem 4

A pump was installed with 6m suction head and 10m discharge head. Its discharge
rate is 60 liters per minute. What is its water horsepower?

Given:
Suction head - 6 m
Discharge head - 10 m
Discharge rate - 60 lpm

Required: Water Horsepower

Solution:

WHP = TDH x Q / 102


= (6 m + 10 m) x (60 lpm x min/60sec) / 102
= 0.157 kW x Hp/0.746 kW
= 0.210 hp
Problem 5

A pump, which discharges 4 liters per second at a head of 6 meters, is driven by an


electric motor. The input current of the motor is 1.5 amp while the input voltage is
220 volt. What is the overall efficiency of the pump if the motor power factor is 0.98?

Given:
Pump discharge - 4 lps
Head - 6m
Input current - 1.5 amp
Input voltage - 220 volt
Power factor - 0.98
Required: Pump Overall Efficiency
Solution:
WHP = TDH x Q / 102
= (6 m) x ( 4 lps) / 102
= 0.235 kW
IP = V x I x cos θ
= 220 volt x 1.5 amp x 0.98
= 0.323 kW
ξp = (WHP / IP) x 100
= (0.235 kW / 0.323 kW) x 100
= 72.7 %

Problem 6

A pump was tested for flow rate measurement using a 90-degree triangular weir.
Tests have shown that the head of water into the weir averages at 13 cm. What is
the average rate of flow of the pump?

Given:
Machine - pump
Test rig - 90 deg triangular weir
Ave head reading - 15 cm
Required: Pump Flow Rate
Solution:
Q = 0.0138 H 5/2
= 0.0138 x (15 cm) 5/2
= 12.02 lps
Problem 7

At 1200 rpm, a rotary pump is used to deliver water at 6m head. The rate of flow of
the pump is 0.01 m3 /sec. What is the fluid and brake horsepower of the pump shaft
if it has a 75% efficiency? What is the motor horsepower if the transmission and
motor efficiencies are 91% and 99%, respectively?

Given:
Flow rate - 0.1 m3 /s
Total head - 6m
Specific gravity of milk - 1.1
Pump efficiency - 75%
Transmission efficiency - 91%
Motor Efficiency - 99%

Required: Fluid Horsepower


Brake Horsepower
Motor horsepower

Solution:

FHP = QH γ
= (0.1 m3 /s ) ( 6 m ) (1.1 x 1000 kg/m3 )
= 76.2 kg-m/sec-hp
= 0.87 hp

BHP = FHP / ξp
= 0.87 hp / 0.75
= 1.16 hp

MHP = BHP / (ξt x ξm)


= 1.16 hp / ( 0.91 x 0.99)
= 1.29 hp therefore use 1-1/2 hp motor
Problem 8

What will be the flow rate, the working head, and the power requirement of the
pump system in Problem 7 if its speed is increased to 2100 rpm?

Given:
Speed1 - 1200 rpm
Flow rate - 0.1 m3 /s
Head1 - 6m
Horsepower 1 - 0.87 hp
Speed2 - 2100 rpm

Required: Flow Rate (Q2 )


Working Head (H2 )
Power Requirement (Hp2 )

Solution:

Q2 = (N2 /N1 ) x Q1
= (2100 rpm/1200 rpm) 0.1m3 /s
= 0.0175 m3 /s

H2 = (N2 /N1 )2 x H1
= (2100 rpm/1200 rpm)2 x 6 m
= 18.4 m

HP2 = (N2 /N1 )3 x HP1


= (2100 rpm/1200 rpm)3 x 0.87 hp
= 4.66 hp
Problem 9

A 4in.-diameter, 6in.-stroke lift pump is used to


lift water from a 20ft well and to deliver the
water to a cylindrical tank at 12 ft height. If the
volumetric efficiency of the pump at 10 lifting
strokes per minute is 90%, what is the pump
capacity? Also, compute for the power required
to operate the pump manually if its mechanical
efficiency is 80%? How long will the pump be
required to operate to fully fill the 600liter tank
if its operating efficiency is 70%?

Given:
Pump diameter -
4 in.
Pump stroke -
6 in.
Lifting height -
20 ft
Delivery height -
12 ft
Pump speed -
10 lifting stroke
per min
Volumetric efficiency - 90%
Mechanical efficiency - 80%
Operating efficiency - 70%

Required:
Pump Capacity
Power to Lift
Time to Fill the Tank

Solution:

Pump Capacity = (π/4) (4 in)2 x 6 in. x 0.90 x 10stroke/min


= 68.75 in3 /min x (2.54 cm/in)3
= 11 26.6 cm3 / min x li/1000 cc
= 1.126 li/min
Power to Lift = (1.126 lpm x m3 /1000 l x 60 m/hr) (32 ft x m/3.28 ft ) / 273 x 0.80
= 0.003 hp
Time to Fill Tank = 600 li / (1.126 li/min x 0.70)
= 12.68 hrs
Problem 10

Design a water pump system required to


supply water to a poultry farm with 10,000
heads of broiler. The source of water is 20
feet below the ground and is required to be
elevated to a 30feet-high water tank. The
specific water requirement of the broiler is
0.025 m3 /day-head. Assume a 20% friction
head loss from the suction and discharge
head loss. Pump operating time is 20 hrs
per day.

Given:
No. of broilers - 10,000 heads
Hs - 20 ft
Hd - 30 ft
SWR - 0.025 m3 /day-chicken
Friction Head - 20% of the Hs & Hd
Operating time - 20 hr per day

Required:
Pump Flow Rate
Total Head
Water Hp, Brake Hp and Motor Hp

Solution:

Pump Flow Rate = 0.025 m3 /day-head x 10,000 heads x 1 day/20 hrs


= 12.5 m3 /hr
Total Head = (20 ft + 30 ft) (1.2)
= 60 ft / (3.28 ft/m)
= 18.3 m
Water Hp = 12.5 m3 /hr (18.3 m) /273
= 0.84 hp
Brake Hp = 0.84 hp / 0.60
= 1.4 hp
Motor Hp = 1.4 hp/ (0.95 x 0.8)
= 1.84 hp ∴ use 2-hp electric motor
Problem 11

Design a water pump system required to


supply water to a 5-hectare rice farm. The
source of water is from a river 10 feet
below, and discharges water 2 ft above the
ground surface. Consider 25% friction head
from the suction and from the discharge
head of the piping system. Assume a crop
water requirement for rice of 10 mm/day
and 2 mm/day losses due to percolation
and seepage. Conveyance losses is 10% of
the irrigation requirement. Assume a 60%
pump efficiency, an 80% transmission
efficiency, and an 80% engine efficiency.
Operating time per day is 8 hrs.
Given:
Area of the farm - 5 has
Hs - 10 ft
Hd - 2 ft
Hf - 25% of Hs & Hd
Crop Water Requirement - 10 mm/day
Seepage & percolation - 2 mm/day
Conveyance losses - 10% of IR

Required:
Pump Capacity, Total Head Loss
Water Hp, Brake Hp, & Motor Hp

Solution:
Irrigation Requirement = (10 mm/da + 2 mm/day)/(1000 mm/m) 50,000 m2
= 600 m3 /day
Diversion Requirement = (600 m3 /day ) 1.1 = 660 m3 /day

Pump Discharge Requirement = (660 m3 /day)(day/8 hrs) = 82.5 m3 /hr


Total Head = (10 ft + 2 ft)(1.25)/(3.28 ft/m) = 4.5 m
Water Hp = 82.5 m3 /hr x 4.5 m / 273 = 1.36 hp
Brake Hp = 1.36 / 0.6 = 2.26 hp
Engine Hp = 2.26 hp / 0.8 /0.8 = 3.5 hp use 5 hp engine
REFERENCES

James, L. 1988. Principles of Farm


Irrigation System Design. John Wiley &
Sons. USA. 543pp.

PAES. 2001. Centrifugal Pump-


Specifications. Philippine Agricultural
Engineering standards Volume I. AMTEC,
CEAT, UPLB, College, Laguna. A-142 to A-
148.

PAES. 2001. Centrifugal, Mixed Flow, and


Axial-Flow Pump-Specifications. Philippine
Agricultural Engineering standards Volume
I. AMTEC, CEAT, UPLB, College, Laguna. A-
150 to A-168.
Questions and Problems
1. A device used to lift or to transfer water 7. A type of pump that uses the
from one source to another. centrifugal force imparted to the fluid by
a. Pump one or more rotating elements called
b. Wind turbine impellers to increase the kinetic energy
c. Electric motor and the pressure energy of the fluid.
d. None of the above a. Centrifugal
b. Reciprocating
2. A pump is used to supply water ___. c. Axial
a. for domestic and group of households d. None of the above
b. for farm animals
c. for conveyance canal by gravity 8. A type of pump that develops most of
irrigation the suction and of the discharge head by
propelling or by the lifting action of the
d. All of the above
impeller vanes in the water.
3. Pumps are also used to ___. a. Centrifugal
a. provide pressure for sprinkler and drip b. Mixed-flow
irrigation c. Axial-flow
b. provide pressure in applying chemicals d. None of the above
into irrigation system
9. A type of pump characterized by the
c. All of the above combination of features of centrifugal
d. None of the above
and axial pumps in which head is
4. Which of the following can be used as a developed partly by the centrifugal force
power drive for water pumps? and partly by the lift of the vanes in the
a. Electric motor, human and/or animal water.
b. Gasoline or diesel engine a. Self-priming pump
c. Solar and wind b. Diffuser-type centrifugal pump
d. All of the above c. Mixed-flow pump
d. None of the above
5. The type of pump commonly driven
manually or with the use of a windpump in 10. A type of pump that uses gears,
drawing water from a deep well. vanes, lobe or screw to trap and to
a. Centrifugal pump convey the fluid from the inlet to the
b. Lift pump outlet of the pump.
c. Diffuser pump a. Centrifugal
d. None of the above b. Reciprocating
c. Rotary
6. A type of pump that uses the back-and- d. None of the above
forth motion of mechanical parts, such as
piston or diaphragms, to pressurize the 11. The type of pump commonly used for
fluid. pumping irrigation water.
a. Centrifugal a. Centrifugal pump
b. Reciprocating b. Diffuser pump
c. Axial c. Volute pump
d. None of the above d. All of the above
12. A type of pump with impellers rotating 17. The diffuser vanes have ___ near the
inside a closed casing which draws water impeller and are gradually enlarged
into the pump through a central inlet toward their outer diameter where the
opening and forces water out through a water flows into the chamber and
discharge outlet at the periphery of the around the pump discharge.
housing by centrifugal force. a. big openings
a. Diffuser pump b. medium-size openings
b. Centrifugal pump c. small openings
c. Volute pump d. None of the above
d. None of the above
18. The type of a centrifugal pump with
13. The type of pump that moves water by spiral-curve casing.
the upward and downward motions of the a. Diffuser pump
plunger. b. Volute pump
a. Axial-flow pump c. Self-priming pump
b. Mixed-flow pump d. None of the above
c. Lift pump 19. The casing of the volute pump is
d. None of the above proportioned to gradually reduce the ___
as it flows from the impeller to the
14. A lift pump is to be installed to provide discharge, thus discharging velocity heat
water for a village using a 4-in casing. If the to the pressure head.
source of water is 12 meters below the a. velocity of water
ground surface, what would you b. pressure of water
recommend? c. volume of water
a. Install the pump on ground surface. d. None of the above
b. Install the pump down below the water
surface. 20. The type of impeller for centrifugal
c. Replace the lift pump with gear pump. pumps.
d. None of the above a. Open
b. Semi-open or Semi-enclosed
15. The type of centrifugal pump in which c. Enclosed
its impeller is surrounded by a set of d. All of the above
vanes.
21. Which of the following statements is
a. Diffuser pump true?
b. Centrifugal pump a. Open-type impeller is designed to
c. Volute pump pump clear water.
d. None of the above b. Semi-open or semi-enclosed is used
to pump water with some amount of
16. This type of pump is sometimes called suspended sediments.
a turbine pump. c. Enclosed-type impeller is used to
a. Diffuser pump pump water with considerable amount
b. Volute pump of small solids.
c. Self-priming pump
d. All of the above
d. None of the above
22. The type of suction inlet for centrifugal 27. A centrifugal pump that develops
pumps. vacuum enough for the atmospheric
a. Single action pressure to force the liquid through the
b. Double action suction pipe into the pump casing,
c. All of the above without initially feeding the pump to
d. None of the above create suction.
a. Pressure pump
23. The type of suction inlet for which its b. Self-priming pump
impeller has suction cavity on both sides. c. Mixed-flow pump
a. Single-action d. None of the above
b. Double-action
c. Any of the above 28. The amount of discharge of a pump
d. None of the above at maximum efficiency.
a. Head
24. The axis of rotation of centrifugal b. Cavitation
pumps that has a vertical impeller c. Capacity
mounted on a horizontal shaft. d. None of the above
a. Horizontal axis
b. Vertical axis 29. The formation of cavities filled with
c. Any of a & b water vapor due to local pressure drop
d. None of the above which collapses as soon as the vapor
bubbles reach high pressure region.
25. The filling up of the pump with water a. Bubbling
to displace or to evacuate the entrapped b. Cavitation
air through a vent and create a liquid seal c. Water hammer
inside the casing. d. None of the above
a. Feeding
b. Recharging 30. The amount of energy required to lift
c. Priming and to move the water from any arbitrary
d. None of the above datum.
a. Head
26. A centrifugal pump that needs to be b. Friction head
initially filled with water by pouring it into c. Static suction head
the pipes using a bucket and thereafter the d. None of the above
foot valve will keep water in the system
even after the pump is not used for quite 31. The volume of water pumped per
some time. unit time.
a. Non-selfpriming pump a. Capacity
b. Self-priming pump b. Discharge
c. Mixed-flow pump c. Static discharge head
d. None of the above d. None of the above
32. A pump was tested to measure the 37. The theoretical power required by
flow rate using a 90-degree triangular weir. the pump in pumping water.
Tests have shown that the height of water a. Brake horsepower
from the weir notch averages at 13 cm. b. Pump efficiency
What is the average rate of flow of the c. Water horsepower
pump? (Problem 6) d. None of the above
a. 8.4 li/s (12.02 li/s)
b. 22.2 li/s 38. The power required to drive the
c. 32.2 li/s pump shaft.
d. None of the above a. Brake horsepower
b. Pump efficiency
33. The equivalent head required to c. Water horsepower
overcome the friction caused by the flow d. None of the above
through the pipe and pipe fittings.
a. Friction head 39. The ratio of the power output to the
b. Static suction head power input of the pump.
c. Static discharge head a. Brake horsepower
d. None of the above b. Pump efficiency
c. Water horsepower
34. The vertical distance from the d. None of the above
centerline of the pump to the discharge
40. The power requirement of a 4-inch
water level.
shallow-tube-well pump.
a. Friction head
a. 10-hp diesel engine
b. Static suction head
b. 5-hp gasoline engine
c. Static discharge head
c. 3-hp diesel engine
d. None of the above
d. None of the above
35. The vertical distance from the free 41. A pump was installed with 6-meters
suction water level to the centerline of the suction head and 10-meters discharge
pump. head. Its discharge rate is 60 liters per
a. Friction head minute. What is the water horsepower of
b. Static suction head the pump? (Problem 4)
c. Static discharge head a. 0.210 hp
d. None of the above b. 0.120 hp
c. 0.210 kW
36. Compute the friction head of a 6meter-
d. None of the above
depth suction pipe of a centrifugal pump
that discharges water at 0.2 m3 /sec. The 42. This illustrates the interrelationship
pipe nominal diameter is 4 inches. between capacity, head, power, NPSH,
(Problem 3) and efficiency of the pump.
a. 0.864 m per 1000 meters length of pipe a. Pump characteristics
b. 0.864 m per 100 meter length of pipe b. Performance curve
c. 0.864 m c. Pump laws
d. None of the above d. None of the above
43. A water pump with 0.2meter- diameter 48. Still referring to Item 43, what is its
propeller discharges 60 lpm of water at a motor horsepower?
head of 5-meter depth suction and 1- a. 0.028 hp
meter discharge. It is driven by an electric b. 0.039 Hp
motor at 1720 rpm. The pump and the c. 0.041 hp
motor efficiencies are 72% and 90%, d. None of the above
respectively. Compute the water
49. One-thousand gallons of water is to
horsepower (WHP).
be pumped from a river per minute
a. 0.05 hp
(1000 gpm) to a farm 150 feet above the
b. 0.08 hp
river. What is the required horsepower
c. 0.09 hp
to drive the pump, considering a 70%
d. None of the above
pump efficiency and 20% friction losses
and static head? (Problem 1)
44. What is the brake horsepower (BHP) of
a. 64.8 hp
the water pump in Item 43?
b. 68.4 hp
a. 0.11 hp
c. 48.6 hp
b. 0.12 hp
d. None of the above
c. 0.13 hp
d. None of the above 50. What is the total head loss, in
meters, of the pump system in Item 49?
45. Referring to Item 43, what is its motor (Problem 1)
horsepower (MHP)? a. 54.8 m
a. 0.12 hp b. 48.5 m
b. 0.13 hp c. 84.5 m
c. 0.14 hp d. None of the above
d. None of the above
51. If a gasoline engine will be used to
46. If the speed of the pump in Item 43 is drive the pump in Item 49, what is the
to be reduced to 1200 rpm, what would be recommended horsepower for the
its capacity? engine? Engine efficiency is 80%.
a. 39 lpm (Problem 1)
b. 41 lpm a. 80.1 hp
c. 43 lpm b. 85.5 hp
d. None of the above c. 60.8 hp
d. None of the above
47. What is the working head of the pump
52. If a diesel engine will be used to drive
in Item 43?
the pump in Item 49 above, what is the
a. 2.85 m
power requirement? Engine efficiency is
b. 2.78 m
70%. (Problem 2)
c. 2.63 m
a. 92.5 hp
d. None of the above
b. 102.6 hp
c. 125.7 hp
d. None of the above
53. The overall system efficiency of the 59. The daily water requirement of each
water pump with new electric motor as milk-producing and dry cow is ___,
drive is ___. respectively.
a. 60% a. 0.151 and 0.045 m3 /day
b. 80% b. 0.151 and 0.023 m3 /day
c. 95% c. 0.151 and 0.008 m3 /day
d. None of the above d. None of the above
54. A pump that discharges 4 liters of 60. The recommended material for the
water per second at 6-meters head is manufacture of a drilling rig.
driven by a singe-phase electric motor. a. GI pipe schedule 20
The input current of the motor is 1.5 amp b. BI pipe schedule 40
while the input voltage is 220 volt. What is c. Stainless steel pipe schedule 10
the overall efficiency of the pump if the d. None of the above
motor power factor is 0.98? (Problem 5)
a. 71.2% 61. A device attached at the end of the
b. 72.7% drilling pipe that directly comes in
c. 77.1% contact with the soil formation and
d. None of the above serves as a cutting device during drilling.
a. Drilling pipe
55. The water requirement of corn is ___. b. Drilling rig
a. 3.75 mm/day c. Drilling bit
b. 5.33 mm/day d. None of the above
c. 6.15 mm/day
d. None of the above 62. A structural assembly used to drill
holes for water-well construction.
56. The water requirement of cabbage is a. Drilling pipe
___. b. Drilling rig
a. 3.75 mm/day c. Drilling bit
b. 4.35 mm/day d. None of the above
c. 5.33 mm/day
d. None of the above 63. This material serves as an adaptor for
the drill bit and for the conduit of water
57. The daily water requirement of a jet channel of the drilling rig.
minimum household-size averages at ___. a. Drilling pipe
a. 0.068 m3 /day b. Drilling rig
b. 0.045 m3 /day c. Drilling bit
c. 0.567 m3 /day d. None of the above
d. None of the above
64. A light-weight pipe used during the
58. The water requirement of mungbean high-velocity-flow operation of the
and soybean is ___, respectively. drilling rig.
a. 4.76 and 4.35 mm/day a. Injector
b. 5.33 and 5.23 mm/day b. Surging stem
c. 6.15 and 6.00 mm/day c. Rig jet
d. None of the above d. None of the above
65. The recommended material for the 70. At 1200 rpm, a rotary pump is used
manufacture
. of drill bit of the drilling rig. to deliver water at 6-meters head. The
a. Carbon steel with at least 80% carbon rate of flow of the pump is 0.01 m3 /sec.
content What is the fluid horsepower?
b. High-speed steel with molybdenum (Problem 7)
c. All of the Above a. 0.68 hp
d. None of the above b. 0.74 hp
c. 0.87 hp
66. A farmer has a pump with 100-lpm d. None of the above
water discharge at 10-m head. The pump is
driven by an electric motor at a speed of 71. What is the brake horsepower of
1700 rpm through a 4” x 4” pulley-drive the pump shaft if it has a 75%
combination for the motor and for the efficiency? (Problem 7)
pump, respectively. What would be the a. 1.16 hp
pump output if the motor pulley is changed b. 1.45 hp
to 6-inches diameter? c. 2.12 hp
a. 145 lpm d. None of the above
b. 150 lpm
c. 155 lpm 72. What is the motor horsepower if
d. None of the above the transmission and motor efficiencies
are 91% and 99%, respectively?
67. The size of a diesel engine commonly (Problem 7)
used for 4-in. STW pump. a. 1.02 hp
a. 8 hp b. 1.29 hp
b. 12 hp c. 2.10 hp
c. 16 hp d. None of the above
d. None of the above
73. If the speed of the pump above is
68. Fuel consumption rate of diesel engine increased to 2100 rpm? What will be
in pumping water for STW pump. the flow rate of the pump? (Problem 8)
a. 0.4 to 0.8 liter per hour a. 0.0175 m3 /s
b. 0.8 to 1 liter per hour b. 0.0125 m3 /s
c. 1.0 to 1.2 liter per hour c. 0.0156 m3 /s
d. None of the above d. None of the above

69. Throughput capacity of an STW pump in 74. What will be the working head of
rice irrigation. the pump above? (Problem 8)
a. 5 lps for 24 hours per ha a. 17.4 m
b. 10 lps for 24 hours per ha b. 18.4 m
c. 15 lps for 24 hours per ha c. 19.3 m
d. None of the above d. None of the above
75. What is the horsepower
requirement of the pump above?
(Problem 8)
a. 3.56 hp
b. 4.01 hp
c. 4.66 hp
d. None of the above

76. A 4inches-diameter, 6inches-


stroke lift pump is used to lift water
from a 20-ft well and to deliver the
water to a cylindrical tank at 12 ft
height. If the volumetric efficiency
of the pump at 10 lifting strokes per
minute is 90%, what is the pump
capacity? Also, compute for the
power required to operate the pump
manually if its mechanical efficiency
is 80%? How long will the pump be
required to operate to fully fill the
600-liter tank if its operating
efficiency is 70%?
XIX. SELECTION OF AGRICULTURAL MACHINES

Introduction Trade Name - name by which an article is


n There is a need to understand some called or the name given by a
more additional items in the careful manufacturer to an article to distinguish it
selection of agricultural machinery. as one produced by the company. It is
n The following are important factors to also called as brand name.
judge and influence in the selection of n Trade or brand name can be registered
agricultural machines: to have statutory protection in a
manner similar to trademark.
Trademark - distinguishing mark, device, n It can only be found in only one
or symbol affixed by a manufacturer, particular machine. It is presently
merchant, or trader to his goods in order abandoned in favor of model
to identify them as his goods and designation.
distinguish them from the goods
manufactured, sold, or dealt in by others.
n Trademark gives statutory protection
when they are registered according to
law which gives the owner the
exclusive right to use its product.
n Trademark provides guarantee of
reliability of the machine since the
company will stand behind its product
or machine.

Model - indicates the type, size,


improvement, or new design of an old
machine, a special purpose or
combination of one or more of machine
features.
n Model designation is presented by
series of numbers or letters or
combination of both. Example, a
combine model SP-12 is a self propelled
combine having a width cut of 12 ft.
Repair

n Before considering a purchase of the


machine, it is well to look into the
source of repairs.
n Can the repair be made at hand or it
need somebody to send from a
distance?
n Machine breakdown can cause delay
and lost of crops.
n Some manufacturers provide quick
service for the machinery they
supplied.
n Be sure that repair and supply of
machine parts are readily available in
the locality. Information Needed in Making Order for
Repair of Parts
Technical Manual – booklet provided by
n Name and address of manufacturer
manufacturers in which the step-by-step
procedure for repair or disassembly of n Brand name, model number, and year
parts are given. made or purchased
n Number of parts wanted
n If the number of the parts cannot be
determined, get the numbers of the
parts with which it works.

Keep the pamphlets that are furnished


with the machine especially the one
containing the list of repair parts. When
repair parts are needed, the part
numbers and their description will be
found in the lists parts.

Operator’s Manual – booklet form


provided by manufacturers in which
adjustment, care and maintenance
procedure, and safety of the machine are
indicated.
Design - arrangement of the parts to Ease of Adjustment
show the difference in the make up of
the machine from those of the same n Careful study should be made on the
type. methods of adjusting the various parts
n Gears should run in a sealed oil bath. of the machine.
n Devices designed to simplify adjusting
n Provision of safety devices should be
the equipment are time and labor
carefully considered.
savers.
n The owner ’s or operator ’s manual
should be studied thoroughly to
understand the method of adjusting the
equipment as planned by the designers
and the test engineer.
n Many operators of agricultural machines
are not inclined to take sufficient time to
make needed slight adjustments.

Does the machine have finished


appearance and style without sacrificing
strength and performance?

Ease of Operation
n Many implements that look well are
fond to require an unnecessary
amount of power and labor to make
them operate successfully.
n A dealer who is a good one will usually
take the machine out, have the farmer
go along, make necessary adjustments
and see that the machine is running
perfectly before turning it over to the
farmer.
Adaptability to Work and Conditions Comfort

n There are many implements on the n As the operator of power equipment


market which are not adaptable to must spend days upon days riding
every conditions. upon it, the comfort and safety of the
n A machine may work in one locality seat should be considered.
and be an absolute failure in another n A comfortable seat should be
because it is adapted to a certain soil supported with shock-absorbing
conditions or type of crop grown. devices. The seat should be stable and
adjustable to suit different-sized
Quick Changes of Units individuals.
n Cabs give protection from cold, heat,
n The time and labor required to dust, and excessive noise.
dismount one unit and mount
another unit are important
considerations in selecting
agricultural equipment.
n Some lines of equipment are built in
unit packages and are designed so
that changes can be made in a few
minutes from a plow to a planter,
from planter to cultivator, or from
one unit to another for all the unit
supplied.

Maneuverability Other Factors

n As a general rule, tractor-mounted • Power requirements


equipment are provided with power • Cost of operation
and hydraulic lifts. The units can be • Initial cost
lifted and the tractor maneuvered • Years of service expected
almost as though no equipment was • Whether the purchase of the
attached. equipment is economical in relation to
n When trailing equipment is attached the size of the farm and work to be
to the drawbar of a tractor, turns performed by the equipment
cannot be made so sharply as with
mounted equipment.
REFERENCES

n Smith, H. P. and L.H. Wilkes. 1976.


Farm Machinery and Equipment. 6th
Edition. Tata McGraw-Hill Publishing
Company Ltd. New Delhi. 488pp.
QUESTIONS and PROBLEMS

1. Authorized dealer of a distributor or 5. Service offered to customers by


manufacturer to supply, trade, sell, and manufacturers in repairing and replacing
service agricultural machinery to end unintentionally-damaged part of a
users. machine for a specified period.
a. Supplier a. Manufacturing service
b. Dealer b. Customer ’s service
c. Service provider c. After-sales-service
d. None of the above d. All of the above

2. Which of the following is not covered 6. After 4 months of continuous use, the
under warranty of agricultural machines? pillow-block bearing of a brand new rice
a. Accidental or natural disaster. thresher was heavily worn out. As an
b. Improper operation and maintenance Agricultural Engineer, what would you
of the machine. advise to the farmer-owner?
c. Unauthorized repair and/or use of a. Bring the bearing to the manufacturer
non-genuine parts. of the thresher for replacement .
d. All of the above b. Buy a new bearing and replace it.
e. None of the above c. Ask the manufacturer to pay for the
bearing unit.
3. Based on the Philippine Agricultural d. None of the above
Engineers Standard (PAES), warranty for
parts and services of a heated-air 7. The handle of a power tiller was
mechanical grain dryer is within ____ detached from the main frame assembly
after the acceptance of the machine by due to a tremendous lateral load during
the buyer. plowing. The machine was purchased 4
a. 12 months months ago from a nearby manufacturer.
b. 18 months What would you recommend to the
c. 24 months farmer?
d. None of the above a. Buy new handle and replace the old
one.
4. According to PAES, warranty for the b. Consult the manufacturer and request
construction and durability of mechanical for replacement of the handle.
rice thresher shall be within ____ after c. Bring a welding machine and weld the
the purchase of the machine. handle to the frame of the power tiller.
a. 6 months d. None of the above
b. 12 months
c. 18 months
d. None of the above
8. Assurance of the quality of materials
and of workmanship of the product
offered for sale or the length of
satisfactory use expected from a product
under normal use.
a. After-sales service
b. Warranty
c. Insurance
d. None of the above
XX. SAFETY IN AGRICULTURAL MACHINES

n Operation of agricultural equipment Operator’s Responsibility for Safety


has many hazards, and accidents can
be reduced if the safety precautions • Avoid climbing over running machines.
are carefully followed. • Stay away from loose banks and ditches.
n “Safety first” is a slogan that should • Do not get in front of moving machines
be kept in mind at all times. to make adjustments.
n Many of the accidents that occur in • Check haying machines for unsafe rope,
operating agricultural equipment are pulley, etc.
the direct results of thoughtlessness • Always keep shields and guards over
and carelessness. power shafts, chains and gears.
n Accidents are more likely to occur • Keep doors of building open when
with power-operated equipment that running gasoline equipment inside.
has moving parts to perform its • Be sure tractor and other are out of gear
operating function. before starting.
• Do not attempt to put on or take off
belts from power running pulleys.
• Check hazards and be careful at all times.
• When pulling heavy objects, hitch under
center of the tractor.
• When driving on highways, use adequate
signal lights at night and red flags by day.
• Keep extra riders off moving tractors and
vehicles.
• Prevent upset and overturns by careful
driving up and down and across steep
slopes.
• Hold speed within safe limits of 4.5 mph.
• Stop corn picker and other machines
before attempting to unchoke them.
• Keep children away from machinery both
stationary and moving.
• Avoid loose clothing and ragged gloves
around moving machinery.
Cabs for Farm Equipment Safety Emblem

n Cabs serves as safety devices on n The National Safety Council of US


tractors and it should be reinforced recommends that all slow-moving
with a roll bar or a roll-on-protective vehicles operated on highways should
structure. have a safety emblem attached to the
n Strong cabs protect the operator from equipment in a manner so that the
being crushed in case a tractor rolls emblem can be easily seen.
over. It also protects the operators n The emblem is triangular in shape. The
from dusts, noise, heat, and cold. central portion is fluorescent orange
n When cab is used, a check should be while the 4 –in border is reflective dark
made to be sure that the exhaust pipe red.
from the engine is extended far n If a sow-moving vehicle move at night,
enough away from the cab to prevent it should be equipped with amber-
the exhaust fumes from entering the colored flashing lights.
cab.
n All tractors not equipped with strong
cabs should be provided with safety
roll bars.
Safety Precautions

n Power-driven implement.
• Ensure all guards are in place,
• Do not make adjustments on moving
parts or clear blockages while the
machine is running,
• Always disengage the PTO drive
when work ceases even if only
temporarily, and
• Even when the PTO is disengaged, it
is safer to stop the tractor engine
before working on the machine.

n Do not work under implement held in


the raised position by the tractor
hydraulic.

n Never leave the driving seat while


outfit is in motion.

n Always attach trailed implement to the


proper drawbar and never to a high
position on the rear of the tractor.

n Always use proper draw pin with a


locking device.

n Never leave self-lift implement in the


raised position.

n Allow adequate headlands especially


when using trailed implement near
bank, river, and ditches.

n Ensure that tractor is jacked securely


when altering wheels for row-crop
work.

n Avoid the temptation to clear tooth


harrows by lifting them manually
when the outfit is in motion and never
leave such harrows inverted in the
field after use.
REFERENCES

Smith, H. P. and L.H. Wilkes. 1976. Farm


Machinery and Equipment. 6th Edition.
Tata McGraw-Hill Publishing Company Ltd.
New Delhi. 488pp.
QUESTIONS and PROBLEMS

1. A safety gray color at the ASABE safety 5. Protective device which is usually
code is used for ____. made of rail, fence or frame to keep
a. floors or work areas around dangerous parts of a machine out of
machines and equipment reach inadvertently.
b. traffic and housekeeping areas a. Shield
c. areas with radiation hazard b. Casing
d. All of the above c. Enclosure
d. None of the above
2. When pipeline is painted with green
and white color, it indicates that the pipe 6. Protective device designed and fitted
contains ____. to prevent dangerous parts of a machine
a. liquid of inherently low hazard from being reached from the side.
b. fire extinguishing materials a. Shield
c. flammable or explosive materials b. Casing
d. All of the above c. Enclosure
d. None of the above
3. The ASABE classification for slow-
moving vehicles such as animal-drawn or 7. Protective device designed and fitted
motorized conveyance, including to prevent dangerous parts of a machine
implements in tow, should not exceed from being reached from all sides.
____ travel speed. a. Shield
a. 30 kph b. Casing
b. 40 kph c. Enclosure
c. 50 kph d. None of the above
d. None of the above
8. In ASABE’s safety code for laboratory
4. For safety purposes, slow-moving equipment and typical applications, the
vehicles such as agricultural machines safety yellow color stands for ____.
must be provided with ____. a. danger stop
a. rear bumper to protect the machine b. warning
from impact c. caution
b. identification emblem to guide d. None of the above
motorists that the machine is slow
moving
c. signal light when travelling
d. None of the above
XXI. PRODUCTION AND OPERATING COSTS OF
OF AGRICULTURAL MACHINES

• Careful appraisal and good planning Depreciation – reduction in value of an


must be undertaken before making an asset with the use and time. The actual
investment in an agricultural machines. total depreciation can never be known
until the equipment has been sold. It can
• Analysis on the cost to produce and be estimated by using different
operate agricultural machines are of computational methods, depending on
important when planning to build or to the objective.
operate them. Analyzing the costs that
will be incurred and compare it with the Interest – fee charged for the use of
different alternatives is most viable to do money invested in a machine. Simple
when investing in producing or just say interest on the average investment over
purchasing and operate agricultural the life of the machine can be added to
machines. the annual depreciation to estimate the
yearly capital cost of ownership.

Methods of Appraising Investment Repair and Maintenance Cost – cost for


the amount of use of the machine. It is
• Discounted Method - involves expressed in the accumulated form to
discounting and allows payment and reduce variability.
receipts occurring at different times in
the future to be converted to a common Fuel Cost – cost incurred for the fuel used
standard in terms of their present value. by the machine.

• Undiscounted Method - does not Labor Cost – cost incurred for the wage of
consider timing of cost and benefit flows labor in operating the machine based on
and the changing value of money over the 8-hour rate. For owner operator,
time. labor cost is determined from alternative
opportunities for use of time. This cost
varies depending on the prevailing cost of
Terminology labor in a certain area.

Cost Factors – total costs incurred in using Machine Custom Cost – amount paid for
the machine which include charges for hiring the equipment and operator
ownership and operation. services to perform a certain task.
• Fixed Cost – ownership cost that is
independent of use. Operating cost – cost incurred associated
• Variable Cost – cost associated with the with operating the machine which
operation of the machine, which vary includes labor, fuel, lubrication, and
directly with the amount of use. repair and maintenance costs.
Ownership Cost –cost which do not Specific Fuel Consumption – fuel
depend on the amount of use of the consumed by an engine to deliver a given
machine. It includes depreciation, amount of energy.
interest on investment, taxes, insurance,
and storage. Repair – restoration of a machine to
operative condition after breakdown,
Lease – contract for the use of a machine excessive wear, or accidental damage.
for an agreed period of time in return for
periodic payment. The ownership
remains with the lessor and the lessee Commonly Used Parameters in Analyzing
acquire the right of temporary possession the Economics of Agricultural Machines
and use of the machine.
1. Production Cost Analysis – How much
Obsolete – condition of a machine when is the cost to produce a certain
it is out of production and parts to repair agricultural machine?
and update it are not available from
normal suppliers, or it can be replaced by 2. Operating Cost analysis – How much is
other machine or method that will the cost to operate the machine?
produce a greater profit.
3. Payback Period – How long the
Price – market value per unit of a investment on the machine can be
machine. recovered?

Rent – short-term contract for the use of 4. Benefit-Cost Ratio – How much benefit
machinery in exchange for a fee. can be derived per unit investment on
the machine?
Gross – return for sale of product and the
value received for a service or product 5. Return on Investment – How many
before expenses are deducted. percent can be derived from the
investment of the machine?
Net – return for sale of a service or
product and the value received for a 6. Internal Rate of Return
service or product less all expenses
except income taxes.

Maintenance and Service – periodic


activities to prevent premature failure
and to maintain good functional machine
performance.
Production Cost Analysis Example 1

• Production Cost – cost incurred to An axial flow coconut decorticator is to be


build a machine, either one or several produced to provide a way farmers can
units. extract coco fiber and coco peat. The list
and prices of each material to be used are
• Production Cost Analysis – concerned given in Table 1. Determine the total
with analyzing the money value of material cost, labor, and fabrication cost.
inputs such as labor and raw Considering the different assumptions,
materials, which help in making what is the direct and indirect costs of the
decision as to the level of production machine? What is production cost of the
output. machine considering a profit margin and
taxes of 25% and 15%, respectively?
I. Direct Cost
1. Material Cost
a. Fabrication Materials
b. Standard Material
c. Consumables
2. Labor (60% of MC)
3. Contingency (10% of MC)

II. Indirect Cost


1. Electricity, etc. (15% of DC)
2. Administrative Cost (10% of
DC)

III. Fabrication Cost


Direct Cost + Indirect Cost

IV. Contingency (10% of FC)

V. Total Production Cost (FC +


contingency)

VI. Mark-Up (25% of Total Production


Cost)
VII. Taxes (3% percentage sales tax +
12% income tax)
VIII. Selling Price (Total Production Cost +
Mark-Up + Taxes)

Not included are sales


commission and other costs
Bill of Materials for the Decorticator
A. Fabrication Materials

Qty Unit Material Unit Cost Total Cost


(PHP/unit) (PHP)
1 plate MS plate 6mm, 1.2m x 2.4m 2,000.00 /sheet 2,000.00

1.5 plate MS plate 3 mm, 1.2m x 2.4m 1,200.00/sheet 1,800.00

1 length Angle Bar 6mm x 3 in. x 6m 300.00/length 300.00

1 pc CRS Hexagonal Bar 2 in. x 1.2m 800.00/foot 3,148.80

1 pc CRS shaft 3 in D x 70 cm 700.00/foot 1,607.20

1 plate MS Plate 10mm, 1.2m x 2.4m 10,000.00/plate 10,000.00

3 length CRS Shafting ¾ in D x 10m L 300.00/length 900.00

Total 19,756.00

B. Standard Materials

1 pc Engineering Plastic 6 in.D x 70 cm 300.00/foot 688.80

2 pcs Pillow block 2in. 320.00/pc 640.00

1 pc V-Pulley 2groove, 5 in. 220.00/pc 220.00

1 pc V-Pulley 2 groove, 4 in 190.00/pc 190.00

2 pc V-belt, type B approx B70 60.00/pc 120.00

1 unit 18 hp diesel 20,000.00 20,000.00

16 pcs Hexagonal bolt, nut, washer 12 mm x 1 in. 3.50/pc 56.00

10 pcs Hexagonal, bolt, nut, & washer 20mm x 2 in. 2.50/pc 25.00

Total 21,939.80

C. Consumables
10 kg Welding Rod 6013 110.00/kg 1,100.00

2 Liter2 Enamel paint 300.00/liter 600.00

2 Liter2 Lacquer thinner 120.00/liter 240.00

2 pcs Cutting disk 14 in. 190.00/pc 360.00

2 pc Grinding disc 4 in 80.00.00/pc 160.00

Total 2,460.00

Direct Cost Material Cost 44,155.80

Labor (60% Material Cost) 26,493.48

Contingency (10% of Material Cost 4,415/58 75,064.86

Indirect Cost Utilities (15% of Direct Cost) 11,259.73

Administrative (10% of Direct Cost) 7,506.49 18,766.22

Fabrication Cost Direct + Indirect Cost 93,831.08

Total Production Cost Fabrication Cost + 10% Contingency 103,214.19

Mark-Up 25% of Total Production Cost 25,803.55

Taxes 3% sales tax + 12% Income Tax 19,352.66

Selling Price Total Production Cost + Mark-Up = Taxes PHP148,370.49


Operating Cost Analysis Procedure ¡ Determine the total operating cost of the
machine by adding the total fixed costs
¡ Determine the annual fixed cost which and the total variable cost.
includes the depreciation, interest on ¡ Determine the operating cost of the
average capital investment, repair and machine by dividing the total cost by its
maintenance, and tax and insurance. field or functional capacity. This should
Usually, depreciation can be determined be expressed in P/hr, P/hectare, or P/kg.
using the straight line method with this ¡ Calculate the BEP by dividing the
formula purchase price by the difference between
the commercial rate of the machine and
D = (P-S) / L the computed operating cost.
¡ Calculate the PBP which is the length of
where: time from the start of purchasing the
D - depreciation, PHP/year machine until its net benefit equal cost.
P - principal, PHP
S - salvage value, PHP
L - life span, Years
I. Fixed Cost
• The salvage value is usually 10% of the a. Depreciation
I. t
purchase price of the machine. The life b. Interest on Investment
span depends on the machine and is c. Repair and Maintenance
usually 5 to 7 years for field equipment d. Insurance
while 10 to 15 years for farmstead Sub-total
equipment.
II. Variable Cost
• The interest on investment depends on a. Fuel (biomass, diesel, etc.)
the bank rate and is usually 24%. b. Labor
Repair and maintenance is usually 10% c, Diesel/Gasoline/Electricity
while the tax and insurance is 3%. Sub-total

All these are determined considering the III. Total Cost


investment cost for the machine. IV. Operating Cost
V. Savings
¡ Determine the variable costs which V. Payback Period
includes fuel, oil, and wage of the VI. Benefit-Cost Ratio
operator. It depends on the use and VII. Return on Investment
operation of the machine. The fuel
consumption and the oil requirement of
the machine can be determined from
the manufacturer ’s brochure or
catalogue. The cost of labor depends on
the prevailing wage for laborer in the
area, depending on the skills of the
operator.
Undiscounted Measures Discounted Measures

• Break-Even Point – level of operation at • Return on Investment (ROI) –


which the costs of operation equal the performance measure used to evaluate
revenues derived. To calculate the the efficiency or profitability of an
break-even point in units use the investment or compare the efficiency of a
formula number of different investments. A high
ROI means the investment’s gains
Break-even Point (units) = Fixed Costs / compare favorably to its cost. The basic
(Sales price per unit – Variable costs ROI formula is
per unit)
ROI = (Net Profit / Total Investment)
It is useful in determining the level or x 100
the number of sacks of palay that must
be dried in the dryer in one year to • Benefit-Cost Ratio (BCR) – ratio of the
cover the annual costs associated in total benefits derived from operating the
acquiring and operating the dryer. machine to the total cost incurred in the
operation. It is used to determine the
• Payback Period – amount of time it viability of cash flows from an asset or
takes to recover the cost invested on project. A BCR exceeding one indicates
the machine, e.g., dryer. In other that the machine is expected to generate
words, it is the length of time an incremental value. The formula is
investment reaches a break-even point.
It is expressed in years and fractions of n CFt [Benefits]
years. To calculate, use the formula Σ ----------------------
t=0 (1 + i)t
Payback Period = Amount invested on BCR = --------------------------------
the dryer /Annual net cash flow n CFt [Costs]
Σ ----------------------
It is useful in choosing among the t=0 (1 + i)t
investment alternatives when there is a where:
high degree of risk involved and/or CF - Cash Flow
financial resources are limited to decide i - Discount rate
for a short period. n - Number of periods
t - Period that the cash flow occurs
Operating Cost Analysis of Power Tiller
(As of December 31, 2012)
Field Capacity (Hactare per day)
Plower 1
Harrowing 3

Investment Cost (P) 1/


Power Tiller, Plow, Harrow & Trailer 58,000.00
8 hp Kubota Engine 62,000.00
Leveler 5,000.00
Total 125,000.00

Fixed Cost (P/day)


Depreciation 2/ 30.82
Interest on Investment 3/ 8.22 Make an operating cost analysis of a power
Repair and Maintenance 4/ 3.42 tiller to be used for custom plowing and
Insurance 5/ 1.03 harrowing of rice field. The power tiller
Sub-Total 43.49 can do plowing at 1 ha/day and harrowing
at 3 ha/day operation. The investment
Variable Costs (P/day)
requirement for the power tiller including
Fuel 6/ 451.20
Labor 7/ 250.00
plow, harrow, and trailer is PHP125,000.00.
Sub Total 701.20 The 8 horsepower diesel engine cost
PHP62,000.00 and a leveler of
Total Costs (P/day) 744.69 PHP5,000.00. Life span is 7 years with 10%
Operating Cost salvage value. Interest, repair and
Plowing Per Hectare 744.69 maintenance, and insurance is 24%, 10%,
Harrowing Per Hectare 248.23 and 3% of the investment cost,
Total Cost for Land Preparation
One Plowing& two harrowing 1241.16
respectively. The engine consumes 1.2 liter
Savings (P/Hectare) per hour at PHP47.00 per liter. One person
Tractor Tiller Preparation 8/ 2258.84 will operate the machine at P250.00 per 8-
Power Tiller Preparation 9/ 3258.84 hour day. If tractor tiller will be used
Payback Period (Ha) 38.36 followed by power tiller harrowing, the
Tractor Tiller Preparation 55.34 custom rate is P3,500 per hectare whereas
Power Tiller Preparation 38.36 if power tiller and harrowing will be
1/ Life Span - 7 years
employed, the custom rate is P4,500 per
2/ Straight line with 10% salvage value
3/ 24% of the investment cost hectare. What is the cost of using the
4/ 10% of the investment cost machine in preparing the field. How much
5/ 3% of the investment costs savings can be derived when investing in
6/ 1.2 liters per hour at P47 per liter the tiller with tillage implement. What is
7/ One (1) person @ P250.00 per 8 hour-day the payback period of the machine?
8/ At P3,500 per hectare if tractor tiller is used
followed by power tiller harrowing
9/ At P4,500 per hectare if power tiller is used for
plowing and harrowing
A rice husk gasifier that can generate 3kWe electricity is
to be purchased from a supplier. The investment cost for
the machine is PHP55,000.00. It has an estimated life
span of 5 years. Considering an interest on investment
of 24%, repair and maintenance of 10%, and insurance of
3%, what is the total fixed cost of the machine? If the
cost of rice husks fuel is PHP1.00 per kg and the cost of
gasoline to start up the operation of the gasifier is PHP55
per liter at a rate of 09 liter per hour, what is the
variable cost of operating the gasifier if it also consumes
1 liter of engine oil (PHP200 per liter) per every 600
hours. The gasifier parasitic load is 0.9 kWe. Cost of
electricity from the grid is P10.50 per kW-hr.

Operating Cost Analysis of 3kWe Rice Husk Gasifier Plant


(As of March 2012)
Investment Cost (P) 1/ 3 kWe Power plant
6 hrs 8 hrs 12 hrs
Gasifier and Gas Conditioning 35,000.00 35,000.00 35,000.00
Engine and Generator 20,000.00 20,000.00 20,000.00
Total 55,000.00 55,000.00 55,000.00

Fixed Cost (P/day)


Depreciation 2/ 27.12 27.12 27.12
Interest on Investment 3/ 7.23 7.23 7.23
Repair and Maintenance 4/ 3.01 3.01 3.01
Insurance 5/ 0.90 0.90 0.90
Sub-Total 38.27 38.27 38.27

Variable Costs (P/day)


Fuel 6/ - - -
Gasoline 7/ 28.88 28.88 28.88
Engine Oil 8/ 2.00 2.67 4.00
Sub Total 30.88 31.54 32.88

Total Costs (P/day) 69.15 69.82 71.15


Generation Cost per kW-Hour 9/ 5.76 4.36 2.96
Savings per Day 10/ 56.85 98.18 180.85
Payback Period (Years) 2.65 1.53 0.83
1/ Complete set; 5 years life span
2/ straight line with 10% salvage value
3/ 24% of the investment cost
4/ 10% of the investment cost
5/ 3% of the investment costs
6/ P1.00 per kg of rice husks
7/ 0.9 liter per hour for 35 minutes @ P55/liter per day operation
8/ P200.00 per liter @ 1 liters per engine-oil change every 600 hours
9/ At a parasitic load of 0.9 kw
10/ At a prevailing cost of electricty of P10.50 per kw-hr
REFERENCES

ASAE. ASAE Standard 1997. 44th Edition.


American Society of Agricultural
Engineers.

Maranan, C. L. Farm Economics:


Undiscounted Techniques for Investment
Appraisal. Break-Even Point (BEP) and
Payback Period (PBP). Handout Sheet.
Agricultural Engineering Training Course.
The International Rice Research Institute.
May-June 1986. 12pp.
QUESTIONS and PROBLEMS

1. Sum of money required to acquire 7. Cost associated with the production of


machines. a machine but not classified as materials
a. Production cost or labor, like transport of materials,
b. Overhead cost consumables, etc.
c. Investment cost a. Production cost
d. None of the above b. Overhead cost
c. Operation cost
2. Cost of using a machine per unit time, d. None of the above
area, output, etc.
a. Fixed cost 8. Cost associated with the utilization of a
b. Variable cost machine which includes depreciation,
c. Operating Cost interest on investment, repair and
d. All of the above maintenance, insurance, etc.
a. Fixed cost
3. Costs of materials, labor, and overhead b. Variable cost
in producing a machine. c. Total cost
a. Material cost d. None of the above
b. Production cost
c. Investment cost 9. Cost associated with the operation of a
d. None of the above machine, particularly in terms of fuel,
electricity, etc. consumed and of labor
4. Costs of fabricated and standard employed.
components of a machine. a. Fixed cost
a. Material cost b. Variable cost
b. Production cost c. Total cost
c. Manufacturing cost d. None of the above
d. All of the above
10. Difference between the investment
5. Cost of manpower in fabricating a cost and the salvage value divided by the
machine. life span of the machine.
a. Fabrication cost a. Depreciation
b. Overhead cost b. Interest on investment
c. Labor cost c. Insurance
d. None of the above d. None of the above

6. Cost associated with the use of a 11. Amount charged for a unit of
machine. machines based on manufacturing cost
a. Operating cost plus tax payable, and incorporating a
b. Rental cost certain percentage for mark-up.
c. Custom cost a. Investment cost
d. All of the b. Administrative cost
c. Selling price
d. None of the above
12. Sum of the production cost, selling, 17. Internal rate of return of a project is
and administrative cost. analyzed to see whether it is viable or not.
a. Production cost Based on this, which of the statements
b. Overhead cost below is true?
c. Manufacturing cost a. The higher the internal rate of return,
d. None of the above the more viable is the project.
b. The lower the internal rate of return,
13. Itemized list and cost estimates of the more viable is the project.
fabricated and standard materials needed c. The internal rate of return must be
in the fabrication of a machine. equal with the prevailing rate to make it
a. Bill of lading viable.
b. Bill of payment d. None of the above
c. Bill of materials
d. All of the above 18. Useful life of a machine before it
becomes unprofitable for its original
14. Number of years required to recover purpose due to obsolescence or wear.
the cost of investment for a machine. a. Economic life
a. Benefit cost ratio b. Life span
b. Internal rate of return c. Profitable life
c. Payback period d. All of the above
d. None of the above
19. Ability of a machine to perform a
15. Indicator that attempts to summarize timely operation that optimizes return
the overall value of money in using a from crop considering quality and quantity
machine. of the product.
a. Percentage benefit over cost a. Operating time
b. Cost-Benefit Ratio b. Timeliness
c. Benefit-Cost Ratio c. Functional time
d. None of the above d. None of the above

16. Profitability measure used to evaluate 20. In trade circles, CIF stands for _____.
a. Cash-In-Fist
the efficiency of an investment on a b. Cost, Insurance and Freight
machine, which is determined by dividing c. Cost-In Freight
the return of an investment by the cost of d. None of the above
the investment times 100.
a. Payback period 21. Cost of machine including shipment.
a. Free-on-board
b. Benefit cost ratio
b. Freight-on-board
c. Return on investment c. Cost-in-freight
d. None of the above d. None of the above
21. Amount paid for hiring equipment 25. Using the following additional data,
and for operating services in performing what is the total variable cost that will be
a certain task. incurred for the thresher n Item 23? Size
a. Custom cost of engine = 10 hp,
b. Operating cost Specific fuel consumption = 0.85 lb/BHP-hr,
c. Rental cost Cost of fuel = P13.60/liter,
d. None of the above Fuel specific gravity = 0.76, and
Salary of Laborers = P100.00/day-person.
22. Change in the value of a machine due a. 159, 882.00/year
to wear and tear. b. 234, 765.00/year
a. Cost reduction c. 456,854.00/year
b. Depreciation d. None of the above
c. Lost of value
d. None of the above 26. What is the threshing cost for the
thresher in Item 23?
23. A thresher has a 30cavans-per-hour a. P3.85/cavan
capacity. Three men are required to b. P2.87/cavan
operate it. Other data are as follows: c. P2.78/cavan
Purchase price = P40,000 (including d. None of the above
engine); Utilization = 1,500 hours per
year, 8 hours/day; Estimated life = 7
years. What is the depreciation cost
using straight line method?
a. 6, 568.35/year
b. 3, 784.56/year
c. 5, 142.86/year
d. None of the above

24. What is the total fixed cost that will


be incurred for the thresher in Item 51 if
the cost of money is 21% per annum?
a. 10, 542.86/year
b. 13, 542.86/year
c. 12, 542.86/year
d. None of the above

You might also like